You are on page 1of 257

‫بسم الله الرحمن الرحيم‬

‫هذه السأئلة تم جمعها و حلها من قبل مجموعة من خريجي كلية الطب بجامعة القصيم ‪ .‬وهم )‬
‫‪ -1‬محمد الحربي ‪ -2 ,‬عبدا فيصل الخميس ‪ -3 ,‬عبدا المنيفي ‪ -4 ,‬مهند الحربي ‪-5 ,‬‬
‫عبدا صالح الخميس ‪ -6 ,‬هاني الجهني ‪ -7 ,‬ياسأر الوابلي ‪ -8 ,‬ماجد الحربي ‪ -9 ,‬لؤي العبدي‬
‫‪ -10 ,‬منصور الصغير (‬

‫أتمنى أن تكون هذه السأئلة جيده من حيث الترتيب و التنسأيق ‪ ,‬والجابات الصحيحة ‪ ,‬علما ا بأن‬
‫الجابات هي عبارة عن اجتهاد قد تحتمل الخطأ ‪ .‬وهذا الملف يحتوي على قرابة اللف سأؤال هذه‬
‫السأئله تم جمعها من خلل موقع الكاديميه الطبيه و من بعض الذين سأبق لهم الختبار من‬
‫‪ .‬مختلف المناطق ‪ .‬فشكرا ا لكل من قام بكتابة السأئله‬

‫تمنياتي للجميع بالتوفيق‬


‫أخوكم محمد الحربي‬
1) 25 year teacher have fear attack and worry before enter the class ( I forgot all the
scenario) what is the initial treatment:
a. Selective serotonin reuptake inhibitor
b. Tricyclic depressant
c. Beta blocker
d. -----
e. ------
-This is a case of social phobia and the initial treatment is answer a (SSRIs).
…………………………………………………………………………………………………………………………………………………

2) The best initial TTT for depression is:


a. SSRIs
b. Tricyclic depressant
c. MAO inhibitors
d. Beta blocker
e. -----------
-The correct answer is a (SSRIs).
…………………………………………………………………………………………………………………………………………………

3) Using the following classification :

Risk factor Case (disease) Non case total


Present A B A+b
Absent C D C+d
Total A+C B+D
Relative risk of those with risk factor to those without risk factor is:
a. A/A+B , C/C+D
b. A/A+B
c. C/C+D
-The correct answer is a.

Positive predictive value (PPV) : a/ (a+b)


Negative predictive value (NPV) : d / (C+d)
Sensitivity : proportion of people with disease who have a positive test: a / (a+c)
Specificity : proportion of people without disease who have a negative test : d/ (b+d)

Relative risk (RR) : incidence in expose ÷ incidence in unexposed: ( a / (a+b) ) / (c/


(c+d) )
Attributable risk : incidence in expose ‫ ـــ‬incidence in unexposed: ( a / (a+b) ) ‫( ــ‬c/ (c+d) )
Odd ratio (OR) : Odds that a disease person is exposed ÷ Odds that a nondisease person
is exposed : ad / bc

4) 50 year old Man presented to ER with sudden headache, blurred of vision, and eye
pain. The diagnosis is:
a. Acute glaucoma
b. Acute conjunctivitis
c. Corneal ulcer
d. -----
-The correct answer is a.

…………………………………………………………………………………………………………………………………………………
5) Which heart condition is tolerable during pregnancy:
a. Eisenmenger syndrome
b. Aortic stenosis
c. Severe mitral regurge
d. Dilated cardiomyopathy with EF 20%
e. Mitral stenosis and the mitral area is 1 cm (or mm).

-the correct answer is c.


…………………………………………………………………………………………………………………………………………………
6) Diffuse abdominal pain “in wave like” and vomiting. The diagnosis is:
a. Pancreatitis
b. Appendicitis
c. Bowel obstruction
d. Cholelithiasis

The correct answer is c. (colicky pain: is the pain that comes in waves “wave-
like” and it is associated with bowel obstruction, cholelithiasis, nephrolithiasis)
…………………………………………………………………………………………………………………………………………………

7) Which type of contraceptive is contraindicative in lactation:


a. OCPs
b. Mini pills (progesterone-only birth control pills)
c. IUD
d. Condom
e. Depo-Provera (progesterone-only birth control injection)

-The correct answer is a. because combined OCPs decrease the quantity of milk.
Progesterone –only contraceptive are not believed to decrease the quantity of
milk.

8) A long scenario about patient with polydipsia ad polyuria. I don’t remember the
scenario but they mention osmolality in urine and serum, measurement of Na.
I encourage you to read about central VS nephrogenic diabetes insipidus.

- Q IS NOT COMPLETE .
(Diabetes Insipidus) : failure to concentration of urine as a result of central or
nephrogenic ADH. Present with polyurea, polydepsia and hyponatremia.
Diagnosis central or nephrogenic by administration of Desmopressin acetate (DDAVP) ,
synthetic analog of ADH, can be used to distinguish central from nephrogenic DI.
1- Central DI : DDAVP challenge will decrease urine output and increase urine
osmalirity
2- Nephrogenic DI : DDAVP change will not significant decrease urine output
…………………………………………………………………………………………………………………………………………………
9) A 5 year old child came with earache on examination there is fluid in middle ear and
adenoid hypertrophy. Beside adenoidectomy on management, which also you should
do:
a. Myringotomy
b. Grommet tube insertion
c. Mastidectomy
d. Tonsillectomy
e. -----
-The correct answer is a.

- Myringotomy (is used for bulging acute otitis media)


-Grommet tube insertion (is used for recurrent acute otitis media)
…………………………………………………………………………………………………………………………………………………

10) How the randomized control study become strong or of good validity: ( sorry I forgot
the answers)

Q IS NOT COMPLETE
-By doing blinding and allocation concealment. The best way to increase the power of
the study is to increase the sample size.
…………………………………………………………………………………………………………………………………………………

11) Woman with postpartum depression, beside the medical TTT, whch should be include
in therapy:
a. Family therapy or support
b. -------
-Nonpharmacological treatment strategies are:cognitive behavioral
therapy,psycho educational or support group,increase partner
support,additional help with child care.
12) Classic Scenario of stroke on diabetic and hypertensive patient. What is the
pathopysiology of stroke:
a. Atherosclerosis
b. Anyresm
c. -----
-The correct answer is a.
…………………………………………………………………………………………………………………………………………………

13) Middle aged patient with an acyanotic congenital heart disease the X-ray show
ventrical enlargement and pulmonary hypertension:
a. VSD
b. ASD
c. Trancus arteriosus
d. Pulmonary stenosis

-The correct answer is b, bcause the pt. in middle age and acynotic.
…………………………………………………………………………………………………………………………………………………

14) Best food in travelling is:


a. Boiling water
b. Water
c. Ice
d. Partial cocked fish and meat
e. ---
-The correct answer is a.
…………………………………………………………………………………………………………………………………………………

15) Mechanism of vitamin C in wound healing :


a. Epithiliazation
b. Aerobic fibroblast synthesis
c. Collagen synthesis
d. Enhance vascularization
e. ----
-The correct answer is c.
…………………………………………………………………………………………………………………………………………………
16) Open globe injury . TTT is:
a. Contineuos antibiotic drops
b. Continuous wate and NS drops
c. Continuous steroids drops
d. Sterile cover and the refered
e. -----
-The correct answer is d.
17) Adolescent female counseling on fast food. What you should give her:
a. Ca + folic acid
b. Vit C + folic acid
c. Zinc + folic acid
d. Zinc + Vit C
-The correct answer is a.
…………………………………………………………………………………………………………………………………………………
18) What is the TRUE about backache with osteoporosis:
a. Normal x ray vertebra exclude the diagnosis
b. Steroid is beneficial TTT
c. Vit D defiency is the cause
d. Another TTT I don’t remember

-The correct answer is c , because vit. D deficiency is cause osteomacia.


Osteomalcia is considering secondary causes of osteoporosis. Choice d not
mention may be more accurate answer.
…………………………………………………………………………………………………………………………………………………
19) Adolescent female with eating disorder and osteoporosis
a.Weight gain
b.Vit D
c.Bisphosphonates
d.--------
-The correct answer is a.
…………………………………………………………………………………………………………………………………………………
20) Fourth degree hemorrhoids, Management is:
a. Hemoridectomy
b. band ligation
c. sclerotherapy
d. fiber diet
-The correct answer is a.

- Classification of internal hemorrhoid :


First degree: hemorrhoid do not prolapsed
Second degree: hemorrhoid prolapsed upon defecation but spontaneously reduce
Third degree: hemorrhoid prolapsed upon defecation, but must be manually reduce
Fourth degree: hemorrhoids are prolapsed and cannot be manually reduce

-treatment:
First & Second: life style modification (fiber dite)
Third: life style modification with (band ligation, sclerotherapy or cryotherapy) , if
Failed go to surgery.
Fourth: surgery (hemorrhiodectomy)
21) nulligravida at 8 weak gestational age, follow up for genetic screening, she refused the
invasive procedure but she agree for once screening , what is the appropriate action
now:
a. do ultrasound
b. 1st screening
c. 2nd screening
d. 3rd screening
e. Amncentesis
-The answer is correct B
…………………………………………………………………………………………………………………………………………………
22) 70 year old male with chronic Hepatitis B virus antigen carrier. The screening of choice
is:
a. Alfaprotien + liver ultrasound
b. Alfaprotien + another tumor marker
c. Abdominal CT + abdominal ultrasound
d. ----
-The correct answer is a.
…………………………………………………………………………………………………………………………………………………
23) 35 year old smoker , on examination sown white patch on the tongue, management:
a. Antibiotics
b. No ttt
c. Close observation
d. ---------
-This is a case of leukoplakia and the management includes:ask the pt. to stop
smoking, do a biopsy for the lesion; if there is pre-cancerous changes or cancer
in the biopsy ; surgical excision should be done.
…………………………………………………………………………………………………………………………………………………

24) Regarding GDM:


a. Screening for GDM at 24 t0 28 weeks
b. Diet control is always successful TTT
c. Screening at 8 weeks
d. ---------
-The correct answer is a.
…………………………………………………………………………………………………………………………………………………

25) After doing CPR on child and the showing asystole:


a. Atropine
b. Adrenaline
c. Lidocane
d. ------
-The correct answer is b.
26) Scenario of trauma , on face examination there is shifted mouth angle, loss of
sensation of ant. Third of tongue, which CN is affected:
a. Facial nerve
b. Trigeminal nerve
c. ----
-The correct answer is b , facial nerve for taste ant. Third of tongue, but
trigeminal nerve for sensation .
…………………………………………………………………………………………………………………………………………………

27) On examination of newborne the skin show papules or (pastules) over erythema base:
a. transient neonatal pustular melanosis
b. erythema toxicum neonatorum
c. ----
-The correct answer is b.
…………………………………………………………………………………………………………………………………………………

28) The most common causes of precocious puberty:


a. Idiopathic
b. Functional ovary cysts
c. Ovary tumor
d. Brain tumor
e. Adenoma

-The correct answer is a.


…………………………………………………………………………………………………………………………………………………

29) Mother worry about radiation from microwave if exposed to her child. What you tell
her:
a. Not all radiation are dangerous and microwave one of them
b. Microwave is dangerous on children
c. Microwave is dangerous on adult
d. ----
- In international study : all microwave is dangerous on children ( very vague Q )

…………………………………………………………………………………………………………………………………………………
30) Earlier sign of puberty in male is:
a. Appearance of pubic hair
b. Increase testicular size
c. Increase penis size
d. Increase prostate size
e. ---
-The correct answer is b.
31) Question about nutritional marasmus on definition.(protein energy malnutrition).

Marasmus is a form of severe protein-energy malnutrition characterized by


energy deficiency. A child with marasmus looks emaciated. Body weight may be
reduced to less than 80% of the average weight that corresponds to the height.
Marasmus occurrence increases prior to age 1, whereas Kawasaki occurrence
increases after 18 months. It can be distinguished from Kawasaki in that Kawasaki
is protein wasting with the presence of edema. The prognosis is better than it is
for Kawasaki.

…………………………………………………………………………………………………………………………………………………

32) What is the most risk of antihypertensive drugs on elderly patient:


a. Hypotension
b. Hypokalemia
c. CNS side effect
d. –
-The correct answer is a. because orthostatic hypotension will increase the risk
of fall and fractures in elderly.
…………………………………………………………………………………………………………………………………………………

33) Asymptomatic woman with trichomonas:


a. Treat if symptomatic
b. Treat if she is pregnant
c. Treat her anyway
d. –

-The correct answer is c.


Single dose of metradinazole for symptomatic and asymptomatic pt. and treatment
partner (because it is sexual transmitted disease). But, not treatment of
asymptomatic trichomniasis in first trimester of pregnancy. (Contraindication in
pregnancy)
…………………………………………………………………………………………………………………………………………………

34) What is the risk of GDM on her life later:


a. DM type 1
b. DM type 2
c. Impaired fasting glucose

-The correct answer is b.

35) What is the major sign that can tell you that patient have polycythemia vera rather
than secondary polysythemia:
a. Hepatomegaly
b. Splenomegaly
c. Venous engorgement
d. Hypertension
e. ----
-The correct answer is b.
in polycythemia vera there is clonal proliferation of a pluripotent marrow stem cell.
Also, level of erythropietine may be useful distinguish PCV, level are low, from other
causes of polycythemia. Secondary polycythemia is associated with excessive
dirusis, severe gastroenteritis and burn.
…………………………………………………………………………………………………………………………………………………
36) What is the deficient vitamin in infantile beri beri :
a. B1(thiamine)
b. C
c. E
d. Niacine
-The correct answer is a.

* VITAMINES DEFICIENCY :
Vit. A : night blindness & dry skin
Vit, B1 (thiamine): Beriberi (polyneuritis, dilated cardiomayopathy, edema)
Vit. B2 (riboflavin): angular stomatitis, cheilosis. Corneal vascuarization.
Vit. B3 (nacine) : pellagra ( Diarrhea, Dermatitis, Dementia )
Vit. B5 (pantothenate): dermatitis, enteritis, alopecia, adrenal insufficiency
Vit. B6 (pyridoxine): convulsion, hyperirritability
Vit B12 (cobalamin): macrocytic megaloblastic anemia, neurologic symptoms
FOLIC ACID : macrocytic megaloblastic anemia without neurologic symptoms.
Vit C : scurvy ( swollen gums, brusing, anemia, poor wound healing)
Vit D : rickets in children , osteomalasia in adult
Vit. E : increase fragility of RBC
Vit. K : neonatal hemorrhage, increase PT & PTT, normal BT
………………………………………………………………………………………………………………………………………………
37) 17 year boy admit to involve in recurrent illegal drug injection , what the screening test
to do:
a. HIV
b. Hepatitis B
c. Hepatitis C
d. -----
-All of the above choices are correct.?

38) 20 year old male found to have hepatitis b surface antibodies :


a. Previous vaccination
b. Previous infection
c. Active infection
d. ----
-The correct answer is a.

HBsAg : indicate carrier state.


HBsAb : indicate provide immunity to HBV
HBcAg: associated with core of HBV
HBcAB: during widow period, HBcAb-IgM indicate recent disease

…………………………………………………………………………………………………………………………………………………

39) Most common cause of secondary hypertension in female adolescent is:


a. Cushing syndrome
b. Hyperthyroidism
c. Renal disease
d. Essential HTN
e. Polycystic ovary disease

-The correct answer is c.


Most common causes of secondary HTN in young female is renal artery stenosis
( caused by fibromuscular dysplasia )
40) clomiphene citrate:
a. induce ovulation
b. ------
-The correct answer is a.
…………………………………………………………………………………………………………………………………………………

41) Scenario about perimenstrual depression syndrome ????

Premenestral syndrome : define as a symptoms complex of physiological emotional


symptoms sever enough to interfere with everyday life and occur cyclical during luteal
phase of menses
Premenestral dysphoric disorder : is a sever form of pf premenestral syndrome
characterized by severe recurrent depressive and anxity symptoms with premenestral
( luteal phase) onest that remit a few days after the start of menses.

…………………………………………………………………………………………………………………………………………………

42) Child present with stiffing neck, fever, headach. You suspect meningitis what is your
initial TTT should be:
a. Tobramycin
b. Levoflaxicine
c. Peneciline(ampicillin)
d. Doxycycline
 I remember there is no vancomycine, ceftriaxone or dexamethasone
on the choices.

-The correct answer is c.

AGE Causative organism Treatment

< 1 MONTH GBS, E coli Ampicillin + cefotaxime or gentamicin

1-3 MONTHS S.pneumonia, H.infulenza Vancomycin + cefotaxime or cefriaxone


meningocci

3 MONTHS - ADULT Pneumoccoci, meningococci Vancomycin + cefotaxime or ceftiaxone

>60 YEAR\acoholism Pneumococi, meningococci Ampicillin + vancomycin + ceotaxime or


Gram –ve bacilli ceftiaxone

43) A patient have tender, redness nodule on lacriminal duct site. Before referred him to
ophthalmologist what you will do:
a.
Topical steroid
b.
Topical antibiotics
c.
Oral antibiotics
d.
Nothing
– e.
The correct answer is c.(this is a case of dacrocystitis).
…………………………………………………………………………………………………………………………………………………

44) About DM in KSA:


a. about < 10 %
b. Most of the pt of insulin dependant type
c. female more affected with type 2 DM
d. most of NIDDM are obese

-The correct answer is d.


…………………………………………………………………………………………………………………………………………………

45) Major aim of PHC in Saudi Arabia :


a. To provide comprehensive maternal & child health
b. ------------
-other aims are:community participation, immunization, referral, chronic disease
management, prescribing, health education, management of communicable
disease and environmental health.
…………………………………………………………………………………………………………………………………………………
46) 17 y.o,she missed her second dose of varecila vaccine,the first one about 1 y ago what
you'll do:
a. give her double dose vaccine
b. give her the second dose only
c. revaccinate from start
d. see if she has antibody and act accordingly

-The correct answer is b.


…………………………………………………………………………………………………………………………………………………
47) there is outbreak of difteria and tetanus in community , regaring to pregnant woman:
a. contraindication to give DT vaccine
b. if exposed , terminate pregnancy immediately
c. if exposed , terminate after 72 hour
d. give DT vaccine anyway

-The correct answer is d.


48)All of the following are live vaccine except:
a. MMR
b. Oral plio
c. Varicella
d. Hepatitis B vaccine
e. BCG
-The correct answer is d.
…………………………………………………………………………………………………………………………………………………

49) Pt with scoliosis, you need to refer him to the ortho when the degree is:
a. 5
b. 10
c. 15
d. 20

-The correct answer is d.


…………………………………………………………………………………………………………………………………………………

50) 87 year old who brought by his daughter, she said he is forgettable, doing mess
thing in room , do not maintain attension , neurological examination and the
investigation are normal
a. Alzheimer disease
b. Multi-Infarct Dementia
c. ---

-The correct answer is a.


…………………………………………………………………………………………………………………………………………………

51) Mechanical intestinal obstruction


a. Nasogastric tube decompression

…………………………………………………………………………………………………………………………………………………

52) Patient with cancer. You want to break bad news, which of the following is the
answer:
a. Inform his family
b. Find out how much the patient know about it
c. Let social service inform him
d. Don’t tell him

-The correct answer is b.

53) HIV patient has hemorrhagic lesion in the mouth and papules in the face. Skin
biopsy show spindle cells and vascular structures:
a. Kaposi sarcoma
b. ………………….

Kaposi sarcoma : malignant multifocal neoplasm characterized by vascular tumors of


skin and viscera , caused by Human herpesvirus 8 (HHV8) , most commom associated
with AIDS patient. Lesion usally papules . in biobsy show spindle shape of cell.
…………………………………………………………………………………………………………………………………………………

54) Patient with retrosternal chest pain , barium swallow show corkscrew appearance
a. Achalasia
b. Esophagitis
c. GERD
d. Diffuse esophageal spasm

-The correct answer is d.

- Alchalasia : show esophagus dilation with (bird beak) tapering of distal esophagus
-Diffuse esophagus spasm: show corkscrew appearance.
…………………………………………………………………………………………………………………………………………………

55) Most common cause of intra cerebral hemorrhage:


a. ruptured aneurysm
b. Hypertension
c. Trauma
d. –

-The correct answer is b.

56) Rt upper qudrent pain and tenderness , fever, high WBC , jaundice, normal hepatic
marker
a. Acute cholecyctitis
b. Pancreatitis
c. Acute hepatitis
d. –

-The correct answer is a.


…………………………………………………………………………………………………………………………………………………
57) 5 y.o child with h.o fever and swelling of the face ant to the both ears (parotid
gland enlargement) what is the most common complication:
a. Labrynthitis
b. meningitis
c. encephalitis
d. orchitis.
-The correct answer is b.
Orchitis: (this one is the most common complication in postpubertal males-
adolscents and adults-). This pt. is prepuberty .

…………………………………………………………………………………………………………………………………………………
58) classic symptoms of tension headache .

Tension headach : is the most common type of headach diagnosed in adult. Present
with tight, band-like pain . may be generalized or most intense in the frontal,
occipital, and neck region. Usually occurs at the end of day. Treatmet avoidance of
exacerbating factors, NSAID and acetaminophen.

…………………………………………………………………………………………………………………………………………………
59) prophylaxis of Asthma
a. oral steroid
b. inhaler steroids
c. inhaler bronchodilator B agonists

-The correct answer is b.


…………………………………………………………………………………………………………………………………………………
60) 30 year woman with dysmenorrhea, menorrhagea, infertility, and on examination
found immobile mass on uterosacral ligaments :
a. uterine fibroid
b. endometriosis
c. ---
-The correct answer is b
61) Classic symptoms of primary dysmenorrheal

Primary dysmenorrhea: menestral pain associated with ovulatory cycle in abcense


of pathological finding.
Secondary dysmenorrhea: Menestral pain wich organic causes exist, most common
cused by endometriosis, adenomyosis, tumor and fibroid.
…………………………………………………………………………………………………………………………………………………

62) Newly married woman complain of no pregnancy for 3 month with unprotective
sexual intercourse :
a. Try more ( infertility is defined as no pregnancy for one year)

…………………………………………………………………………………………………………………………………………………

63) Younger diabetic patient came with abdominal pain, vomiting and ketones smelled
from his mouth. What is frequent cause:
a. Insulin mismanagement
b. Diet mismanagement
c. ----
-The correct answer is a.. But if the infection is one of the answers , you should
choose it because the most common cause of DKA is infection.
…………………………………………………………………………………………………………………………………………………

64) Cause of syncope in aortic stenosis


a. Systemic hypotension

………………………………………………………………………………………………………………………………………………
65) On stroke management: except
a. IV fluid not include dextrose
b. Diazepam for convulsion
c. No indication of anticonvulsive drugs
d. ----

-The correct answer is c.


…………………………………………………………………………………………………………………………………………………
66) Patient use illegal drug abuse and the blood show RNA virus. Which hepatitis
a. A
b. B
c. C
d. D

-The correct answer is c.


67) Classic symptoms and signs of infectious mononucleosis (EBV) ?

Most common occur in young adult, usually cused by EBV , transmission through
exchange of body fluid include saliva. Present with triad ( fever, sorethroat,
lymphadenopathy) , also, may present tonsillar exudates, splenomegaly,
maculopapular rash and bilateral upper eyelid edema. Management is mostly
supportive and corticosteroids are indicated for airway compromise due to tonsillar
enlargement.

…………………………………………………………………………………………………………………………………………………

68) Treatment of EBV ( in scenario there patent with tonsiller exudates,


lymphadenopathy, splenomegaly) :
a. Oral acyclovir
b. Oral antibiotic
c. IM or IV acyclovir
d. Supportive TTT
e. Observation

-The correct answer is d.


…………………………………………………………………………………………………………………………………………………

69) 25 year old woman with weight loss, heat intolerance, irritable ….
a. Hyperthyroidism

…………………………………………………………………………………………………………………………………………………

70) Patient with coloured pastules around his mouth, organism show herpes simplex
type 1, what is the treatment:
a. Oral antiviral
b. Iv antiviral
c. Supportive
d. –

-The correct answer is a.


…………………………………………………………………………………………………………………………………………………

71) Acute otitis media , initial TTT:


a. Amoxicilline

72) Man with sudden onset of scrotal pain , also had Hx of vomiting, on examination
tender scrotom and there is tender 4 cm mass over right groin, what you will do:
a. Consult surgeon
b. Consult urologist
c. Do sonogram
d. Elective surgery

-The correct answer is b , if expected tortion of tests , don't wast time on


radiological study
…………………………………………………………………………………………………………………………………………………
73) Most common symptoms of renal cell carcinoma is
a. Hematuria
b. Abdominal mass
c. Flank pain
-The correct answer is a.
…………………………………………………………………………………………………………………………………………………
74) Link the ttt with organism:
a. Shegella  metronidazole
b. Salmonella  erythromycin
c. Combylobacter  amoxacilline
d. Gardia  I foregut ( but I remember it is antibiotic)

-All of the above answers are wrong.?


…………………………………………………………………………………………………………………………………………………
75) Medical student had RTA systolic pressure is 70 mmhg, what you will do next in
management:
a. IV fluid therapy
b. ECG
c. Abdominal U/S
d. ---
-The correct answer is a.
…………………………………………………………………………………………………………………………………………………
76) 20 year old male had been stabbed on midtriceps , ‫ممممم ممممم مممم ممممم‬
‫ ممممم ممممم مممم مممم مممم‬.
On microscopic examination of this greenish fluid show gram positive cocci in
chains:
a. Streptococcal gangrene
b. Chlostrideal gangrene
c. Fournier’s gangrene
d. meningocemia
e. –
-The correct answer is a.
77) Patient around his nose there are pastules, papules and telangiectasia lesions. The
diagnosis is:
a. Rosacea

…………………………………………………………………………………………………………………………………………………

78) Child with radial head dislocation, what is the next in management:
a. Reduction
b. x ray
c. MRI
d. –

-The correct answer is a.


…………………………………………………………………………………………………………………………………………………

79) In cervical LNs there are well differentiated thyroid cells, during operation you find
no lesion on thyroid what will you do next
a. Total thyroidectomy
b. Total thyoidectomy + radical cervical LNs dissection
c. Total thyroidectomy + specific LNs dissection
d. Thyoid lobectomy with -----

-The correct answer is b.


…………………………………………………………………………………………………………………………………………………

80) Irritable bowel syndrome , ( they mention a specific mechanism” I do not


remember” that produce which symptom
a. Vomiting
b. Diarrhea
c. Constipation
d. Abdominal pain
???????????? Q IS NOT CLEAR
…………………………………………………………………………………………………………………………………………………
81) Free fluid accumulate in abdominal cavity cause:
a. Hypovolemic shock
b. Cardigenic shock
c. Sepsis
d. Emesis
e. ---

-The correct answer is c.


82) Woman with cyclic bilateral nodularity in her breast since 6 month, on
examination there is 3 cm tender mobile mass in her breast : what you will do next
a. FNA with cytology
b. Mammogram
c. Biopsy
d. Follow up for next cycle
e. Observation

-The correct answer is d. ( I'm not sure )


…………………………………………………………………………………………………………………………………………………

83) Most common symptoms of soft tissue sarcoma :


a. Paralysis
b. Ongrowing mass
c. Pain
d. ---

-The correct answer is b.


Ongrowing mass: painless and slow-growing.
…………………………………………………………………………………………………………………………………………………

84) the most common symptom in placenta abruption is


a. Vaginal bleeding
b. Uterine tenderness
c. Uterine contractions
d. Fetal distress

The correct answer is a

Placental abruption is mainly a clinical diagnosis with all the above findings. the
most common symptom is dark red vaginal bleeding with pain during the third
trimester of pregnancy (80%) and abdominal or uterine tenderness (70%). Bleeding
may occur at various times in pregnancy Bleeding in the first trimester of pregnancy
is quite common and may be due to the following: miscarriage (pregnancy loss)
ectopic pregnancy (pregnancy in the fallopian tube) . Bleeding in late pregnancy
(after about 20 weeks) may be due to the following: placenta previa or placental
abruption.
85) Female presented with vaginal discharge, itching, and on microscope showed
mycoleous cells and spores. This medical condition is most likely to be associated
with:
a. TB
b. Diabetes
c. Rheumatoid Arthritis

The correct answer is b

Vaginal thrush is a common infection caused by a yeast called Candida albicans.


Vulvovaginal candidiasis is usually secondary to overgrowth of normal flora Candida
species in the vagina. Conditions that interrupt the balance of normal vaginal flora
include: antibiotic use, oral contraceptives, contraceptive devices, high estrogen levels,
and immunocompromised states such as diabetes mellitus and HIV. Women are prone to
vaginal thrush between puberty and the menopause because, under the influence of the
hormone estrogen, the cells lining the vagina produce a sugar and yeasts which Candida
albicans are attracted to. That is why thrush is rare before puberty.
…………………………………………………………………………………………………………………………………………………
86) Primigravida in her 8th week of gestation, presented to your clinic wanting to do
genetic screening, she declined invasive procedure . the best in this situation is

a. Amniocentesis
b. 1st trimester screening
c. 2nd trimester screening
d. Ultrasound

The correct answer is b


…………………………………………………………………………………………………………………………………………………

87) Trichomoniasis is classically have:


a. Clue cells
b. Greenish frothy discharge

The correct answer is b

Trichomoniasis is caused by the flagellated protozoan Trichomonas vaginalis; it's the


most common curable sexually transmitted disease in the world. It usually presents
with frothy yellow-green vaginal discharge, strong-unpleasant odor, pain during
urination and sexual intercourse.
…………………………………………………………………………………………………………………………………………………

87) Obsessive neurosis:


a. Treatment is east
b. Clomipramine doesn’t not work
c. Mostly associated with severe depression
d. Can be cured spontaneously
the correct aswer is c
88) Patient came to you complaining of hearing voices, later he started to complain of
thought get into his mind and can be taken out
a. SCZ
b. Mood
c. Mania
d. Agoraphobia

The correct answer a


…………………………………………………………………………………………………………………………………………………

89) Female had history of severe depression, many episodes, she got her remission for
three months with Paroxitine ( SSRIs) .. now she is pregnant .. your advise
a. Stop SSRi's because it cause fetal malformation
b. Stop SSRi's because it cause premature labor
c. Continue and monitor her depression
d. Stop SSRIs

The correct answer is c or a

In general, SSRI have the least side effects then other classes of antidepressants.
Some SE: Sleep disturbance, dizziness, sexual dysfunction, Nervousness, and
diarrhea
……………………………………………………………………………………………………………………………………………
90) Hallucinations and Paranoia:

a. SCZ
b. Mood
c. Mania
d. Phobia

The correct answer is a


…………………………………………………………………………………………………………………………………………………

91) Female presented with thirst and polyurea.. all medical history is negative and she
is not know to have medical issues.. .she gave history of being diagnosed as
Bipolar and on Lithium but her Cr and BUN is normal. What is the cause of her
presentation
a. Adverse affect of lithium
b. Nephrogenic DI
c. Central DI

The correct answer is a

Nephrogenic diabetes insipidus is characterized by a decrease in the ability to


concentrate urine due to a resistance to ADH action in the kidney. Nephrogenic
diabetes insipidus can be observed in chronic renal insufficiency, lithium toxicity,
hypercalcemia, hypokalemia.
92) Most common medical problems encountered in Primary care is:
a. Coryza
b. UTI
c. HTN

The correct answer is a


…………………………………………………………………………………………………………………………………………………

93) Regarding diabetes mellitus in pregnancy :


a. Prevelance of diabetes mellitus in pregnancy is 10%
b. Glucose screeing is best in 24-28 week
c. Diabetic and non diabetic have same perinatal mortality
d. Gestational diabeted can be diagnosed by abnormal FGS test

The correct answer is b


…………………………………………………………………………………………………………………………………………………

94) Left red eye, watery discharge, photo phobia, peri-auricular non-tender lymph
nodes .. Dx
a. Bacterial conjctvitis
b. Viral conjctvitis

The correct answer is b


…………………………………………………………………………………………………………………………………………………

95) One of the Anti-psychotics causes ECG changes , Leukopenia, drooling :

a. Respiredone
b. Colzapine
c. Amisulpride

The correct answer is b


…………………………………………………………………………………………………………………………………………………
96) One of the following decrease chance of colon cancer :

a. Zinc
b. Vit. E
c. Vit C
d. Folic acid

????????

Both C, E, Beta carotene all are Antioxidants, they are correct but C is more
likely. A big dose of vitamin C fights the big "C." some others: Fiber ,Vitamin C,
Calcium, Vitamin E, Selenium.
97) Best sentence to describe specifity of screening test,is the population of people
who :
a. Are negative of disease, and test is negative
b. Are positive of disease, and test is negative
c. Are positive comparing to total other people
d. Negative disease , positive test
e. Positive disease , negative test

The correct answer is a

Specificity measures the proportion of negatives which are correctly identified


(e.g. the percentage of healthy people who are correctly identified as not having
the condition).
Sensitivity (also recall rate) measures the proportion of actual positives which
are correctly identified (e.g. the percentage of sick people correctly identified as
having the condition).
…………………………………………………………………………………………………………………………………………………

98) Heavy smoker came to you asking about other cancer, not Lung cancer, that
smoking increase its risk:
a. Colon
b. Bladder
c. Liver

The correct answer is b

smoking-associated cancers are lung, head &neck (like esophagus and larynx),
bladder and kidney, pancreatic, cervical and stomach.
…………………………………………………………………………………………………………………………………………………

99) Mid clavicle fracture :


a. Surgery is always indicated if fracture is displaced
b. Figure-8-dressing has better outcomes than simple sling
c. Figure-8-dressing is strongly indicated in patient with un-union risk
d. Both figure-8 and simple sling has similar outcomes

the correct answer is d


- simple sling has been to give the same result as a figure-8 (more comfort and
fewer skin problem).
…………………………………………………………………………………………………………………………………………………

100)patient complains of "can't breathe air in one nostril "; on examination showed
edematoud mucosa structure, best to give initially :
a. Corticosteroids
b. Decongestants
c. Alfa-adrenergic blockers

The correct answer is a


101)Pediatric came to you in ER with wheezing, dyspnea, muscle contraction ( most
probably asthma), best to give initially is :
a. theophillin
b. Albuterol neubelizors
c. oral steroids

the correct answer is b

………………………………………………………………………………………………………………………………………………

102) Female presented with complain of neck pain and occipital headache , no other
symptoms , on X-ray has cervical spine osteophyes and narrow disks :
a. cervical sponylosis ?

??????
…………………………………………………………………………………………………………………………………………………

103) Patient complaining of pain at night when he elevated his arm, tingeling on lateral
arm side and lateral three fingers , Dx
a. brachial plexus neuropathy
b. shoulder impengment syndrom
c. brachial artery thrombophebitis
d. thoracic outlet problem

the correct answer is d

Branchial plexus neuropathy is characterized by acute onset of intense pain in the


shoulder or arm followed shortly by focal muscle weakness.

Impingement syndrome, swimmer's shoulder, and thrower's shoulder, is a clinical


syndrome which occurs when the tendons of the rotator cuf muscles become
irritated and inflamed. This can result in pain, weakness and loss of movement at
the shoulder. The pain is often worsened by shoulder overhead movement and may
occur at night, especially if the patient is lying on the affected shoulder.

Thoracic outlet syndrome: numbness and tingling in the fingers; pain in the neck,
shoulder, and arm; weakness of the arm and dropping things from the hand;
worsening of the symptoms when elevating the arm to do such things as comb or
blow dry one's hair or drive a car; and coldness and color changes in the hand. The
symptoms are often worse at night or when using the arm for work or other
activities.
104) Young adult presented with pain on lateral elbow, tingeling of lateral arm, he
plays Squash:
a. carbel tunnel
b. tennis elbow

the correct answer is b


-Lateral epicondylitis( inflammation of common extensor tendon ) also known as
(tennis elbow, shooter's elbow and archer's elbow is a condition where the outer
part of the elbow becomes sore and tender. It is commonly associated with
playing tennis and racquet sports.

-Medial epicondylitis (inflammation of common flexor elbow) also know (golfer


elbow)
…………………………………………………………………………………………………………………………………………………

105) male came to you complaining of sudden progressive decreasing in vision of left
eye over last two/three days, also pain on the same eye, on fundoscopy optic disk
swelling was sees , Dx :
a.central retinal artery occlusion
b.central retinal vein occlusion
c.optic neuritis
d.macular degeneration

they said the correct answer is c

 Ask about symptoms of temporal arteritis in the older population. Patients complain of
sudden, painless, nonprogressive vision loss in one eye. History of headaches, jaw
claudication, scalp tenderness, proximal muscle and joint aches, anorexia, weight loss, or
fever may be elicited.
 Some patients may reveal a history of amaurosis fugax involving transient loss of vision
lasting seconds to minutes but which may last up to 2 hours. The vision usually returns
to baseline after an episode of amaurosis fugax.

 Central retinal Artery occlusion: presenting complaint is an acute sudden painless


unilateral loss of vision. Shows a classic cherry red spot

 Central retinal vein occlusion: usually sudden painless variable visual loss; the fundus
may show retinal hemorrhages, dilated tortuous retinal veins, cotton-wool spots,
macular edema, and optic disc edema.

 Optic neuritis: Major symptoms are sudden loss of vision (partial or complete), or
sudden blurred or "foggy or washed out" vision, and pain on movement of the affected
eye. Many patients with optic neuritis may lose some of their color vision in the affected
eye (especially red). The optic disk becomes swollen

 Macular degeneration is a leading cause of permanent painless irreversible vision loss in


the elderly.
Retinal Detachment; Symptoms are decreased peripheral or central vision, often described
as a curtain or dark cloud coming across the field of vision. Associated symptoms can
include painless vision disturbances, including flashing lights and excessive floaters.

106) unilateral headache, exaggerated by excersice and light , Dx :


a. migraine
b. cluster headach
c. stress headache

the correct answer is a


…………………………………………………………………………………………………………………………………………………

107) 70 years old with progressive demntia , no personality changes , neurological


examination was normal but there is visuodeficit , on brain CT shower cortex
atrophy and ventricular dilatations :
a. multi micro infract dementia
b. alzehimer demenita
c. parkinsonism dementia

the correct answer is b

 alzehimer dementia :
most common cause of dementia. age and family history are risk factors for AD.
Etiology unknown but toxic b-amyloid deposit in brain. Present with amnesia for
newly acquired information is usually the first presentation, followed by language
deficit , acaluia, depression, agitation and finally apraxia(inability to perform skilled
movement). Diagnosis by exclusion that can be definitive diagnosis only on autopsy:
suggested by clinical feature and by progressive cognitive course without
substantial motor impairment. MRI & CT may show atrophy , venticule enlargement
and can rule out other causes. On brain microscopy amyloid plaques and
neurofibrially tangle. Death usually occurring secondary to aspiration pneumonia .
treatment by supportive therapy for Pt. and family , and cholinesterase inhibitor .

 multi mico infarct dementia ( vascular demensia )


dementia associated with history of strok. Criteria for vascular dementia include
presence of dementia and 2 or more of the following:
1- focal neurological signs
2- symptoms onset that was abrupt , stepwise, or related to strok
3- brain imaging showing evidence of fold infarction or extensive deep white matter
changes secondary to chronic ischemia.

…………………………………………………………………………………………………………………………………………………
108) 70 years old with progressive dementia , on brain microscopy amyloid plaques
and neurofibrillary tangles are clearly visible also Plaques are seen : Dx
a. lewy dementia
b. Parkisonism
c. Alzehimer

The correct answer c


109) after bite, pediatric patient presented with abdominal pain and vomiting , stool
occult blood , rash over buttock and lower limbs , edema of hands and soles ,
urine function was normal but microscopic hematurea was seen:
a. Lyme
b. Henoch-Schonlein Purpura

The correct answer is b


Henoch-Schönlein purpura (HSP) is a small-vessel vasculitis characterized by
purpura, arthritis, abdominal pain, and hematuria.
…………………………………………………………………………………………………………………………………………………

110) for the above disorder (Q 109 ), which one is considered pathological
a. gross hematurea
b. microscopic hematurea
c. rashes
d. ..

The correct answer is a (I'm not sure )

…………………………………………………………………………………………………………………………………………………

111) Young adult presented with painless penile ulcer rolled edges .. what next to do :
a. CBC
b. Darkfeild microscopy
c. Culturing

The correct answer is b

Syphilis also known as "great imitator" is a sexually transmitted disease caused


by the spirochete bacteria Treponema pallidum. Classically presents as single
painless non-itchy skin ulceration with sharp borders. T pallidum is too small to
be seen under the light microscope. So use darkfield microscopy when sores are
present. Blood tests can confirm the presence of antibodies. The antibodies
remain in your body for years, so the test can be used to determine a current or
past infection.
…………………………………………………………………………………………………………………………………………………

112) Diabetic female ger 24h-urine proteinn is 150mg ,,


a. start on ACEIs
b. refer to nephrologist
c. do nothing , this is normal range
the correct answer is a

113) which prevent or decrease incidence of getting post herpetic neuralgia


a. Amitriptylin
b. Acyclovir
c. Varicella vaccination

The correct answer is c

All are true, but if I have to choose the best it will be varicella vaccination,
because it completely prevents.

…………………………………………………………………………………………………………………………………………………

114) Adult Polycystic kidney mode of inheritance:


a. Autosomal dominant

…………………………………………………………………………………………………………………………………………………

115) Female came with complain of diahrrea in the last 6 months, she lost some
weight, she reported that mostly was bloody , when you preformed
sigmoidoscopy you found fragile mucosa with bleeding ,Dx
a. colon cancer
b. Chron's
c. Ulcerative colitis
d. Gastroenteritis
e. Hemrrohids

The correct answer is c

…………………………………………………………………………………………………………………………………………………

116) Anal fissure commonest site


a. Posteriorly

…………………………………………………………………………………………………………………………………………………

117) mother gave bitrh of baby with cleft lip and palate, she want to get pergnant
again what is the percentage of recurrence
a. 1%
b. 4%
c. 15%

The correct answer is b

118) 4 years old girl presented with her parents to er with sore throat and seroangious
vaginal discharge with no pain what is most propable cause,
a. Candida
b. Foreign body
c. Chlamedia
d. Gonococci
e. streptococcus
The correcr answer is e

 Pediatric vaginal discharge:


infectious vulvovaginitis: present with malodorous , yellow green, most common caused by -1
group A streptococcus. ( may be present with sexual abus "STDs" )
2-foreign body
3-candidal infection: may associated with diabetes … measure glucose.
4-Sarcoma botryoids (rhabdomyosarcoma ): malignant lesion appearance of "bunches of graps"
within vagina
………………………………………………………………………………………………………
………………………………………

119)patient coplaining of pain along median nerve ditribution , And positive tinel sign
treatment include casting of both hand in what position
a. Dorsiflexion
b. plantar flexion
c. extention
d. Dduction

The correct answer is c


…………………………………………………………………………………………………………………………………………………

120) dermatomyosistis what is true


a. distal muscle weakness
b. Underlying malignancy
c. Generalized ??? Skin rash

The correct answer is c

 Polymositis & dermatomyositis :


Polymyositis : progressive , systemic tissue disease characterized by immune-mediate
striated muscle inflammation, present with symmetric progressive proximal muscle
weaknes and pain .

DERMATOMYOSITIS : present polymyositis plus cutaneous involvement, heiotrop rash


(violaceous periorbital rash) , shawl sign ( rash involving the shoulder, upper chest and
back ) , Gottron"s papule ( popular rash with scale ).

-both : increase serum CK and anti-Jo antibodies . muscle biopsy uscle fiber and
inflammation.

121) pt with hypertrophic subaortic stenosis referred from dentist before doing dental
procedure what is true
a. 50 % risk of endocarditis
b. 12 % risk of endo carditis
c. No need for prophylaxis
d. post procedure antibiotic is enough

the correct answer is c


…………………………………………………………………………………………………………………………………………………

122) pt want to quit smoking you till him that symptoms of nicotine withdrawal appear
after
a. 1-2 days
b. 2-4 days
c. 5-7 days
d. 8- 10 days

The correct answer is a

…………………………………………………………………………………………………………………………………………………

123) pt taking bupropion to quit smoking what is SE


a. Arrythmia
b. Seizure
c. xerostomia
d. Headache

The correct answer is b

…………………………………………………………………………………………………………………………………………………

124) 14 years old girl complaining of painless vaginal bleeding for 2-4 days every
3Weeks to 2 months ranging from spotting to 2 packs per day; she had 2ry sexual
ccc 1 year ago and had her menstruation since 6 months on clinical examination
she is normal sexual ccc, normal pelvic exam appropriate action
a. OCP can be used
b. You should ask for FSH and prolactin level
??????????????

125) pt want to do dental procedure , he was dx to have mitral valve prolapse clinically
by cardiologist, he had never done echo before what is appropriate action
a. Do echo
b. No need for prophelaxis
c. give ampicillin
d. Give amoxicillin calvulinic

I think I will do Echo, but this are some of the information I found so I cant decide.

This is from MYO clinic

Doctors used to recommend that some people with mitral valve prolapse take antibiotics
before certain dental or medical procedures to prevent endocarditis, but not anymore.
According to the American Heart Association, antibiotics are no longer necessary in most
cases for someone with mitral valve regurgitation or mitral valve prolapse.

Still, if you've been told to take antibiotics before any procedures in the past, check with
your doctor to see how these new recommendations apply to you.

This is from MedicinNet

The vast majority of patients with mitral valve prolapse have an excellent prognosis and
need no treatment. For these individuals, routine examinations including
echocardiograms every few years may suffice. Mitral regurgitation in patients with mitral
valve prolapse can lead to heart failure, heart enlargement, and abnormal rhythms.
Therefore, mitral valve prolapse patients with mitral regurgitation are often evaluated
annually. Since valve infection, endocarditis, is a rare, but potentially serious
complication of mitral valve prolapse, patients with mitral valve prolapse are usually
given antibiotics prior to any procedure which can introduce bacteria into the
bloodstream. These procedures include routine dental work, minor surgery, and
procedures that can traumatize body tissues such as colonoscopy, gynecologic, or
urologic examinations. Examples of antibiotics used include oral amoxicillin and
erythromycin as well as intramuscular or intravenous ampicillin, gentamycin, and
vancomycin.
…………………………………………………………………………………………………………………………………………………

126) ttt of cyclical mastalgia


a. OCP, analgisc, NSAID, Fat reduction, and magnisuem

Mastalagia : painful breast tissue that can be cyclic and usually associated with
hormonal change, often bilateral . management : stop current hormonal
therapy , reassurance, stop smoking, fat reduction, analgesic, NSAID, OCP.

127) 4years old child what can he do


a. Copy square and triangle
b. Speak in sentences
b. ..

The correct answer is a

…………………………………………………………………………………………………………………………………………………

128) baby can sit without support, walk by holding fourniture. Pincer grasp, pull to
stand how old is he
a. 8 months
b. 10 months
c. 12 month
d. 18 month

The correct answer is b

…………………………………………………………………………………………………………………………………………………

129) repeated question about Alzheimer disease

 alzehimer dementia :
most common cause of dementia. age and family history are risk factors for AD.
Etiology unknown but toxic b-amyloid deposit in brain. Present with amnesia for
newly acquired information is usually the first presentation, followed by language
deficit , acaluia, depression, agitation and finally apraxia(inability to perform skilled
movement). Diagnosis by exclusion that can be definitive diagnosis only on autopsy:
suggested by clinical feature and by progressive cognitive course without
substantial motor impairment. MRI & CT may show atrophy , venticule enlargement
and can rule out other causes. On brain microscopy amyloid plaques and
neurofibrially tangle. Death usually occurring secondary to aspiration pneumonia .
treatment by supportive therapy for Pt. and family , and cholinesterase inhibitor .

…………………………………………………………………………………………………………………………………………………
130) repeated Q also about multiinfarct dementia
 multi mico infarct dementia ( vascular demensia )
dementia associated with history of strok. Criteria for vascular dementia include
presence of dementia and 2 or more of the following:

1- focal neurological signs


2- symptoms onset that was abrupt , stepwise, or related to strok
3- brain imaging showing evidence of fold infarction or extensive deep white matter
changes secondary to chronic ischemia.
131) 73 year patient complain of progressive loses of memory with decrees in
cognition function . C.T reveal enlarge ventricle and cortical atrophy , diagnosis is

a- Alzheimer
b-multi infarct dementia
c- multiple sclerosis
d-????????

The correct answer is a

 Alzheimer (Dx by exclusion. Its associated with progressive memory loss,


decreased cognition function , & enlarged ventricles with cortical atrophy)
 multi infarct dementia (NOT progressive & it has focal neurological abnormality)

 multiple sclerosis (recurrent relapsed & complete remission. Its associated with
demylenation of gray-matter)

…………………………………………………………………………………………………………………………………………………

132) 62 female with –ve pap smear you should advice to repeat pap smear every:
A- 6m
b- 12m
c- 18m
d- no repeat

The correct answer is d

Screening pap smear:


1- starting at age 21 years or no more than 3 years after becoming sexually active.
2- women > 30 years who have three consecutive normal test screening ( 1 / 3yeasr).
3- screening should be discontinue for women > 60-70 years who have had 3 or more
normal pap smear.

…………………………………………………………………………………………………………………………………………………

133) all following are criteria of chronic fatigue syndrome except


b-???????

chronic fatigue syndrome:


characterizes by profound mental and physical exhaustion. In associated with
multiple system and neurospsychiatric symptoms that last at least 6 mounth. Must
be new ( not life long ) , must not be relived by rest, and must result in greater than
50% reduction in previous activity. Presentation with 4 or more of the following :
poor memory / concentration, myalgia, arthalagia, sore throat, tender lymph node,
recent onset headach, unrefreshing sleep, excessive tiredness with exercise.
Treatment by : cognitive and excercise therapy .also, diet, physiotherapy, dietary
supplements, antidepressants

134) 12 year boy with sinsoneurl hear loose , C.T scan show mass (site in brain???) so
diagnosis is
a- aqustic neroma
b-meningioma
c- barotraumas
d????????

The correct answer is a (I'm not sure )

- aqustic neroma (benign tumor of cranial nerve VIII. Mostly occur between age of 30-60 years. ,
Mostly unilateral except if it's associated with neurofibromatosis in which its bilateral)
-meningioma (mostly benign,  with age, more with female. Its occur in the cerebellopontin)
- barotraumas (mostly in the divers. The damage occur due to  pressure)

..……………………………………………………………………………………………….………………………………………………

135) 50 y with uncontrolled diabetes ,complain of black to brown nasal discharge. So


diagnoses is
a- mycomyosis
b- aspirglosis
c-foreign body
d????????

The correct answer is a

- mycomyosis (fungal infection caused by Mycorales, affect nasal sinus & lungs, .
…………………. characterized by black nasal discharge, Dx by biopsy).
………………………………………………………….……………………………………………………………………………………

136) 55 y complain of dyspnea, PND with past history of mitral valve disease diagnosis is
a-LT side HF
b- RT side HF
c- pnemothrax
d-P.E

The correct answer is a


……………………………………………………………………………………………………………………………………
137) clonidin is decrease effect of
a- benzotropin (anticholinirgic for Parkinson. Not affected by Clonidin)
b-levo dopa (for Parkinson. Changed in the brain to Dopamen. Clonidin  the
effect of Levodopa through Unknown mechanism)
c-rubstin?????

the correct answer is b


Clonodin is α2 agonist used to TTT hypertension. α2 receptor in the brain cause  of
both COP & peripheral resistant .

138) 2 y baby with gray to green patch in lower back, no redness or hotness, diagnosis
is
a- child abuse
b-no ttt need
c- bleeding tendency
d????

The correct answer is b (I'm not sure )

I think it is " Mongolian spot " , visible in 6 month and normally disappear to 3-5
years. No need treatment.

…………………………………………………………………………………………………………………………………………………

139) 15y boy with unilateral gyncomastia your advice is


a- my resolve spontiniously
b-there is variation from person to person
c-decrease use of soda oil or fish oil
d-???????????

the correct answer is a

- uni- or bilateral gynecomastia occur normally in newborn & at puberty

…………………………………………………………………………………………………………………………………………………

140) 6m baby with mild viral diarrhea , ttt by ORS as


a-100ml/kg for 4 hour then 50 ml/kg /day after
b-50>>>>>>>>>>>>>>>>.50>>>>>>>>>>
c-100>>>>>>>>>>>>>>>>100>>>>>>>>
d-50>>>>>>>>>>>>>>>>>100>>>>>>

141) 36 y female with breast mass mobile and change with menstrual cycle , no skin
dimple or fathering. Your advice is
a-repeat exam after 2 cycle
b-make biopsy
c-fine needle aspiration (there is NO singe of breast cancer. It is Fibroadenoma.
Just do FNA to exclude cancer & relive the pt)
d-oral contraception

The correct answer is a (I'm not sure )

…………………………………………………………………………………………………………………………………………………

142) 50y female with breast cancer and CA125 elevate. So elevation due to
a-breast cancer
b-associate with ovarian cancer
c-due to old age
d-normal variation

The correct answer is a , beause this pt. high risk of breast cancer.

- CA125 is a tumor marker mostly used for ovarian Ca, but it's also used with
endometrial, fallopian, breast, & GIT Ca

…………………………………………………………………………………………………………………………………………………

143) 25y female with bradicardia and palpitation. ECG normal except HR130 and apical
pulse is 210 .past history of full ttt ovarian teratoma, so your advice is
a- struma ovari should be consider
b-vagal stimulate should be done
c- referred to cardiology

The correct answer is a

…………………………………………………………………………………………………………………………………………………

144) pt with alcohol drinked complain of headache , dilated pupil hyperactivity,


agitation .he had history of alcohol withdrawal last week so ttt is
a-diazepam
b-naxtrol
c-haloperidol
d?????????
the correct answer is a

-diazepam (for seizure  given 1st occur in the 1st 12h).


-haloperidol (for hallucination  given 2nd occur in the 1st 2 days).

145) 50y man with chronic psychosis and not complains for ttt .your advice
a- depot haloperidol or floxtin
b-oral lorasepam
c-oral buspiron
d-???????

The correct answer is a

…………………………………………………………………………………………………………………………………………………

146) 15y boy appear patch in rt lower leg these patch is clear center , red in peripheral,
no fever no other complain so diagnosis
a-contact dermatitis
b-tinea corpora
c- lyme disease
d-???????

The correct answer is b

-tinea corpora: worm fungal infection which transmitted by contact skin & has
clinical picture as in the question .

…………………………………………………………………………………………………………………………………………………

147) old pt with of IHD complain for 2 mon of redness in lower leg and pulse dim-
inched in dorsalis pedis these redness increase in dependant position and limp is
cold and no swelling ,diagnosis is:
a-arterial inssuficncy
b-thrmbophibitis
c-cellulites
d??????

the correct answer is a

-thrombophilibitis: present with pain, swelling and redness .


-celluitis: present with redness, hotness and tender.

…………………………………………………………………………………………………………………………………………………

148) pt with heart disease complain of LL ischemia your advice


a-referred to cardiology
b-""""""""""""vascular surgery
c- start heparin
d-??????????/

??????
???

149) female after sexual attack on exam hymen tear in


a-2 o'clock
b-4"""""""
c-6""""""
d-8""""""""

the correct answer is c

-tear appear between 5 and 7 o'clock

…………………………………………………………………………………………………………………………………………………

150) 7y boy complain of limb. CT show a vascular necrosis in epiphysis of femur your
advice
a- surgical ttt
b-splint for 6m
c- physiotherapy
d????????

Q not complete: but with these MCQs , the correct answer is b

This case is "perth's diseas" : vascular necrosis of femoral head. 5- 10 years . usually self
limiting with symptoms lasting< 18 months. Present with painless limb , limited
abduction and internal rotation.
Treatment: 1- observation if there is limited femoral head involvement or full ROM .
2- if extensive or if decrease ROM, consider bracing, hip abduction with cast.

…………………………………………………………………………………………………………………………………………………

151) pt with trachoma in eye . for prevention you should


a- water
b- """""""""""""""""+eradication of organism
c- mass ttt
d???????????

the correct answer is a

-trachoma in the eye is a bacterial infection caused by Chlamydia trachomatis which


is transmitted by poor haygen & contaminated H2O. TTT by antibiotic as
erythromycin & Doxycycline. Surgery to prevent scar

…………………………………………………………………………………………………………………………………………………

152) your advice to prevent plaque disease is


a-hand washing
b-rodent eradication
c-spry insect side
d-?????????? ?????

153) pt with severe headache and decrease in visual acuity ,pupil is dilated, so ttt
a-pilocarpin drop and ophthalmology referred
b-ergotamine
c-NSID
d???????

the correct answer is a

- this is closed angle glaucoma which characterized by sudden severe headache, red
eye,  visual acuity, & dilated pupil. Pilocarpin is parasympathomimmic which help
in relive the pain

…………………………………………………………………………………………………………………………………………………

154) main ttt of non inflammatory acne is


a-ritonic acid
b-clindmycin;
c-azalic acid
d-erythromycin

the correct answer is a

-clindamycine & erythromycin are treatment of inflammatory acne.


. –azalic acid : treatment of non-inflammatory & inflammatory acne.
. – ritonic acid : treatment of sever acne & non-inflammatory
…………………………………………………………………………………………………………………………………………………

155) pregnant with insulin dependant with good control, so to decrease risk of
congenital disease
a-good metabolic control before pregnancy
b-"""""""""""""""""""""""1st trimester
c-""""""""""""""""""""""""2nd """""
d-""""""""""""""""""""""""3rd """""""

the correct answer is b(I'm not sure )

…………………………………………………………………………………………………………………………………………………

156) female not married with normal investigation except FBS=142. RBS196. so ttt
a-give insulin subcutaneous
b-advice not become married
c-barrier contraceptive is good
d- BMI control

the correct answer is d

157) diapetic pt come to you with disturbance in conscious RBS : 65. so main drug that
cause hypoglycemia:
a-sulphnylurea
b-bugunid
c-acabos α-glucosidase inhibitor
d-pheniform

the correct answer a

-sulphnylurea ( insulin secretion, so, it can cause hypoglycemia)


-bugunid ( insulin sensitization, so, can not cause hypoglycemia)
-acabos α-glucosidase inhibitor ( used for postprandial hyperglycemia, so, can
. . not cause hypoglycemia)
-pheniform ( insulin sensitization, so, can not cause hypoglycemia)
…………………………………………………………………………………………………………………………………………………

158) 6m boy with fever you should give antipyretic to decrease risk of
a-febrile convulsion
b- epilepsy
c- disseminate bacteria
d???????????????

The correct answer is a

…………………………………………………………………………………………………………………………………………………

159)picture in computer appear vesicle , bulla and erythama in chest skin so ttt
a- acyclovir cream
b-betamethzone cream
c-floclvir
d- erythromycin

multiple choice not clear.


This case is "herpes zoster" . treatment of herpes zoser are antiviral, analgesic.
Antiviral are ( systemic ) and include: acyclovire, famciclovire, acyclovire. In multiple
choice there is "floclvir" and there is not drug have this name. I think the writer Q
written in wrong spelling. I think the choice c is famciclovire and it is correct
answer. Famciclovir: 500 mg T.I.D for 7 days

160) pt with scale in hair margin and nasal fold and behind ear with papule and irregular
erythema so ttt is
a-nizoral cream
b- atovit
c- acyclovir
d-antibiotic

the correct answer is a ( this is seborrheic dermatitis )

…………………………………………………………………………………………………………………………………………………

161) 14y girl with athralgia and photosensitivity and malar flush. And protinurea so
diagnosis is
a-RA
b-lupus nephritis
c-UTI
d-??????????

The correct answer is b

…………………………………………………………………………………………………………………………………………………

162) question about diarrhea and Yesinia bacteria

Yersiniosis : it is infectious disease caused by Yesinia bacteria. There are 3 types of


Yesinia bacteria : 1- Y.entercolitis , 2- Y.pseudotuberculosis, 3-Y. pestis.
Y.enetercolitis cause bloody diarrhea, terminal ileitis and mesenteric adenitis.
Diagnosis by serological : rise in antibody titer . Treatment : usually self limiting and
tetracycline for sever infection.

…………………………………………………………………………………………………………………………………………………

163) paraplegia pt with ulcer in lower back 2+2 cm and lose of dermis and epidermis
these ulcer in stage
a- I
b-II
c-III
d-IV

the correct answer is b

-stage I : non-blanchable redness that NOT subside after relive of the pressure
-stage II : damage to epidermis & dermis but NOT deeper
-stageIII : subcutaneous tissue involvement
-stageIV : deeper than subcutaneous tissue as muscles & bones

164) PREGNANT LADY prim at labor pain , on exa cervical in stage I of labor so pain
management is
a-morphine IM
b-epidural anesthesia
c-general"""""""""
d-local"""""""

The correct answer is b

…………………………………………………………………………………………………………………………………………………

165) Psycatric pt on antipsychotic drug most drug that lead to impotence with
antipsychotic is
a- proprnlol
b-NSAI
c-ACEI
d-?????????

the correct answer is a


…………………………………………………………………………………………………………………………………………………

166) man present with painless ulcer in his penis with indurate base and everted edge
so diagnosis is
a- syphilis
b- gonorrhea
c- choncroid
d- HSV

The correct answer is a

- syphilis : painless ulcer in the penis


- gonorrhea: STD by bacterial infection cause pain & discharge but NO ulcer
- choncroid: STD by bacterial infection cause pain & discharge but NO ulcer
- HSV : STD by virus that cause painful ulcer with discharge.
…………………………………………………………………………………………………………………………………………………

167) man have long history of urethral stricture present with tender right testis and
WBC in urine so diagnosis is
a-epddimorchitis
b- testicular torchin
c- varicosel
d-??????

the correct answer is a


-epddimorchitis : occur with UTI & urinary retension)
- testicular torchin : sudden acute severe testicular pain with elevated
. transversely ling testes)
- varicosel : due to  intra-abdomenal pressure.

168) man use saldinafil (Viagra), to prevent hypotension you should not use
a-nitrate
b-B blocker
c-ACIE
d-CCB

the correct answer is a

…………………………………………………………………………………………………………………………………………………

169) female complain of painless odorless and colorless vaginal discharge that appear
after intercourse so ttt
a-give antibiotic
b-douche after intercourse
c- cervical cancer should be consider
d-may be due to chronic salpingitis

the correct answer is b

Color Smell Pain OR Itching Other

 during
Normal vaginal
Clear OR milky Odorless NO ovulation, sex,
discharge
breastfeeding

Bacterial White-gray OR
Fishy Yes  during sex
vaginosis yellow

Yes especially
Trichomoniasis Watery Or yellow
during urination
White-cheesy Yes especially
Candida
thick sticky during sex
…………………………………………………………………………………………………………………………………………………

170) UTI>14 day, most probably cause pylonphritis


a-,05%
b-,5%
c-5%
d-50%

Difficult Q , This Q referral to urologist.

…………………………………………………………………………………………………………………………………………………

171) patient complain of right iliac fossa mass so diagnosis


a-diverticulitis
b-appendicitis
c-pancrtitis
d-chrons disease
- the correct answer is d
172) pt with long history of UC on endoscopes see polyp and cancer lesion on left colon
so ttt
a-ttt of anemia
b-left hemicolctomy
c-total colctomy
d- remove polyp

…………………………………………………………………………………………………………………………………………………

173) female with hair on different site of body and refuse intake of food and BMI<18
and feel as body is fat so diagnosis
a-anorexia nervosa
b-bulimia nervosa
c-body dismorphic syndrome
d- anxiety

the correct answer is a

-anorexia nervosa: decrease body wt. amenorrhea and lanugo (hair ).


-bulimia nervosa: normal or increase body wt. restrict eating following by
overeating then guilt.

…………………………………………………………………………………………………………………………………………………

174) obese female, insulin resistance and hairstisim so diagnosis


a-poly cystic ovary
b-hyperprolctinmia
c-familial
d-?"????????

the correct answer is a

…………………………………………………………………………………………………………………………………………………

175) boy 3 day after flue symptom develop conjunctivitis with occipital and neck L.N
enlarged so diagnosis is
a-adenoviruses
b-streptococcus
c-HSV
D??????????

the correct answer is a

…………………………………………………………………………………………………………………………………………………

176) child with asthma use betamethazone, most common side effect is
a-increase intraocular pressure
b-epilepsy
c-growth retardation
d-?????????????
. the correct answer is c

177) curve of HBV marker

answered in Q 38

…………………………………………………………………………………………………………………………………………………

178) sickling pt after acute attack , discharge on


a- penicillin
b-iron
c-vitamin
d???????????

the correct answer is a

prophylactic of pneumococcal infection by vaccination and oral penicillin

…………………………………………………………………………………………………………………………………………………

179) 6m with cough and wheezy chest .diagnosis is


a- asthma
b- broncholitis
c-pneumonia
d-F.B aspiration

The correct answer is b


- asthma : after 2 years old)
- broncholitis : before 2 years old)
-pneumonia : associated with crypitation
-F.B aspiration : sudden wheezing

…………………………………………………………………………………………………………………………………………………

180) 15y old with pilonidal sinuse so ttt


a-incision surgery
b- local antibiotic
c-daily clean
d-?????????

The correct answer is c


. pilonidal sinus : sinus tract witch commonly contain hairs. Treatment , firstly
conservative ttt in mild case remove all hair, washing cleaning . if not relive: surgery

181) Female pt 8 wks postpartum,not smoker diagnosed to have asthma,her asthma


was not controlled she attended ER 3 times last month,on B2 agonist and oral
steroid,she came c/o wheezing and s.o.b mildly cyanosed using her intercostal
muscles,wheezy chest,BP:160/100 P:120 PO2:72 PEF:36,there is oedema in her
foot up to the knee,the most likely diagnosis:
a. COPD
b. pulmonary embolism
c. Acute asthma attack
d. Angioedema

The correct answer is a or c

……………………………………………………………………………………………………………………………………………………

182) Female pt developed sudden loss of vision(both eyes) while she was walking
down the street,also c/o numbness and tingling jn her feet ,there is discrepancy
b/w the complaint and the finding,
O/E reflexes and ankle jerks preserved,there is decrease in the sensation and
weakness in the lower muscles not going with the anatomy,what is your action:

a. Call ophthalmologist
b. Call neurologist
c. call psychiatrist
d. reassure her and ask her about the stressors
the correct answer is : d

……………………………………………………………………………………………….........................................................

183) same scenario in Q.182 what is the diagnosis:

a. Conversion disorder
b. Somatoform disorder

The correct answer is a

1 – somatization disorder: female before age 30 years . symptoms include: .


. two GIT , four site of pain , one sexual dysfunction , one pseudoneuron .
. 2- conversion disorder: symptoms include voluntary or sensory .
. 3- hypochondriasis: fear from life threatening disease.
. 4- body dysmorphic disorder: aware from his imaging.
. 5- somatoform pain disorder: intensity pain is main symptoms

184) male pt developed corneal ulcer in his Rt eye after trauma what is the Mx:
a. Antibiotic and cycloplagia is mydrasis and refer to ophthalmology
b. topical steroid

the correct answer is a

Because infection is a common occurrence in corneal ulcers, your ophthalmologist


will prescribe antibiotic eyedrops. If the infection appears very large, you may need
to use these drops as often as one drop an hour. -Oral pain medications will be
prescribed to control the pain. Pain can also be controlled with special eyedrops
that keep your pupil dilated (Anticholinergics such as atropine, hyoscyamine, and
scopolamine)

...........................................................................................................................................................

185) female pt with Rt eye pain and redness with watery discharge,no h.o
trauma,itching,O/E there is diffuse congestion in the conjunctiva and watery
discharge what you'll do:
a. give Ab
b. give antihistamine
c. topical steroid
d. refer her to the ophthalmologist
???????????????

……………………………………………………………………………………………………………………………………………………..

186) Epidemic disease in poor sanitation areas affecting children and young adults:
a. hep A
b. B
c. C
d. D

The correct answer is a

187) mths baby with crying episodes+current jelly stool,looks slightly pale,signs of
obstruction wht is your Mx:
a. barium enema
b. immediate surgery
c. I.v fluid & wait for resolution

The correct answer is a

Intussusceptions :

Condition in which one portion of bowel invaginates into an adjacent segment


"usually proximal to ileocecal valve" . most common in first two years of life "
usually between 3 months and 3 years of life" . Abrupt –onest , colicky abdominal
pain , often accompanied by flexed knee and vomiting , ( one-off pain ) child may
appear well between episodes. Classic triad : abdominal pain , vomiting , blood per
rectum" only one third of pt." . Late signs : bloody mucus in stool "currant jelly
stool" , abdominal tenderness , palpable "sausage- shape " . RUQ abdominal mas.

Investigation & treatment :


-correct any volume and electrolyte abnormality and check ( cbc)
Abdominal film may be normal in early stage , and see obstruction , perforation in
late stage, US see " target sign"
In setting of high clinical suspicion >>> air-contrast barium enema, should be
performed without delay. As diagnostic in 95% of cases and curative in 80% of cases
perform surgical reduction of gangrenous bowel.

……………………………………………………………………………………………………………………….………………………......

188) 17 y.o adolescent, athletic ,with h/o Rt foot pain planter surface,diagnosis is:
a. planter fasciaitis
b. valux……
c. valux…..

Q not complete.

Planter fasciitis( heel spur syndrome ) :

Repetitive strain injury causing micro tears and inflammation of planter fascia.
Common in athletes , also associated with D.M , obesity , seronegative and
seropositive arthritis.
Clinical feature: morning pain and stiffiness , intense pain when walking from rest
and that subsides as pt. continuous to walk. Swelling and tenderness over sole,
greatest at medical calcaneal tubercle and 1-2 cm distal to along planter fascia . pain
with toe dorsiflexion

189) pregnant lady 16 wks presented with vaginal bleeding ,enlarged


abdomen,vomiting ,her uterus is smaller than expected for the gestational
age,BhCG 80,U/S snowstorm appearance,diagnosis:
a. complete hydatiform mole
b. partial hydatiform mole

The correct answer is a

………………………………………………………………………………………………………………………………………………….....

190) 10. 12 y.o boy c/o abdominal pain after playing football, he denied any h/o
trauma, the pain is in the Lt paraumbilical region what inx you want to do:
a. CXR
b. ultrasound kidney

Q not complete

………………………………………………………………………………………………………………………………………………….....

191) 5 y.o child with h.o fever and swelling of the face ant to the both ears (parotid
gland enlargement) what is the most common complication:
a. Labrynthitis
b. meningitis
c. encephalitis
d. orchitis

The correct answer is b

- most common complication of mumps after puberty . but it is rare occur pre-
puberty.

…………………………………………………………………………………………………………………………………………………

192) . what is the meaning of difficulty breathing:


a. dyspnia
b. tachycardia

The correct answer is a

193) A female patient on the 3rd week postpartum. She says to the physician that the
frequently visualizes snakes crawling to her baby’s bed. She knows that it is
impossible but she cannot remove the idea from her head. She says she wakes up
around 50 times at night to check her baby. This problem prevents her from
getting good sleep and it started to affect her marriage. What is this problem she
is experiencing?
a. An obsession
b. A hallucination
c. A postpartum psychosis
d. A Delusion

The correct answer is a ( 100 % correct  )

-obsession : persistent, unwanted, and intrusive ideas , thoughts, impulses or images


-hallucination: perception of objective or event without existing external stimulus
-illusion: false perception of actual external stimulus.
- delusion: fixed false idiosyncratic belief.

………………………………………………………………………………………………………………………….………………………….

194) female pt c/o sever migraine that affecting her work,she mentioned that she was
improved in her last pregnancy,to prevent that:
a. biofeedback
b. propranolol

The correct answer is b

- migraine prophylactic :
1- b-blocker (propranolol) : first line treatment (note: contindication in pregnancy)
2- TCA (amitriptyline )
3- anticovulsant
4- calcium channel blocker

……………………………………………………………………………………………………………………………………………………..

195) About DM in KSA:


a.. most of NIDDM are obese ( correct )

…………………………………………………………………………………………………………………………………………………….

196) Flu vaccine not given to the baby who is allergic to:
a. egg

contraindication of flu vaccine are egg allergic and hypersensitivity to thimerosal

197) 17. Pt with asymptomatic Trichomniasis:


a. treat her anyway regardless
b. treat her if she is symptomatic only

The correct answer is a

-treatment of trichomnias :

Single dose of metradinazole for symptomatic and asymptomatic pt. and


treatment partner ( because it is sexual transmitted disease )
But not treatment of asymptomatic trichomniasis in first trimester of pregnancy.
(contraindication in pregnancy )

…………………………………………………………………………………………………………………………..…………………………

198) 7 y.o,she missed her second dose of varecila vaccine,the first one about 1 y ago
what you'll do:
a. give her double dose vaccine
b. give her the second dose only
c. see if she has antibody and act accordingly

The correct answer is b


…………………………………………………………………………………………………………………………………………………….

199) pt with gonorrhea infection what else you want to check for
a. Clamydia trachomatis

…………………………………………………………………………………………………………………………………………………….

200) female pt with Aortic stenosis,she developed syncope while she was in the class
and she recovered immediately,what is the cause of syncope:
a. valvular rupture
b. systemic hypotension

The correct answer is b

……………………………………………………………………………………………………………………………………………………..

201) diabetic pt well controlled,she came with h.o dizziness and sweating after taking
a medication BS:60 what drug that cause her prob:

-Sulfanyurea can cause this side effects.

202) male pt with acute urine retention what is your action:


a. insert folly's cath and ask him to come back to the clinic

…………………………………………………………………………………………………………………………………………………….

203) In battered women which is true:


a. mostly they come from poor socioeconomic area
b. usually they marry a second violent man
c. mostly they come to the E/R c/o………..
d. mostly they think that the husband respond like this because they still have
. strong feeling for them

Answer ( d )

Battered women is Women who are physically and mentally abused over an
extended period, usually by a husband or other dominant male figure.
Characteristics of the battered woman syndrome are helplessness, constant fear,
and a perceived inability to escape. So I think choice d is correct answer

……………………………………………………………………………………………………………………………………………………..
204) smoking withdrawal symptoms peak at:
a. 1-2 days
b. 2-4 days
c. 5.7 days
d. 10-14 days

The correct answer is a

……………………………………………………………………………………………………………………….…………………………….

205) Mother who is breast feeding and she want to take MMR vaccine what is your
advice:
a. can be given safely during lactation
b. contain live bacteria that will be transmitted to the baby
c. stop breast feeding for 72 hrs after taking the vaccine

The correct answer is a

MMR : contraindication during pregnancy , and women should be avoid


pregnant in 4 weeks followed MMR vaccine . MMR is safe during lactation.

206) male pt c/o pain in his Rt elbow,he said that he is using the hummer a lot in his
work diagnosis:
a. lateral epichondylitis

-Lateral epicondylitis( inflammation of common extensor tendon ) also known as (tennis


elbow, shooter's elbow and archer's elbow is a condition where the outer part of the
elbow becomes sore and tender. It is commonly associated with playing tennis and
racquet sports.
-Medial epicondylitis (inflammation of common flexor elbow) also know (golfer elbow)

……………………………………………………………………………………………………………………………………………………

207) 50 y.o male with difficulty swallowing food with wt loss:


1. Oesophageal cancer

question not complete . but most likely is cancer.


causes of dysphagia food than liquid are :
1- carcinoma … 2- stricture … 3- plummer vision syndrome = web ( iron deficieny
anemia + golssitis )

……………………………………………………………………………………………………………………………………………………
208) . young female with pain in her elbow(lateral epichondylitis) best treatment is :
a. ………….+NSAID
b. electric ………….

question not complete .

- treatment of lateral epichondylitis:


1- first line : NSAID +rest + ice
2- second line : corticosteroid injection
3- third line : surgery .percutaneous release of common tendon

………………………………………………………………………………………………………………………………..…………………..

209) what drug that improve the survival in CHF


1. digoxin
2. Hydralazin

question is not complete .

Improve mortaility in CHF : B-blocker + ACE + ARB ( most important is ACE )

210) old man with bilateral knee pain and tenderness that increase with walking and
relieved by rest;
a. RA
b. OA

The correct answer is b

OA: pain with activity and weight bearing . and improve with rest .
RA: morning stiffness > 1 hour . painful and warm swelling of multiple symmetric
joint .

……………………………………………………………………………………………………………………………………………………

211) Regarding peritonitis:


a.Complicated appendectomy the cause is anerobe organism
b. rigidity and the cause is paralytic ileus
c. can be caused by chemical erosions
d. …………..

The correct answer is c

……………………………………………………………………………………………………………………………………………………..
212) regarding MI all of the following true except:
a. unstable angina,longer duration of pain and can occur even at rest.
b. stable angina,shorter duration and occur with excertion
c. there should be q wave in MI
d. even if there is very painful unstable angina the cardiac enzymes will be normal
e. ………………

all of the above are correct ???

……………………………………………………………………………………………………………………………………………………

213) Pt with scoliosis, you need to refer him to the ortho when the degree is:
a. 5
b. 10
c. 15
d. 20

The correct answer is d

214) 6 mths baby with undescending testis which is true:


a. till the mother that he need syrgery
b. in most of the cases spontaneous descent after 1 year
c. surgery indicated when he is 4 years
d. unlikely to become malignant

The correct answer is a

-after 1 year testes not desending. And surgery(orchiopexy) do from 6 –18months

……………………………………………………………………………………………………………………………………………………

215) 24 y.o Pt with asymptomatic congenital inguinal hernia:


a. immediate surgery
b. surgery indicated when he is >35 y
c. elective surgery if it is reducible
d. …………..

The correct answer is c


…………………………………………………………………………………………………………………………………………………..

216) the most effective thing regarding counseling:


a. family rapport
b. well adjusted appointment before counseling
c. ……………….

?????

…………………………………………………………………………………………………………………………………………………..
217) 5 years old C/O limping in CT there is a venous necrosis ttt is:
a. surgery
b. splent
c. physiotherpy

Q not complete:

This case is "perth's diseas" : vascular necrosis of femoral head. 5- 10


years . usually self limiting with symptoms lasting< 18 months. Present
with painless limb , limited abduction and internal rotation.
Treatment: 1- observation if there is limited femoral head involvement
or full ROM . 2- if extensive or if decrease ROM, consider bracing, hip
abduction with cast.

218) Pt. has DM and renal impairment when he had diabetic nephropathy:there is
curve for albumin
a. 5y
b. 10y
c. 20y
d. 25y

……………………………………………………………………………………………………………………………………

219) Pt has alzahimar disease and halusination and delusion ttt:


a. Halopridole

Psychotic symptoms (e.g. hallucinations and delusions), agitation and aggressive


behavior are common in patients with Alzheimer’s Disease. A study suggests
that haloperidol at a dose of 2-3 mgs/day is effective and well tolerated by most
patients.
220) Generalize anexity disorder best ttt:
a. SSRIs
b. tricyclic A D

The correct answer is a


……………………………………………………………………………………………………………………………………

221) Major depression management:


a. Intial therapy even sever

 Management of major depression disorder:


1-pharmacotherapy: effective in 50 – 70% .allow for 2-6 weeks to take effect , treat
more than 6 months ( SSRI, TCAs, MAOIs). 2-
psycotherapy: psychotherapy combined with antidepressant is more effect than
either treatment alone.
3- Electroconvulsion ( ECCT ).
4- phototherapy: effective for pt. who has a seasonal pattern.

……………………………………………………………………………………………………………………………………

222) Psychiatric pt with un compliance of drugs ttt:


a. depro halopredol

……………………………………………………………………………………………………………………………………

223) Strongest risk factor for strok:


a. Hypertension
b. Atrial fibrillation

The correct answer is a

224) Pt. with salpingitis and there is swelling in pelvis in posterior fornex and it is
fluctuant m:
a. Colpotomy
b. Laproscopic

The correct answer is a (I'm not sure )

colpotomy, also known as a vaginotomy, is a procedure by which an incision is


made in the vagina.
Purpose: A colpotomy is performed either to visualize pelvic structures or to
perform surgery on the fallopian tubes or ovaries.

……………………………………………………………………………………………………………………………………

225) Child swallowing battery in the oesophegus, management?


a. broncoscoby
226) In the appendisits the histology is:
a. leukocyte in muscle
b. layer of lymphoid
c. tumor
d. plasma cell

the correct answer is a

in appendicitis : neutrophil exudation throughout mucus, submucus, and


muscularis

……………………………………………………………………………………………………………………………………

227) water in the body:


a. 40%
b. differ depend on age and sex

the correct answer is b


Guyton's Textbook of Medical Physiology states that "the total amount of water in a
man of average weight (70 kilograms) is approximately 40 liters, averaging 57% of
his total body weight. In a newborn infant, this may be as high as 75% of the body
weight, but it progressively decreases from birth to old age, most of the decrease
occurring during the first 10 years of life. Also, obesity decreases the percentage of
water in the body, sometimes to as low as 45%".

……………………………………………………………………………………………………………………………………

228) Corneal ulcer ttt:


a. Antibiotic and cycloplagia is mydrasis and refer to ophthalmology

-Because infection is a common occurrence in corneal ulcers, your ophthalmologist


will prescribe antibiotic eyedrops. If the infection appears very large, you may need
to use these drops as often as one drop an hour. -Oral pain medications will be
prescribed to control the pain. Pain can also be controlled with special eyedrops
that keep your pupil dilated (Anticholinergics such as atropine, hyoscyamine, and
scopolamine)

229) regarding drainage of the abscess one of the following is true:


a. Carbuncle and frunculosis need drainage
b. Usually give ciphtriaxon and penicillin post draing

the correct answer is a

Frunculosis and carbuncles are pus-filled infected lumps on the skin. They usually
occur as a one-off in a healthy person. Treatment commonly involves draining the
pus and taking a course of antibiotics. If you have recurring boils you may be
advised to have tests to check for an underlying cause.

……………………………………………………………………………………………………………………………………

230) Salpingitis and PID on penicillin but not improve the most likely organism is :
a. Chlamydia trachomatis
b. Neisseria gonorrhoeae

The correct answer is b


Empiric antibiotic regimens should be aimed at treating likely causative agents, that
is, N. gonorrhoeae, C. trachomatis, genital mycoplasmas, and bacterial vaginosis-
associated endogenous microflora. The latter include anaerobic (Bacteroides and
Prevotella species and anaerobic streptococci) as well as aerobic organisms (G.
vaginalis, E. coli, and facultative streptococci). Except for N. gonorrhoeae and some
anaerobes, resistance is not yet a clinical problem.

……………………………………………………………………………………………………………………………………

231) Wound at end inflammatory phase when:


a. Epithelial tissue formation
b. Angiogenisis
c. when the wound clean
d. Scar formation

The correct answer is b

232) Juvenile RA ttt:


a. Aspirine
b. Steroid
c. Penicillamine
d. dydrocloroquin

The correct answer is a

The treatment of JRA focuses on suppressing inflammation, preserving and


maximizing function, preventing deformity, and preventing blindness. NSAIDs are
the first choice in the treatment of JRA.

……………………………………………………………………………………………………………………………………

233) Pt. has hemorrhoid with 4 degree ttt:


a. Hemoridectomy
b. band ligation
c. sclerotherapy
d. fiber diet
-The correct answer is a.

- Classification of internal hemorrhoid :


First degree : hemorrhoid do not prolapsed
Second degree : hemorrhoid prolapsed upon defecation but spontaneously reduce
Third degree : hemorrhoid prolapsed upon defecation , but must be manually reduce
Fourth degree : hemorrhoids are prolapsed and cannot be manually reduce

-treatment:
First & Second : life style modification ( dite)
Third : life style modification with (band ligation, sclerotherapy or cryotherapy ) , if
Failed go to surgery.
Fourth : surgery ( hemorrhiodectomy )

234) Pt. come with history of tinia capitis ttt:


a. tar shampoo
b. Fluconazol

The correct answer is b

Newer antifungal medications, such as ketoconazole, itraconazole,


terbinafine, and fluconazole, have been reported as effective alternative
therapeutic agents for tinea capitis. Of these agents, itraconazole and
terbinafine are used most commonly.

……………………………………………………………………………………………………………………………………

235) Pt. with history of COPD the most action to prevent complication is:
a. Pnemoccoccal vaccine
b. Smoking stop
c. Oral steroid
d. B2 agonist

The correct answer is b or a


……………………………………………………………………………………………………………………………………

236) Pt. with congenital hip dislocation :


a. abducting at flexed hip can causes click or tali

Congenital hip dislocation (CHD) occurs , more commonly in girls than in boys. The left hip is
twice as often involved as the right and bilateral dislocation occurs in more than 25 percent of
affected children. The criteria for the diagnosis of congenital dislocation of the hip include both
physical and radiographic findings. Certain clinical signs have been identified that are helpful in
the evaluation of newborns and infants for possible CHD, which include the following:
 limited abduction of the flexed hip, due to shortening and contraction of the hip
adductors;
 increase in depth or asymmetry of the inguinal or thigh skinfolds;

 shortening of one leg;

 Allis' or Galeazzi's sign -- lower position of knee of the affected side when knees and hips
are flexed, due to location of femoral head posterior to acetabulum in this position;

 Ortolani's "jerk" sign ("clunk of entry" or reduction sign);

 Barlow's test ("clunk of exit" or dislocation sign);


 telescoping or pistoning action of thighs, due to lack of containment of femoral head
with acetabulum;

 Trendelenburg's test -- drop of normal hip when child, standing on both feet, elevates
unaffected limb and bears weight on affected side, due to weakness of hip abductors;

 waddling type of gait.

There are characteristic radiographic presentations which are present in each of the stages of
CHD. The term congenital hip dysplasia generally refers to delayed or defective development of
the hip joint leading to a deranged articular relationship between an abnormal acetabulum and
a deformed proximal end of the femur. This condition is considered a precursor of the clinical
entities of subluxation and dislocation of the hip. In congenital subluxation of the hip, there is an
abnormal relationship between the femoral head and acetabulum, but the two are in contact.
Congenital dislocation of the hip, on the other hand, is associated with a complete loss of
contact of the femoral head with the acetabular cartilage. Unfortunately, the femoral head and
acetabulum in the newborn cannot be assessed by direct visualization, since the femoral head is
not ossified and is a cartilaginous body which is not visualized on plain films. The ossification
center for the femoral head generally appears between three and six months; a delay in its
appearance is an indication of congenital hip dysplasia. The neck of the femur is used for
ascertaining the relationship between the acetabulum and the femoral head until the
ossification center appears. The measurement used to evaluate the relationship of the femoral
head and acetabulum include Hilgenreiner line, acetabular index, Perkin-Ombredanne line,
Shenton-Menard line, and Andren-von Rosen line.

The principal treatment for CHD is conservative, especially if diagnosed early. The most common
technique is to reduce the dislocation of the femoral head by means of a flexion/abduction
maneuver, for a sufficient period of time to permit proper growth of the head and acetabulum,
which in turn assures a congruent and stable hip joint. This technique is usually performed on
patients in the very early stages of CHD and in infants under two years of age; which include
splinting, with a Frejka splint or Pavlik harness. Colonna or Buck's skin traction is used in children
2 to 12 years of age, with a well-padded spica cast applied simultaneously to the unaffected side

237) Colon cancer with stage 3 give the chemotherapy:


a. As soon as possible
b. 1 month

The correct answer is a (I'm not sure because MCQs not complete )

Treatment for Stage 3 Colon Cancer

Treatment for stage 3 colon cancer generally consists of a surgical resection followed
by chemotherapy. In a surgical resection, a surgeon removes the part of the colon
affected by the tumor and joins the remaining healthy sections together to form
one long, healthy piece.
The standard chemotherapy regimen used to be six months of treatment with 5-FU
and leucovorin, but that "cocktail" was developed decades ago and is seldom used
anymore. there are many new chemotherapy regimens available for stage 3 colon
cancer.

……………………………………………………………………………………………………………………………………

238) 56 y old present with vasomotor rhinitis


a. Local anti histamine
b. Local decongestion
c. Local steroid
d. Systemic antibiotic

The correct answer is b

Vasomotor rhinitis is a nonallergic condition that causes a constant runny nose,


sneezing, and nasal congestion.

……………………………………………………………………………………………………………………………………

239) Sickle Cell Anemia give prophlaxis:


a. Penicillin
b. Iron
The correct answer is a

TREATMENT
■ Acute crisis: Analgesia and hydration.
■ Hydroxyurea to ↑the amount of fetal hemoglobin.
■ H. influenzae and pneumococcal vaccines; prophylactic penicillin for
Children 5 years of age.
■ Acute chest syndrome: Respiratory support and exchange transfusion.

240) diagnosis of thalasimia minor:


a. Hb A2 and Hb f
b. Microcytosis

the correct answer is a

……………………………………………………………………………………………………………………………………

241) Pt. with MCV decrease and reticulocyt decrease iron deficiency anaemia
investigation:
a. Ferritin level and TIBC and serum iron
……………………………………………………………………………………………………………………………………

242) Born BCG


a. 1month hepatitis b oral polio dpt
b. 2month s
c. 3month s
d. 9 to 12 month mmr
????? I can't understand th Q ( may the writer Q missed some information )

243) E.histolytica cyst is destroyed by:

a. Freezing

b. Boiling

c. Iodine treatment

d. Chlorine

The correct answer is b


Amebiasis (or amoebiasis) is the name of the infection caused by E. histolytica.
To help prevent infection:
• Avoid raw vegetables when in endemic areas as they may have been fertilized
using human feces.
• Boil water or treat with iodine tablets
……………………………………………………………………………………………………………………………………………………

244) Patient after accident, the left ribcage move inward during inspiration and
outward during expiration:

a. Flial chest

Flail chest is a clinical anatomic diagnosis noted in blunt trauma patients with
paradoxical or reverse motion of a chest wall segment while spontaneously
breathing. This clinical finding disappears after intubation with positive pressure
ventilation, which occasionally results in a delayed diagnosis of the condition.

……………………………………………………………………………………………………………………………………………………

245) Greatest risk of stroke:

a. DM

b. Elevated blood pressure

c. Family history of stroke

d. Hyperlipedemia

e. Smooking

the correct answer is b

246) Child has pallor , eats little meat , by investigation :microcytic hypochromic
anemia . what will you do:

a. Trial of iron therary

b. Multivitamin with iron daily

the correct answer is a ( I'm not sure vecause Q is not complete )

……………………………………………………………………………………………………………………………………………………
247) Treatment of mania that does not cause hepatotoxicity

a. Lithium
b. carbamazepine
c. valporic acid
d. lamotrigine

The correct anser is a

Treatment of mood disorder :

Lithium : hepatotoxicity
carbamezapine : agrnulocytosis
valporic asid : neural tube defect

……………………………………………………………………………………………………………………………………………………

248) Sickle cell anemia patient , the macula is cherry red , and absence of afferent
papillary light reflex
a. Retinal artery occlusion.
. b. veine artery occlusion

the correct answer is a , cherry red spot is sign of retinal atery occlusion.

249) Inflammatory bowel disease is idiopathic but one of following is possible


underlying cause

a. Immunological

Inflammatory Bowel Disease Causes

Researchers do not yet know what causes inflammatory bowel disease. Therefore,
IBD is called an idiopathic disease (disease with an unknown cause).
An unknown factor/agent (or a combination of factors) triggers the body’s immune
system to produce an inflammatory reaction in the intestinal tract that continues
without control. As a result of the inflammatory reaction, the intestinal wall is
damaged leading to bloody diarrhea and abdominal pain.

……………………………………………………………………………………………………………………………………………………

250) Patient present with high blood pressure (systolic 200) , tachycardia , mydriasis ,
sweating . what is the toxicity:

a. Anticholenergic

b. Sympathomimetic

c. Tricyclic antidepressant

d. Organophosphorous compounds

the correct answer b

Uses of Sympathomimetics: To treat and prevent reversible bronchospasm


associated with bronchial and nocturnal (nighttime) asthma, chronic
bronchitis,emphysema, exercise-included bronchospasm and other obstructive
airway disease of the lungs.

To treat serious allegoric reactions (epinephrine injection only)

251) Treatment of Chlamydia with pregnancy:

a. Azithromycin

b. Erythromycin base

the correct answer b

For treatment of chlamydia during pregnancy, the CDC recommends:


-erythromycin base, 500 mg orally, four times a day for seven days; or amoxicillin
(Amoxil), 500 mg, three times a day for seven days.
Alternatives include:
-Erythromycin base 250 mg, four times a day for 14 days;
-erythromycin ethylsuccinate 800 mg orally, four times a day for seven days; or
-erythromycin ethylsuccinate 400 mg orally, four times a day for 14 days.

……………………………………………………………………………………………………………………………………………………

252) the maximum body lenght will be reached after menarch by

a -6 months

b. 1 year

A child will have also reached her final adult height about two years after menarche.

……………………………………………………………………………………………………………………………………………………

253) Patient developed sudden loss of vision bilaterally while she was walking in the
street , followed by numbness , the subjective symptoms are different from
objective , and does not match anatomical , what is your diagnosis:

a. Conversion syndrome

Conversion disorder:
Symptoms or deficits of voluntary motor or sensory function (e.g., blindness,
seizure) suggest a condition incompatible with medical processes. Close temporal
relationship to stress or intense emotion. More common in young females and in
lower socioeconomic and less educated groups.
……………………………………………………………………………………………………………………………………………………

254) After inflammatory phase of wound , there will be wound healing by:
a. If the wound is clean
b. Angiogenesis
c. Epithelial tissue
. the correct answer is b

255) Old male with tender knee , pain , crepitus . the diagnosis:

a. Osteorhritis
b. Ankylosin spondylitis
c. Rheumatoid

the correct answer a

Osteorhritis
OARTHRITIS (OA)
■ A chronic, noninflammatory arthritis of movable joints (e.g., DIP joints).
Characterized by deterioration of the articular cartilage and osteophyte
formation at joint surfaces.
■ Risk factors include a _ family history, obesity, and a history of joint
trauma.
■ Hx/PE: Crepitus; decreaserange of motion (ROM); pain that worsens with activity
and weight bearing but improves with rest.

……………………………………………………………………………………………………………………………………………………

256) Mother has baby with cleft palate and asks you what is the chance of having a
second baby with cleft palate or cleft lip ,

a. 25%
b. 50%
c. %1
d. 4%

the correct answer is d

……………………………………………………………………………………………………………………………………………………-

257) 1 liter fluid deficit equals :


. a.1 kg
. Liter of fluid deficit equal 1 kg of fluid as hydration protocol …………………………….
……………………………………………………………………………………………………....................

258) After accident patient with tachycardia, hypotension, what will be your initial
step:
a. Rapid IVF crystalloid
b. CT
Q not complete . management ABC then rapid IVF crystalloid.

259) 6 years child was born to HBS positive mother is HBS positive , he was only
vaccinated by BCG after birth , what you will give him now :
a. HBV + oral polio + DTP + hib
b. HBV + oral polio + dt + MMR +hib
c. HBV + oral polio + Dt + MMR

?????????? all the above are wrong

……………………………………………………………………………………………………………………………………………………

260) Treatment of non inflammatory acne


a. Retinoic acid

Retinoids: Medicines structurally similar to vitamin A are useful in preventing


several types of acne lesions. Topical retinoids are effective in treating the
noninflammatory types of acne (blackheads and whiteheads).

……………………………………………………………………………………………………………………………………………………

261) Treatment of comedones:

a. Topical retinoids.

Comedones: The plural of comedo, the primary sign of acne, consisting of a dilated
(widened) hair follicle filled with keratin squamae (skin debris), bacteria, and sebum
(oil). Comedones may be closed or open.

……………………………………………………………………………………………………………………………………………………

262) Treatment of papules or pustules:

a.Topical benzoyl
b.Peroxide plus topical antibiotics, mainly clindamycin or erythromycin.
c. In severe cases, intralesional steroid injection or oral antibiotics, such as
tetracycline or erythromycin may be added.

The correct answer b

TREATMENT
■ Mild acne: Topical clindamycin or erythromycin; benzoyl peroxide; topical
retinoids.
■ Moderate acne: The above regimen plus oral antibiotics such as tetracycline.
■ Severe nodulocystic acne: Oral isotretinoin (Accutane).

263) Which is not true In emergency management of stroke

a.Give IVF to avoid D5 50%


b.Give diazepam in convulsions .
c.Anticonvulsants not needed in if seizures
d.-Must correct electrolytes
e. Treat elevated blood pressure

the correct answer is c

……………………………………………………………………………………………………………………………………………………
264) SCA complications in adults

a. Cerebral infarction
b. Cerebral hemorrhage

Neurological complications occur in 25% of patients, with transient ischaemic attacks, fits,
cerebral infarction, cerebral haemorrhage and coma. Strokes occur in about
11% of patients under 20 years of age. The most common finding is obstruction of a distal
intracranial internal carotid artery or a proximal middle cerebral artery. 10% of children
without neurological signs or symptoms have abnormal blood-flow velocity indicative of
clinically significant arterial stenosis; such patients have very high risk of stroke. It has now been
demonstrated that if children with stenotic cranial artery lesions, as demonstrated on
transcranial Doppler ultrasonography, are maintained on a regular programme of transfusion
that is designed to suppress erythropoiesis so that no more than 30% of the circulating red cells
are their own, about 90% of strokes in such children could be prevented.
……………………………………………………………………………………………………………………………………………………

265) The most common risk for intracerebral stroke

a. Hypertension

266) The antidepressant used for secondary depression that cause sexual dysfunction

a.Sertatline (SSRIs)
b. Imapiramine
c. Levofluxine

the correct answer a


……………………………………………………………………………………………………………………………………………………

267) Previously healthy female patient presented to ER with dysnea , anexiety , tremor
, and she breath heavily , the symptoms began 20 minutes before she came to ER ,
in the hospital she developed numbness periorbital and in her fingers , what you
will do

a. Ask her to breath into a bag


. b. Take blood sample to look for alcohol toxicity

MCQs not complete

……………………………………………………………………………………………………………………………………………………

268) What is the most important in councling

a. Exclude physical illness

b. Establishing rabbot

c. Family

d. Schedule appointement

?????

269) In breaking bad news

a. Find out how much the patient know


b. Find out how much the patient wants to know

the correct answer a


Robert Buckman's Six Step Protocol for Breaking Bad News

1. Getting started.
2. Finding out how much the patient knows.

3. Finding out how much the patient wants to know.

4. Sharing the information.

5. Responding to the patients feelings.

6. Planning and follow-through.

…………………………………………………………………………………………………………………………………………………

270) Patient with chest pain that aggrevated by couphing, there is added sound on left
sternal border .in ecg you will find

a. St changes

b. Pr prolongation

c. Hypervoltage

the correct answer a . (percarditis)

…………………………………………………………………………………………………………………………………………………

271) The most common site for visceral hemangioma is

a. Liver ( most common site for visceral hemangioma )

A hepatic hemangioma is the most common noncancerous tumor of the liver. It is


believed to be a birth defect. Hepatic hemangiomas can occur at any time, but are
most common in people in their 30s - 50s. Women are affected more often than
men, and usually have bigger tumors than men. Babies may develop a type of
hepatic hemangioma called benign infantile hemangioendothelioma (also called
multinodular hepatic hemangiomatosis). This rare, noncancerous tumor has been
linked to high rates of heart failure and death in infants. Infants are usually
diagnosed by the time they are 6 months old.

272) Child with large periorbital hemangioma , if this hemangioma cause obstruction
to vision , when will be permenant decrease in visual acuity

a. After obstruction by one day


b. By 1 week

c. By 3 months

d. By 6 months

?????????

…………………………………………………………………………………………………………………………………………………

273) The symptoms of soft tissue sarcoma

a. Progressive enlarging mass ( on growing )

In their early stages, soft tissue sarcomas usually do not cause symptoms.
Because soft tissue is relatively elastic, tumors can grow rather large, pushing
aside normal tissue, before they are felt or cause any problems. The first
noticeable symptom is usually a painless lump or swelling. As the tumor grows, it
may cause other symptoms, such as pain or soreness, as it presses against nearby
nerves and muscles. If in the abdomen it can cause abdominal pains commonly
mistaken for menstrual cramps, indigestion, or cause constipation

…………………………………………………………………………………………………………………………………………………

274) 35 year female with bilateral breast pain , that decrease after menstruation , the
breast is nodular with prominent 3 cm mass subareolar , axillary lymph nodes are
not enlarged , what you will do

a. Mammography followed by US

b. See her next cycle

c. Fine needle biopsy followed by tissue studies

the correct answer is b

275) Pregnant with bleeding for 12 hours and tissue , the cervix is 1 cm
a. Complete abortion
. b. Incomplete abortion
. c. Molar pregnancy

the correct answer is b


276) 2 month infant with vomiting after each meal , he is in 50 centile , He passed
meconium early and stool , diagnosis is

a. Midgut volvulus

b. Meconium ileus (not passing early meconium and stool)

c. Hischsprung disease (not passing early meconium and stool)

I think this case is pyloric stenosis. If not mention choose a

…………………………………………………………………………………………………………………………………………………

277) Patient with dysphagia to solid and liquid , and regurg , by barium there is non
peristalsis dilatation of osophagus and air-fluid level and tapering end.diagnosis is

a. Osophageal spasm

b. Achalasia

c. Osophageal cancer

the correct answer is b

…………………………………………………………………………………………………………………………………………………

278) The most common cause of cough in adults is

a. Asthma
b. GERD
c. Postnasal drip

The correct answer c

The most common causes of acute cough is respiratory viral infection. The most
common cause of chronic cough are postnasal drip, asthma, and acid reflux from
the stomach. These three causes are responsible for up to 90 percent of all cases of
chronic cough.
279) Girl with amenorrhea for many months . BMI is 20 and is stable over last 5 years .
diagnosis

a.Eating disorder

b.Pituitary adenoma

…………………………………………………………………………………………………………………………………………………

280) Old female with itching of vulva , by examination there is pale and thin vagina ,
no discharge . what is management

a. Estrogen cream

b. Corticosteroid cream

c. Fluconazole

the correct answer is a

Some women will opt for prescription medications as they go through menopause.
The most common prescriptions include:
1 -Hormone replacement therapy or anti-depressants to minimize hot flashes.
2 -Fosamax or Actonel (non-hormonal medications) to reduce bone loss and reduce
the risk of fractures .
3 -Selective estrogen receptor modulators (SERMs), which mimic estrogen's
beneficial effects on bone density.
4-Vaginal estrogen, administered locally, to relieve vaginal dryness and discomfort
during intercourse

…………………………………………………………………………………………………………………………………………………

281) Patient with dysuria , frequency , urgency , but no flank pain , what is the
treatment

a. Ciprofloxacin po od for 3-5 days

b. Norfocin po od for 7 – 14 days

the correct answer is b


282) Patient with flank pain , fever , vomiting , treatment is

a. Hospitalization and intravenous antibiotics and fluid

This is most likely a case of pyelonephritis which need urgent hospitalization

…………………………………………………………………………………………………………………………………………………

283) Patient present with mid face pain , erethematous lesions and vesicles on
periorbital and forehead , the pain is at nose , nose is erythematous . what is
diagnosis

a. Roseola
. b. HSV
. c. Herpes zoster

the correct answer is c

…………………………………………………………………………………………………………………………………………………

284) Male with itching in groin erythematous lesions and some have clear centers ,
what is diagnosis :
a. Psoriasis
b. Tinea curis
c. Erythrasma

the correct answer is b

Patients with tinea cruris report pruritus and rash in the groin. A history of previous
episodes of a similar problem usually is elicited. Additional historical information in
patients with tinea cruris may include recently visiting a tropical climate, wearing
tight-fitting clothes (including bathing suits) for extended periods, sharing clothing
with others, participating in sports, or coexisting diabetes mellitus or obesity. Prison
inmates, members of the armed forces, members of athletic teams, and people
who wear tight clothing may be subject to independent or additional risk for
dermatophytosis.Large patches of erythema with central clearing are centered on
the inguinal creases and extend distally down the medial aspects of the thighs and
proximally to the lower abdomen and pubic area.-
285) Vasoconstrictive nasal drops complication

a. Rebound phenomenon

…………………………………………………………………………………………………………………………………………………

286) The useful excurcise for osteoarthritis in old age to maintain muscle and bone
Low resistance and high repetion weight training:

a. Conditioning and low repetion weight training

b. Walking and weight exercise

the correct answer is b

Exercise is one of the best treatments for osteoarthritis. The best exercises for
osteoarthritis suffers depend on what joints are affected. Swimming, walking,
and cycling are often the best exercises for people with osteoarthritis. Try to
get thirty minutes of exercise five times per week. The key is to start slowly.

…………………………………………………………………………………………………………………………………………………

287) Unilateral worsening headach , nausea , excacerbeted by movement and


aggrevated by light in 17 old girl.

a. Migraine (Photophobia, vomiting)


b. Cluster

the correct answer is a

288) Diet supplement for osteoarthritis


a. Ginger

A large number of dietary supplements are promoted to patients with osteoarthritis


and as many as one third of those patients have used a supplement to treat their
condition. Glucosamine-containing supplements are among the most commonly
used products for osteoarthritis. Although the evidence is not entirely consistent,
most research suggests that glucosamine sulfate can improve symptoms of pain
related to osteoarthritis, as well as slow disease progression in patients with
osteoarthritis of the knee. Chondroitin sulfate also appears to reduce osteoarthritis
symptoms and is often combined with glucosamine, but there is no reliable
evidence that the combination is more effective than either agent alone. S-
adenosylmethionine may reduce pain but high costs and product quality issues limit
its use. Several other supplements are promoted for treating osteoarthritis, such as
methylsulfonylmethane, Harpagophytum procumbens (devil's claw), Curcuma longa
(turmeric), and Zingiber officinale (ginger), but there is insufficient reliable
evidence regarding long-term safety or effectiveness.

………………………………………………………………………………………………………………………………………………

289) Old male with abdominal pain , nausea , WBC 7. What is true about appendicitis
in elderly?

a. Ct not usefull for diagnosis.


b. WBC is often normal.
c. Rupture is common
d. If there is no fever the diagnosis of appendicitis is unlikely
e. Anemia is common

the correct answer is c

Appendicitis in elderly
Appendicitis is a less common cause of abdominal pain in elderly patients than in younger patients, but
the incidence among elderly patients appears to be rising. Only approximately 10% of cases of acute
appendicitis occur in patients older than 60 years, whereas one half of all deaths from appendicitis
occur in this age group. The rate of perforation in elderly patients is approximately 50%, 5 times higher
than in younger adults. This is largely because 75% of elderly patients wait more than 24 hours to seek
medical attention. The diagnosis can be difficult to make, since more than one half of patients in this
age group do not present with fever or leukocytosis. Further confusing the picture, approximately one
third do not localize pain to the right lower quadrant, and one fourth do not have appreciable right
lower quadrant tenderness. Only 20% of elderly patients present with anorexia, fever, right lower
quadrant pain, and leukocytosis. The initial diagnosis is incorrect in 40-50% of patients in this age
range . ( perforation is the most common complication of appendicitis )
290) Old patient with bilateral enlarged knee , no history of trauma , no tenderness ,
normal ESR and C-reactive proteins . the diagnosis is

a. Osteoarthritis
b. Gout
c. Infectous arthritis

the correct answer is a ( I'm not sure )

………………………………………………………………………………………………………………………………………………

291) Patient has decrease visual acuity bilateral , but more in rt side , visual field is not
affected , in fundus there is irregular pigmentations and early cataract formation .

a. Refer to ophthalmologist for laser therapy

b. Refer to ophthalmologist for cataract surgery ??????????

the correct answer is a

………………………………………………………………………………………………………………………………………………

292) What is the most common treatment for juvenile rheumatoid arthritis

a. Intraarticular injection of steroid

b. Oral steroid

c. Paracetamol

d. penicillamine

e. Asprin

the correct answer is e

the most common important for JRA is ( NSAID )

……………………………………………………………………………………………………………………………………………

293) Which of the following decrease mortality after MI

a. Metoprolol
b. Nitroglycerine
c. Thiazide
d. Morphine
the correct answer is a
only b-blocker ans ASA are mortality benefit for treatment of angina

294) The cardiac arrest in children is uncommon but if occur it will be due to

a. Primary

b. Respiratory arrest

c. hypovolemic shock

d. neurogenic shock

the correct answer is b

Progressive respiratory insufficiency accounts for 60% of all paediatric arrests.

………………………………………………………………………………………………………………………………………………

295) Old female with recurrent fracture , Vitamen D insufeciency and smoker . which
exogenous factor has the gretest exogenous side effect on osteoporosis.

a. Old age

b. Smoking

c. . Vit D insufeciency

d. Continue smoking

e. Recurrent fracture

the correct answer is b

………………………………………………………………………………………………………………………………………………

296) patient presented with sudden chest pain and dysnea , tactile vocal fremitus and
chest movemebt is decreased , by x-ray there is decreased pulmonary marking in
left side , diagnosis

a. atelectasis of left lung

b. spontaneous pneumothorax

c. pulmonary embolism

??????????
297) boy after running for hours , has pain in knee and mass on upper surface of tibia

a. Osgood scatter disease

b. Iliotibial band

the correct answer is a

………………………………………………………………………………………………………………………………………………

298) pancreatitis

a. Amylase is slowly rising but remain for days

b. Amylase is more specific but less sensitive than lipase

c. Ranson criteria has severity (predictive) in acute pancreatitis

d. Pain is increased by sitting and relieved by lying down

e. Contraceptive pills is associated

the correct answer is c

The Ranson and Glasgow scoring systems are based on such parameters and have
been shown to have an 80% sensitivity for predicting a severe attack, although only
after 48 hours following presentation.
risk mortality is 20% with 3-4 signs, 40% with 5-6 signs, 100% 7 signs.
299) Patient has fever , night sweating , bloody sputum , weight loss , ppd test was
positive . x-ray show infiltrate in apex of lung , ppd test is now reactionary ,
diagnosis

a. Activation of primary TB

b. sarcoidosis

c. Case control is

d. Backward study

the correct answer is a

The tuberculosis skin test (also known as the tuberculin test or PPD test) is a test
used to determine if someone has developed an immune response to the
bacterium that causes tuberculosis (TB).

……………………………………………………………………………………………………………………………………………………..

300) patient with DM presented with limited or decreased range of movement passive
and active of all directions of shoulder
a. frozen shoulder
b. impingment syndrome
c. osteoarthritis
the correct answer is a
……………………………………………………………………………………………………………………………………………………..

301) 48 years old with irregular menses presented with fatigue and no menstruation
for 3 months with increased pigmentation around the vaginal area with no other
symptoms. ur next step would be
a. reassure the patient
b. do a pregnancy test
c. do ultrasound

the correct answer b

302) 3 years old presented with shortness of breath and cough at night which resolved
by itself in 2 days. he has Hx of rash on his hands and allergic rhinitis. he most
likely had
a. croup
b. bronchial asthama
c. epilotitis

????????

Croup Epiglottitis

Onset Days Hours

Flu-like symptoms Yes No

Cough Sever Absent

Able to drink Yes No

Drooling saliva No Yes

Fever <38 >38

Stridor Harsh Soft

Voice Hoarse muffed

…………………………………………………………………………………………………………………………………………………
303) a man went on vacation. he noticed a white patch in his chest which became
more clear after getting a sun tan which was spread on his chest.ur Dx is
a . pytriasis versicolor
b. vitilligo
c. pytriasis roscea

the correct answer a


……………………………………………………………………………………………………………………………………………………..
304) a 4 years old presented with 2 day history of shortness of breath a seal like cough
with no sputum and mild fever. on examination he did not look I'll or in distress

a. acute epilossitis
b. croup
c. angioedema

The correct answer b

304)65 years old came with knee pain. and limited movement. on examination had
crepition on knee. dx
a. rhuematiod arthritis
b. osteoporosis
c. osteoarthritis

the correct answer c

……………………………………………………………………………………………………………………………………………………..
305) which vitamin is given to new born to stop bleeding
a. vit. A
b. vit. D
c. vit. K
d. vit E
e. vit C

the correct answer c

……………………………………………………………………………………………………………………………………………………..

306) a child came with congested throught and mild bulging of tympanic membrane ur
Dx was URTI. The
?????????

……………………………………………………………………………………………………………………………………………………..

307) a child presented with erythematous pharynx, with cervical lymph nodes and
rapid strplysin test negative and low grade fever with positive EBV. it next step
a. give antibiotics and anti pyretic
b. give anti pyretic and fluids
d. culture and sensitivity

the correct answer a


……………………………………………………………………………………………………………………………………………………..

308) child with low grade fever and congested throat, negative ASO and positive EBV.
he has
a. infectous mononucleosis
b. URTI

the correct answer is a


……………………………………………………………………………………………………………………………………………………..
309) the most common cause of intracerebral or intraparynchimal bleeding is.
a. Hypertensive angiopathy
b. annurysm
c. AV malformation

the correct answer is a


310) young adult Sickle cell patients are commonly affected with
a. dementia
b. multiple cerebral infarcts

the correct answer b

……………………………………………………………………………………………………………………………………………………..

311) 80 years old living in nursing home for the last 3 months. his wife died 6 months
ago and he had a cornary artery disease in the last month. he is now forgetful
especially of short term memory and decrease eye contact with and loss of
interest. dx
a. alzihiemer
b. depression
c. hypothyroidism

the correct answer a , I think Q is not complete

……………………………………………………………………………………………………………………………………………………..

312) thyroid cancer can be from


a. hypothyroidism
b. graves disease
c. toxic nodule

?????????

……………………………………………………………………………………………………………………………………………………..
313) a mother came with her son who is 7 years old very active never sitting in class
and with poor concentration. ur management would be.
a. olanzipine
b. amitilyne
c. aloxane

the correct answer is b ( I'm not sure )

this case is "attention –deficit hyperactivity disorder"


treatment:
1- Methylphenidate
2- Dextroamphetamine
3- Aderall
4- Atomoxetine
5- Pemoline
6- Antidepressant ( SSRI )

314) from the graph above which of the following statement is true
a. osteoprosis affecting majority of women above 80
b. 20% of women under 70 are affected

?????????
……………………………………………………………………………………………………………………………………………………..

315) a patient presented with progressive weakness on swollowibg with diplopia and
fatigability. the most likely underlying cause of her disease is.
a. antibody against acetylcholine receptors

……………………………………………………………………………………………………………………………………………………..
316) an 69 year old non diabetic. with mild hypertension and no hx of Coronary heart
desease. the best drug in treatment is.
a. thiazides
b. ACEI
c. ARB
d. CCB

the correct answer a


……………………………………………………………………………………………………………………………………………………..

317) which of the following anti hypertensive is contraindicated for an uncontrolled


diabetic patient
a. hydrochlorothiazide
b. Losartan
c. hydralszine
d. spironolactone
the correct answer is a
-side effect of thiazide : hyperglycemia
……………………………………………………………………………………………………………………………………………………..

318) a wound stays in it's primary inflammation untill


a. Escher formation
b. epitheliazation
c. after 24 hours
d. wound cleaning

the correct answer d


……………………………………………………………………………………………………………………………………………………..
319) 70 years old male patient with mild urinary dripping and hesitency ur Dx is mild
BPH. ur next step in management is
a. transurethral retrograde prostatectomy
b. start on medication
c. open prostatectomy

the correct answer b


320) most common cause of secondary HTN is
a. Renal disease - Renal parenchymal disease

……………………………………………………………………………………………………………………………………………………..

321) one of the following food is known to reduce cancer


a. fibers
……………………………………………………………………………………………………………………………………………………..

322) smoking is a definitive risk of


a. sq.c.ca of bladder
b. liver
c. breast

the correct answer is a

……………………………………………………………………………………………………………………………………………………..

323) exercise recommended for patients with CAD. is


a. isometric
b. isotonic
c. yoga

‫؟؟؟؟؟؟؟؟؟؟‬

……………………………………………………………………………………………………………………………………………………..

324)what is the definitive treatment of frostbites


a, Rewarming
……………………………………………………………………………………………………………………………………………………..
325) the most common cause of failure to thrive in pediatric is
a. malnutrition

……………………………………………………………………………………………………………………………………………………..

326) which of the following causes the highest maternal mortality in pregnancy
a. toxoplasma
b. hyperbilirubenia
c. ????

" The major causes of maternal death are bacterial infection, variants of
gestational hypertension including pre-eclampsia and HELLP syndrome,
obstetrical hemorrhage, ectopic pregnancy, puerperal sepsis (childbed fever),

327) Ttt of frostbite


a. Immersion in water 40-45 C
b. Debridement
c. Leave it at room tepmrature

The correct answer is a

……………………………………………………………………………………………………………………………………………………..
328) Case about a child both RBS, FBS are elevated so he has DM1…what's the type of HLA
a.DR3
b.DR4
c.DR5
d.DR6
e.DR7

the correct answer is a

Types of HLA :
DR2 : Good's pasture syndrome & multiple myeloma
DR3 : D.M , SLE , & grave's disease
DR27 : ankylosing spondylaitis & reiter's syndrome
B51 : behceat's disease
D11 : hashimot's disease

…….……………………………………………………………………………………………………………………………………………..
329) Definition of PPV ?

Predictive positive value ( PPV ) : proportion of people with a positive test who have s a
disease : a/ (a+b)
……………………………………………………………………………………………………………………………………………………..
330) What's the most common case in PHC centers
a.UTI
b.HTN
c. Coryza

the correct answer is c

331) all of the following will decrease pt compliance except:


a. involve pt in the plan
b. make simplified regimen
c. give easy written instructions
d. make appointments flexible
e. warn the patient about the danger of missing a pill

the correct answer is e

……………………………………………………………………………………………………………………………………………………..
332) Case about old diabetic patient who still have hyperglycemia despite increase insulin
dose…the problem with insulin in obese patients is
a. Post receptor resistance
?????? Q NOT COMPLETE

…………………………………………………………………………………………………………………………………………………….

333) Case about a child with drooling, fever, barking cough in sitting position, dx:
a. Croup
b. Broncholities
c. Pneumonia

The correct answer is a

……………………………………………………………………………………………………………………………………………………..
334) Mother came to you after her son had hematoma under the nail dt injury:
a. Send home with a pad on the head
b. Send home with acetaminophen
c. Do wedge resection
d. Evacuate the hematoma

The correct answer is d

This case is ( subungual hematoma )


335) What vaccine u'll give to a SCD ( sickle cell disease ) child
a.HBV
b. H.influenza
c. pneumococcal
d. both A and B
e. all of the above

the correct answer is c

……………………………………………………………………………………………………………………………………………………..

336) a mother with HBsAg positive came with her child 6 yrs old who has HBsAg +ve what
will you give him:
a. oral polio, DTP, MMR

the correct answer is a


other choices all had HBV vaccine which u'll not give

……………………………………………………………………………………………………………………………………………………..

337) Patient wil LLQ pain, vomiting, fever, high WBC (17.000), tenderness and rebound
tenderness
a. Diverticulitis
b. Sigmoid volvulus
c. Appendicitis
d. Toxic enteritis

The correct answer is a

……………………………………………………………………………………………………………………………………………………..

338) What's the organism responsible for psuedomembranous colitis:


a. Pseudomonas
b. Colisteridum
c. E.coli
d. Enterococcus fecalis
The correct answer is b

339) Mother came with her child who had botillism, what you will advice her:
a. Never eat canned food again
b. Store canned food at home
c. Boil canned food for 40-50 min
d. Check expiry date of canned food

The correct answer is d

……………………………………………………………………………………………………………………………………………………..

340) Old pt presented with abdominal pain, back pain, pulsatile abdomen what's the step
to confirm dx: this is a case of aortic aneurysm
a. Abdominal US
b. Abdominal CT
c. Abdominal MRI

The correct answer is a b


-initial investigation US – CONFIRM by CT

……………………………………………………………………………………………………………………………………………………..

341) 18 months old came with bite by her brother, what you will do:
a. Give augminten
b. Give titunus toxoid
c. Suture

Q not complete

……………………………………………………………………………………………………………………………………………………..

342) 19 yrs old after bike accident, he cant bring the spoon infront of himself to eat, lesion
is in:
a. Temporal lobe
b. Cerebellum
c. Parietal lobe
d. Occipital lobe

The correct answer is b


343) How to dx DVT:
a. Contrast venography
b. Duplex US

The correct answer is b

……………………………………………………………………………………………………………………………………………………..

344) Pt came after RTA, GCS 14, near complete amputation of the arm, 1 st step:
a. Secure air way
b. Tourniquet on the arm

The correct answer is a

……………………………………………………………………………………………………………………………………………………..

345) Pt with hx of prolonged heavey bleeding 2 hrs post partum, you will give:
a. Ringers lactate
b. NS
c. NS+ packed erythrocytes

The correct answer is c ( Q NOT COMPLETE )

……………………………………………………………………………………………………………………………………………………..
346) What is special about placenta abruption:
a. Abnormal uterine contractions
b. PV bleeding
c. Fetal distress

The correct answer is b , PA : dark , painful vaginal bleeding and complication is fetal
hypoxia

……………………………………………………………………………………………………………………………………………………..

347) Pt 34 wks, hx of PV bleeding for many hrs, dark blood, abdominal tenderness, FHR 120,
uterine contractions every 3 min, Dx:
a. Abruption placenta

348) One of the steps in managing epistaxis:


a. Packing the nose
b. Press the fleshy parts of nostrils
c. Put patient of lateral lying position

The correct answer is b

……………………………………………………………………………………………………………………………………………………..

349) Pt with hx of diarrhea, abdominal pain, agitation, headache, dizziness, weakness,


pulstile thyroid, unsteady gate. Examination was normal. Dx:
a. Hypochondriasis
b. Somatization disorder
c. Thyroid Ca
d. Anxiety

The correct answer is b

……………………………………………………………………………………………………………………………………………………..

350) Child with headache that increase by changing head postion, unilateral, photophobia
a. Sinus headache
b. Migraine
c. Cluster headache
d. Tension headache

The correct answer is b

……………………………………………………………………………………………………………………………………………………..

351) Ttt of isolated fracture of femur (repeated) ???

………………………………………………………………………………………………………………………………………………………

352) Younge pt with pain in LL after running 3 kgs, more at night, swelling, XR was NL
a. Stress fracture

353) HTN pt, with decrease vision, fundal exam showed increase cupping of optic disc dx:
a. Open angle glaucoma
b. Closed angle glaucoma
c. Cataract
d. HTN changes
The correct answer is a ( I'm not sure )

……………………………………………………………………………………………………………………………………………………..

354) Pt with unilateral red eye, tearing, clear tears no swelling or discharge:
a. Give topical antihistamin
b. Give topical AB
c. Give systemic AB
d. Reassurance

The correct answer is d

……………………………………………………………………………………………………………………………………………………..

355) Business man went to Pakistan, came with bloody diarrhea, stool examination showed
trophozoite with RBC inclusion, Dx:
a. Amebic desyntry (entamoeba histlolytica )

……………………………………………………………………………………………………………………………………………………..

356) 5 yrs old child with abdominal pain after 2 wks of URTI, HB 8, retics 12% WBC NL
peripheral blood smear showed target cells, RBC inclusions dx:
a. SCA (the only hemolytic anemia in the answers)
?????

……………………………………………………………………………………………………………………………………………………..

357) Child had hb electrophoresis showed hb 2%


a. HBA 40 HBS 35 HBF 5 HBA2
b. Sickle cell trait
c. Thalacemia major
d. Minor
?????

358) Younge pt with hx of cough, chest pain, fever CXR showed RT lower lobe infiltrate:
a. Amoxicillin
b. Ceferuxim
c. Emipenim
d. Ciprofloxacin
??????
The correct answer is d (I'm not sure )

……………………………………………………………………………………………………………………………………………………..
359) Best thing to reduce mortality rate in COPD:
a. Home O2 therapy
b. Enalipril
c. Stop smoking

The correct answer is c

……………………………………………………………………………………………………………………………………………………..

360) Drug that will delay need of surgery in AR:


a. digoxin
b. verapamil
c. nefidipin
d. enalpril

the correct answer is c

……………………………………………………………………………………………………………………………………………………..
361) Child with skin rash, pericarditis, arthritis dx:
Kawasaki

……………………………………………………………………………………………………………………………………………………..

362) Pic of skin with purple flat topped polygonal papules, dx:
a. Lichen plannus

……………………………………………………………………………………………………………………………………………………..

363) Child with atopic dermatitis, what you will give other than cortisone
a. There were many drugs, steroids and AB and only one strange name maybe
moisturizer. That's the answer

364) A man had increase shoe size and jaw, the responsible is:
a. ACTH
b. Somato??
c. TSH
d. Cortisone

The correct answer is b

……………………………………………………………………………………………………………………………………………………..

365) The best advice to patient travelling is:


a. Boiled water
b. Ice
c. Water
d. Salad and under cooked sea shells

The correct answer is a

……………………………………………………………………………………………………………………………………………………..

366) Seldinfil is contraindicated with:


a. Nitrate
b. Methyldopa
c. Gabapentine

The correct answer is a

……………………………………………………………………………………………………………………………………………………..

367) Pt with TB, had ocular toxicity symptoms, the drug responsible is:
a. INH
b. Ethambutol
c. Rifampicin
d. Streptomycin

The correct answer is b

INH : peripheral neuritis and hepatitis . so add ( B6 )


Ethambutol : optic neuritis
Rifampicine : body fluid organe

368) Another pt treated for TB started to develop numbness, the vit deficient is:
a. Thiamin
b. Niacin
c. Pyridoxine
d. Vit C

THE correct answer is c

……………………………………………………………………………………………………………………………………………………..

369) Pt with bilateral breast disease, dx:


a. Paget
b. Papilloma
c. Meduallary
d. Lobular
……………………………………………………………………………………………………………………………………………………..

370) Child fell on her elbow and had abrasion, now swelling is more, tenderness, redness,
swelling is demarcated (they gave dimensions) child has fever. Dx:
a. Gonoccal arthritis
b. Synovitis
c. Cellulitis of elbow

The correct answer is c

……………………………………………………………………………………………………………………………………………………..

371) About burn


a. You will give 1\2 fluid in the 1st 8 hrs
b. 1\4 in the 1st 8 hrs

The correct answer is a

Parkland formula : fluid in first 24 hours = 4 * wt. in kg * % BSA


50% in first 8 hours and another in next 16 hours

372) You r supposed to keep a child NPO he's 25 kgs, how much you will give:
a. 1300
b. 1400
c. 1500
d. 1600

The correct answer is d

For the first 10 kg of weight, a child needs 100 mL per kg of weight.


For the next 10 kg of weight (11-20 kg), a child only needs 50 mL per kg of weight.
for anything over 20 kg (21 kg of weight and higher), the child only needs 20 mL per kg
of weight.
Take a 35 kg child, for example. He needs 1000 mL for his first 10 kg of weight (10 kg x
100 mL), 500 mL for his second 10 kg of weight (10 kg x 50 mL), and 20 mL/kg for any
weight above 20 kg (15 kg x 20 mL). A 35 kg child, therefore, needs approximately
1800mL of water or free liquids.
……………………………………………………………………………………………………………………………………………………..

373) Young patient with pharyngitis, inflammation of oral mucosa and lips that has whitish
cover and erythmatous base, febrile, splenomegaly. Dx: (this is infectious mono)
a. Scarlet fever
b. EBV
c. HZV

The correct answer is b

……………………………………………………………………………………………………………………………………………………..

374) Female with greenish vaginal discharge, red cervix. Dx:


Trchimoniosis

……………………………………………………………………………………………………………………………………………………..

375) Another female with malodorous discharge and pain maybe. Dx:
Bacterial vaginosis

……………………………………………………………………………………………………………………………………………………..
376) The best way to reduce the weight in children is:
a. stop fat intake
b. Decrease calories intake
c. Drink a lot of water

The correct answer is b


377) You have to advice a teenager that mainly eats fast food to take:
a. Folic acid and Ca
b. Vit C and Ca
c. Ca alone

The correct answer is c ( I'm not sure )

……………………………………………………………………………………………………………………………………………………..

378) Patient has symptoms of infection, desquamation of hands and feet, BP 170\110 dx:
a. Syphilis
b. Toxic shock syndrome
c. Scarlet fever

The correct answer is b

Toxic shock syndrome :


Caused by S.aureus , often with 5 days of onset of menestral period in women who
have used tampons. Feature: abrupt fever ( 39 c or more ), vomiting, diffuse macular
erythematous rash, desquamation especially in palms and soles, nonpurulent
conjunctivitis. Diagnosis : blood culture are –ve .so, diagnosis by clinical. Treatment :
1st step rehydration and antibiotic

……………………………………………………………………………………………………………………………………………………..
379) Snellin chart, if patient can read up to line 3, how much is his vision loss :
Q not complete:
but this pt. see letters at 20 feet , where normal person see it at 70 feet.

380) I cant remember the case but gram +ve cocci were isolated, dx is sterpt bcz it was the
only gm +ve cocci.

……………………………………………………………………………………………………………………………………………………..

381) Diabetic mother asking about risk of diabetes related congenital defect. It will be in:
a. 1st trimester
b. 2nd trimester
c. 3rd

The correct answer is a (I'm not sure )

……………………………………………………………………………………………………………………………………………………..

382) If diabetic mother blood sugar is always high despite of insulin, neonate complication
will mostly be:
a. Maternal hyperglycemia
b. Maternal hypoglycemia
c. Neonatal hypoglycemia
d. Neonatal hyperglycemia

The correct answer is c

……………………………………………………………………………………………………………………………………………………..

383) Obssive neurosis patients will have:


a. Major depression
b. Lake of insight
c. Schizophrenia
The correct answer is a

……………………………………………………………………………………………………………………………………………………..

384) Before giving bipolar patient lithium you will do all of the following except:
a. TFT
b. LFT
c. RFT
d. Pregnancy test

The correct answer is b


385) Patient came with PV small bleeding (she fell down and felt decrease in fetal
movement but on examination baby was ok) all her 3 previous pregnancies were
normal, you will do:
a. Immediate CS
b. Careful observation of the bleeding
c. Medication (strange name)
d. Mg sulphate

Q not complete , depend on date of gestation.

……………………………………………………………………………………………………………………………………………………..
386) Pt came to you missing her period for 7 wks, she had minimal bleeding and abdominal
pain, +ve home pregnancy test, 1st thing to order is:
a. BHCG
b. US
c. Drugs maybe

The correct answer is a

……………………………………………………………………………………………………………………………………………………..

387) Patient came to you and you suspect pre eclampisa, which of the following will make it
most likely:
a. Elevated blood pressure
b. Decrease fetal movement
c. ??

The correct answer is a

……………………………………………………………………………………………………………………………………………………..

388) Old patient male, hematuria, passing red clots and RT testicular pain:
a. Testicular Ca
b. RCC ( renal cell carcinoma )
c. Cystitis
The correct answer is b

389) to get more information from the pt


a. open end question

( avoid leading question to get more information )

……………………………………………………………………………………………………………………………………………………..
390) gingivits most likely cause
a.HSV

( q not complete but : The most common cause of gingivitis is poor oral hygiene that
encourages plaque to form.)

……………………………………………………………………………………………………………………………………………………..
391) watery discharge from eye , conjunctivitis treated by
a. topical corticosteroid

(Its symptoms include watery discharge and variable itch. The infection usually
begins with one eye, but may spread easily to the other
Ttt: cold compresses] and artificial tears)

……………………………………………………………………………………………………………………………………………………..
392) blow out fracture eyelid swelling , redness other syptoms
a. present air fluid level
b. enopthalmos

The correct answer is b


its enophaloms , other name of blow out orbital floor fracture

……………………………………………………………………………………………………………………………………………………..
393) drinking of dirty water causes
a. heptitis A
b. B
c. C
d. D

The correct answer is a

……………………………………………………………………………………………………………………………………………………..
394) giemsa stained blood film
a. malaria

395) regarding peritonitis


a.Complicated appendectomy the cause is anerobe organism
b. rigidity and the cause is paralytic ileus
c. can be caused by chemical erosions.

The correct answer is c

……………………………………………………………………………………………………………………………………………………..
396) fraucture of humerus associated with
a. radial N injury
b. median nerve injury

the correct answer is a ( radial groove )

……………………………………………………………………………………………………………………………………………………..

397) there is case i cant remember they asked about diagnosis of acute lymphocytic
leukemia ALL

The total number of white blood cells may be decreased, normal, or increased, but
the number of red blood cells and platelets is almost always decreased. In addition,
very immature white blood cells (blasts) are present in blood samples examined
under a microscope.
A bone marrow biopsyis almost always done to confirm the diagnosis and to
distinguish ALL from other types of leukemia

……………………………………………………………………………………………………………………………………………………..
398) diagnosis of hemochromatosis
a. serum ferritin?!
Hemochromatosis is suggested by a persistently elevated transferrin saturation in
the absence of other causes of iron overload. It is the initial test of choice .
Ferritin concentration can be high in other conditions, such as infections,
inflammations, and liver disease .
Ferritin levels are less sensitive than transferrin saturation in screening tests for
hemochromatosis .
399) kawasaki disease associated with
a.strawberry tongue

 Kawasaki disease
Multisystem acute vasculitis that primary affected young children. Fever plus
four or more of the following criteria for diagnosis:
1-fever > 40 C for at least five days
2-bilateral, nonexudative, painless conjunctivitis
3-polymorphous rash ( primarily truncal )
4-cervical lymphadenopathy ( often painful and unilateral )
5- diffuse mucous membrane erythema ( strawberry tongue ) , dry red
6- erythema of palm and sole
7- other maindfestation : gallbladder hydrops, hepatitis, arthritis

Untreated Kawasaki diease can lead to coronary aneurysms and even MI


Treatment :
1-high dos ASA ( for fever and inflammation) & IVIG ( to prevent aneurrysmal )
2-referral to pediatric cardiologist

……………………………………………………………………………………………………………………………………………………..
400) most common tumor in children
a.ALL
b. rabdomyosarcoma
c. wilm's tumor

the correct answer is a

-ALL : most common childhood tumor


-rabdomyosarcoma : most common soft tissue tumor
-wilm's tumor: most common intra- abdominal childhood tuomr

……………………………………………………………………………………………………………………………………………………..
401) Most common intra- abdominal tumor in children:
a.wilm's tumor
b. lymphoma

the correct answer is a


the most common intra-abdominal tumor in children are neruoblastoma and
wilm's tumor.
402) pt c/o of hypopigmentful skin , nerve thicking diagnosis
a.leprosy

Leprosy : chronic granulomatous infection caused by Mycobacterium leprae, acid-fast


bacillus preferentially affecting cooler regions of body ( e.g skin , mucousmembrane,
peripheral nerve) . diagnosis by skin lesion ( hypopigmentation or hyperpigmentation
macule ) and enlarge nerves eith sensory loss.

……………………………………………………………………………………………………………………………………………………..
403) regrading COPD to reduce complication we should give
a. theophyline
b. pneumococcal vaccine
c. smoking cessation.

The correcr answer is c

……………………………………………………………………………………………………………………………………………………..

404) pseud-gout its


a. CACO3
b. CACL3

The correct answer is a


Gout : deposition of ( MSUM ) monosodium urate monohydrate –ve of birefringent ,
needle shape,
Psudogout : deposition of ( cppd ) calcium pyrophosphates dehydrate crystal , +ve
birefringent , rhomboid shape, ( CACO3)

.
405) man with history of alcohol assocation with
a. high MCV
b. folic acid deficiency
c. B12 deficieny
e. hepatitis

The correct answer is b

……………………………………………………………………………………………………………………………………………………..
406) neanate 9 days on breast feed develop jandice
a. breast feed jaundice
b. pathological jaundice
c. physiological jaundice

the correct answer is a

1-pathological jaundice : conjugated ( direct ) bilirubin, start in 1st 24 hours, bilirubine >
15 mg / dl , persistent 1 week in term infant and 2 weeks in preterm infant

2-physilogical jaundice: unconjugated ( indirect ) bilirubine , start in 2- 3 days, bilirubine


< 15 mg/dl , persistent 1 week in term infant and 2 weeks in preterm infant.

3-breast feeding jaundice : unconjucated ( indirect ) bilirubine , start in 7 days of life ,


persistent to one or more months.

……………………………………………………………………………………………………………………………………………………..
407) lady c/o headche bandlike pain
a. tension headache

……………………………………………………………………………………………………………………………………………………..
408) regarding breast screening
a. self breast examination early detection of tumor
b. mammogram not advise before 35 y
??????

……………………………………………………………………………………………………………………………………………………..
409) 19 yrs old c/o abdo pain within menestration for last 6 years diagnosis
a. primary dysmenorrhea
b. secondary dysmenorrheal

The correct answer is a


Primary dysmenorrhea :
Onset within 6 months after menarche . Lower abdominal/pelvic pain begins with
onset of menses and lasts 8-72 hours
410) bilateral breast mass diagnosis
a. ductal carcinoma
b. pagets disease
it could be Intraductal papilloma ??????

……………………………………………………………………………………………………………………………………………………..
411) beriberi cause of deficiency
a. VIT B1
b. VIT B2
c. VIT B3

The correct answer is a


……………………………………………………………………………………………………………………………………………………..
412) chronic uses of estrogen association ?????

……………………………………………………………………………………………………………………………………………………..
413) Asystole
a. adrenalin
b. atropine

The correct answer is


asystol has only 3 durgs epinephrine-vasopressine-atropin

……………………………………………………………………………………………………………………………………………………..
414) pt has diarrhea , dermatitis and dementia diagnosis
a. pellagra

……………………………………………………………………………………………………………………………………………………..
415) regarding injectable progestron
a . an cause skin prob ?!!

Injected progesterone ( medroxy-progestrone ) : advantages : light or no periods,


safe with breastfeeding, IM injection every 3 months. Disadvantages: irregular
bleeding, wt. gain, decrease bone mineral density ( reversible ) , delay fertility
discontinue .

……………………………………………………………………………………………………………………………………………………..
416) sencodary prevention is
a. coronary bypass gaft

Secondary prevention generally consists of the identification and interdiction of


diseases that are present in the body, but that have not progressed to the point of
causing signs, symptoms, and dys-function
417) 22 yrs old c/o insomina/ sleep disturb treatment
a. SSRI

Short history ,
But the initial management of insomnia : good sleep hygiene .

……………………………………………………………………………………………………………………………………………………..
418) celluitis in children most commin causes
a. group A steptoccus

In children, the most common cause of cellulitis is S aureus or group A


streptococcus.

……………………………………………………………………………………………………………………………………………………..
419) antideprssants associated with hypertensive crisis treatment
a. SSRI
b. MOAIs
c. TCAs

the correct answer is b


esp hydrazines ( phenelzine )

……………………………………………………………………………………………………………………………………………………..

420) pt his MBI = 24 kg he is


a. noramal weight
b. over weight
c. morbid weight
d. mild weight

The correct answer is a

……………………………………………………………………………………………………………………………………………………..
421) baby c/o fever , chills , rigors and head rigidity +ve kurnings sign rash on his lower
limb diagnosis :
- meningoccal meninigits
???????

……………………………………………………………………………………………………………………………………………………..
422) 48 yrs pt with abdo pain , neusea, vominting tenderness in right hypochondrial
- acute cholecysitis
Fever ?!
423) 29 pt c/o dysurea his microscopic showed G -ve organism is
a. legonealla
b. E. coli

the correct answer is b

……………………………………………………………………………………………………………………………………………………..
424) 30 yrs pt c/o feeling heaviness in the lower abdomen having pulge papable at
the top scrotum that was reducible and icreasing in valsalva maneuver diagnosis
a. hydrocele
b. variocele
c. indirect inguinal hernia
d. direct inginal hernia

The correct answer is c

……………………………………………………………………………………………………………………………………………………..
425) anticoagulation prescrib for
a. one month
b. 6 months
c. 6 weeks
d. one year

The correct amswer is b

……………………………………………………………………………………………………………………………………………………..
426) cushing syndrome best single test to confirm
a. palsma cortison
b. ATCH
c. Dexamethasone Suppression Test

The correct answer is c

……………………………………………………………………………………………………………………………………………………..

427) 23 yrs old history of URTI then he developed ecchomysis best treated
a. local AB
b. local antiviral
c. steroid

The correct answer is c

428) chronic psycotic disorder manged by


a. haloperidol

……………………………………………………………………………………………………………………………………………………..
429) 29 yrs old lady B-HSG 160 c/o vomiting , abdomenal pain which is more accurate
to diagnosis
a. BHCG serial
b. pelvic US
c. laprascopy
Q not complete, but may be this case is molar and chooce is a .

……………………………………………………………………………………………………………………………………………………..
430) 70 yrs old man c/o fever , vesicular rash over forehead mangement
a. IV AB
b. IV antiviral
c. Acyclovir

the correct answer is c (i"m npt sure )

……………………………………………………………………………………………………………………………………………………..
431) celiac disease involves :
a. proximal part of small intestin
b. distal part of small intestin
c. proximal part of large intesin
d. distal part of large intensin

The correct answer is a

……………………………………………………………………………………………………………………………………………………..
432) 6 yes old pt cyanosis past history of similar attack 6 month ago u will do for him
a. CxR
b. PFT
c. secure airway
d. CBC

Q is not clear or not complete , if pt. came with life threatening cyanosis don't waste
time with investigation and start with ABC ( scure airway ) , but if pt. came with past
history cyanosis choose b .

433) side affect of diazepam


Sedation , depence, respiratory supression , anterograde amnesia , confusion
(especially pronounced in higher doses) and sedation

……………………………………………………………………………………………………………………………………………………..

434) endemic means:

Endemic :is the constant presence of a disease or infectious agent in a certain


geographic area or population group. ( usually rate of disease )
Epidemic : is the rapid spread of a disease in a specific area or among a certain
population group. ( excessive rate of disease )

Pandemic : is a worldwide epidemic; an epidemic occurring over a wide


geographic area and affecting a large number of people.

……………………………………………………………………………………………………………………………………………………..

435) Epidemic curve :

a graph in which the number of new cases of a disease is plotted against an interval
of time to describe a specific epidemic or outbreak .

436) Pt with hodgkin's lymphoma , and red strunberg cell in pathology and there is
esinophil lymphocyte in blood so pathological classification is:

a. Mixed-cellularity subtype

b. nodular sclerosis subtype of Hodgkin's lymphoma

the correct answer is b

Classical Hodgkin's lymphoma can be subclassified into 4 pathologic subtypes


based upon Reed-Sternberg cell morphology and the composition of the reactive
cell infiltrate seen in the lymph node biopsy specimen (the cell composition
around the Reed-Sternberg cell(s))
.

Name descreption

Nodular sclerosing CHL Is the most common subtype and is composed of large tumor nodules showing scattered
lacunar classical RS cells set in a background of reactive lymphocytes, eosinophils and
plasma cells with varying degrees of collagen fibrosis/sclerosis.

Mixed-cellularity subtype Is a common subtype and is composed of numerous classic RS cells admixed with
numerous inflammatory cells including lymphocytes, histiocytes, eosinophils, and plasma
cells. without sclerosis. This type is most often associated with EBV infection and may be
confused with the early, so-called 'cellular' phase of nodular sclerosing CHL

Lymphocyte-rich or Is a rare subtype, show many features which may cause diagnostic confusion with
Lymphocytic predominance nodular lymphocyte predominant B-cell Non-Hodgkin's Lymphoma (B-NHL). This form
also has the most favorable prognosis

Lymphocyte depleted Is a rare subtype, composed of large numbers of often pleomorphic RS cells with only few
reactive lymphocytes which may easily be confused with diffuse large cell lymphoma.
Many cases previously classified within this category would now be reclassified under
anaplastic large cell lymphoma

437) 62 y male with DVT and IVC obstruction due to thrombosis so most like dd is
a. neohrotic syndrome
b. SLE
C. Chirstm disease
D. ?!

……………………………………………………………………………………………………………………………………………………..
438) Pt with bdominal pain heamatutea , HTN, and have abnormalty in chromosm
16 , diagnosis is
a. POLY CYCTIC KIDNEY

……………………………………………………………………………………………………………………………………………………..

439) 17 year pt with dyspnea Po2 , PCO2 ,Xray normal PH increase so dd is:

a. acute attack of asthma


b. P E
c. pneumonia
d. pnemothrax

the correct answer is a

……………………………………………………………………………………………………………………………………………………..

440) A long scenario about patient with polydipsia ad polyuria. Serum osmolrity high .
desmoprsin inductin no change urine omolarity and plasma osmolrity so dd is
a. nphrognic type
b. central tupe

The correct answer is a

……………………………………………………………………………………………………………………………………………………..

441) 50 year old Man presented to ER with sudden headach, blurred of vision and eye
pain. The diagnosis is:
a. Acute glaucoma
b. Acute conjunctivitis
c. Corneal ulcer
d. -----

The correct answer is a

442) RTA with hip dislocation and shock so causes of shock is


a. blood lose
b. urtheral injery
c. nurogenic

The correct answer is a

……………………………………………………………………………………………………………………………………………………..
443) most common causes of hand infection
a. truma
b. imunocrombromise
c. ………

the correct answer is a

……………………………………………………………………………………………………………………………………………………..

444) ttt of cholestatoma is


a. antibiotic
b. steroid
c. surgery
d. Grommet tube

the correct answer is c

-Cholesteatoma is a destructive and expanding growth consisting of keratinizing


squamous epithelium in the middle ear and/or mastoid . treatment : Surgery is
performed to remove the sac of squamous debris and a mastoidectomy is
performed.

445) twins one male and other female . his father notice that femle become puberty
before male so what you say to father

a. female enter puberty 1-2 year before male


b. female enter puberty 2-3 year before male
c. female enter puberty at the same age male

the correct answer is b

……………………………………………………………………………………………………………………………………………………..
446) in devolping countery to prevent dental carise , it add to water
a. florid
b. zink
c. copper
d. iodide

the correct answer is a

……………………………………………………………………………………………………………………………………………………..

447) 12 y.o boy c/o abdominal pain after playing football,he denied any h/o trauma
,the pain is in the Lt paraumbilical region what inx you want to do:
a. CXR
b. ultrasound kidney

The correct answer is b , I'm not sure because MCQs are not compete.

……………………………………………………………………………………………………………………………………………………..

448) pt child with back pain that wake pt from sleep


So diagnosis
a. lumber kyphosis
b. osteoarthritis
c. RA
d. Scoliosis

the correct answer is d

449) child with papule vesical on oropharnx and rash in palm and hand so dd:

a. CMV
b. EBV
c. MEASLS
d. ROBELLA

???? Coxsackie A virus. Vesical in buccal mucusa is measls.

……………………………………………………………………………………………………………………………………………………..
450) chid with dental caries and history of bottle feading
So dd
a. nurse milk caris

……………………………………………………………………………………………………………………………………………………..

451) Generalize antxity disorder best ttt: -19


a. Ssri
b. tricyclic a d
d. MAOI

The correct answer is a

……………………………………………………………………………………………………………………………………………………..

452) Major depression management:


a. Intial MONOTHERABY even sever sever deprisson
b. Ttt should be change if no response during 2wk (AT LEAST 6 . WEEKS )

????

……………………………………………………………………………………………………………………………………………………..

453) psychotherapy, medication, and electroconvulsive therap


Psychiatric pt with un compliance of drugs ttt:

a. depro halopredol injection


b. oral colonazepam

the correct answer is a

454) mild diarrhea mangement :


a. oral antibiotis
b. IVF
c. ORS

the correct answer is c

……………………………………………………………………………………………………………………………………………………..

455) salpingitis and pid on penicillin but not improve the most likely organism is :
a. chlamydia
b. nessiria
c. SYPHLIS
d. HSV

the correct answer b

……………………………………………………………………………………………………………………………………………………..
456) Used for treatment of pseudomembranous colitis:

PO metronidazole is the empiric treatment of choice (500 mg TID for 10-14


days). For severe infections, oral Vancomycin is the treatment of choice (125 mg
QID 10-14 days).

……………………………………………………………………………………………………………………………………………………..
457) Lady with of right hypochondrial pain, fever and slight jaundice. What is your
diagnosis?

a. Acute cholecystitis.

Q not complete.

Most common presenting symptom of acute cholecystitis is upper abdominal


pain often radiating to right scapula. Nausea and vomiting generally present.
Patients may report fever.

458) Deep jaundice wit palpable gallbladder ?

.a. Cancer head of pancreas .i

Q not complete

Pancreatic cancer presents with abdominal pain radiating toward the back, as
well as with
jaundice, loss of appetite, nausea, vomiting, weight loss, weakness, fatigue, and
indigestion.
Examination: may reveal a palpable, nontender gallbladder (Courvoisier’s sign)
or migratory thrombophlebitis (Trousseau’s sign). Diagnose by CT. treatment
usually palliative (due to metastasis at Dx).
……………………………………………………………………………………………………………………………………………………..

459) Patient with perforated gallbladder underwent cholecystectomy. Return to you


with fever. On abdominal X-ray, there is elevation of right hemidaiaphragm. What
is the possible diagnosis?

a. Subphrenic abcess

…………………………………………………………………………………………………………………………………………………….
.

Patient with acute perianal pain since 2 days with black mass (460 (460
:2*3 pain 4increase with defecation Rx
a. Evacuation under local anesthesia

Q not complete

……………………………………………………………………………………………………………………………………………………..

461) Rhumatoid arthritis distal interphalengial nodules


a. Hebrerden's nodes

Heberden's nodes are at the DIP while Bouchard's nodes are at PIP.

462) Elderly patient with RLQ fullness, weight loss, changed bowel habit, anemic and
pale. What is the investigation of choice?

a. Colonoscopy.

This is now the investigation of choice if colorectal cancer is suspected provided the
patient is fit enough to undergo the bowel preparation. It has the advantage of not
only picking up a primary cancer but also having the ability to detect synchronous
polyps or even multiple carcinomas, which occur in 5% of cases.
…………………………………………………………………………………………………………………………………………………….
.
463) Investigation of choice in Iron deficiency anemia?
a. serum iron
b. ferritin
c. TIBC

the correct answer is b

Serum ferritin is the most sensitive lab test for iron deficiency anemia.
…………………………………………………………………………………………………………………………………………………….
.

464) In patients with hypertension and diabetes, which antihypertensive agent you
want to add first?
a. β-blockers
b. ACE inhibitor
c. α-blocker
d. Calcium channel blocker

the correct answer is b

Diuretics (inexpensive and particularly effective in African-Americans) and β-


blockers (beneficial for patients with CAD) have been shown to reduce mortality in
uncomplicated hypertension. They are first-line agents unless a comorbid
condition requires another medication. (see table)

Population Treatment
Diabetes with proteinuria ACEIs.
CHF β-blockers, ACEIs, diuretics (including
spironolactone).
Isolated systolic Diuretics preferred; long-acting dihydropyridine
hypertension calcium channel blockers.
MI β-blockers without intrinsic sympathomimetic
activity, ACEIs.
Osteoporosis Thiazide diuretics.
BPH α-antagonists.

465) Patient presented with retinal artery occlusion, which of the following is wrong:
a. painful loss of vision
b. painless loss of vision

the correct answer is a

The most common presenting complaint of retinal artery occlusion is an acute


persistent painless loss of vision.
…………………………………………………………………………………………………………………………………………………….
.
466) ECG finding of acute pericarditis?
a. ST segment elevation in all leads

Classic ECG findings in pericarditis: Low-voltage, diffuse ST-segment elevation.


…………………………………………………………………………………………………………………………………………………….
.
467) Epidemiological study want to see the effect of smoking in the bronchogenic
carcinoma, they saw that is 90% of smokers has bronchogenic carcinoma.
30% of non–smokers has the disease, the specificity of the disease as a risk factor
is:
a. 70 %
b. 30 %
c. 90 %

MCQs not complete

Sensitivity: The probability that a diseased patient will have a positive test result.
Specificity: The probability that a nondiseased person will have a negative test result.

Disease No Disease
Present
Positive test a b
Negative c d
test

Sensitivity = a / (a + c) Specificity = d / (b + d)

Note:
- A sensitive test is good for ruling out a disease.
- High sensitivity = good screening test ( false negatives).
- High specificity = good for ruling in a disease (good confirmatory test).

468) Baby present with pain in the ear, by examination there is piece of a glass deep
in his ear canal, the mother mention a history of a broken glass in the kitchen but
she clean that completely. We treat that by:
a. By applying a stream of solution to syringing the ear.
b. Remove it by forceps.
c. Refer her to otolaryngology

* Foreign bodies in the ear canal:


Techniques appropriate for the removal of ear foreign bodies include mechanical
extraction, irrigation, and suction. Irrigation is contraindicated for organic matter
that may swell and enlarge within the auditory canal (e.g seeds). Insects, organic
matter, and objects with the potential to become friable and break into smaller
evasive pieces are often better extracted with suction than with forceps. Live insects
in the ear canal should be immobilized before removal is attempted.

Consultations: Consult an ENT specialist if the object cannot be removed or if tympanic


membrane perforation is suspected.

…………………………………………………………………………………………………………………………………………………….
.
469) Treatment of open tibial fracture:
a. cephazolin
b. cephazolin+gentamycin
c. gentamicin
d. cephazolin+gentamicin+metronidazole

treatment of open fracture

1- First-generation cephalosporins (Gram-positive coverage) such as cephalexin (1-2


g q6-8h) suffice for Gustilo type I open fractures.
2- An aminoglycoside (Gram-negative coverage) such as gentamycin (120 mg q12h;
240 mg/d) is added for types II and III injuries.
3- Additionally, metronidazole (500 mg q12h) or penicillin (1.2 g q6h) can be added
for coverage against anaerobes.
4- Tetanus prophylaxis should be instituted.
5- Antibiotics generally are continued for 72 hours following wound closure.

Open fracture classification


Gustilo classification
I Low energy, wound less than 1 cm
II Wound greater than 1 cm with moderate soft tissue damage
High energy wound greater than 1 cm with extensive soft tissue damage
IIIa Adequate soft tissue cover
III
IIIb Inadequate soft tissue cover
IIIc Associated with arterial injury

470) Which of the following found to reduce the risk of postherpetic neuralgia:
a) corticosteroids only
b) corticosteroids + valacyclovir
c) valacyclovir only

The correct answer is c

Patients with herpes zoster should receive treatment to control acute symptoms
and prevent complications. Patients over the age of 50, irrespective of other risk
factors, are at much greater risk of developing postherpetic neuralgia and should be
offered treatment. By inhibiting replication of varicella zoster virus, the antiviral
agents acyclovir, famciclovir, and valaciclovir attenuate the severity of zoster—
specifically, the duration of viral shedding is decreased, rash healing is hastened,
and the severity and duration of acute pain are reduced. Attenuation of the severity
of the acute infection and the neural damage it causes should reduce the likelihood
of postherpetic neuralgia.

…………………………………………………………………………………………………………………………………………………….
.

471) 8 months old infant with on & off recurrent crying episodes & hx of current jelly
stools:
a) intussception
b) intestinal obstruction
c) mickel's diverticulitis
d) strangulated hernia

The correct answer is a

472) a man with oblong swelling on top of scrotum increase in size with valsalva
maneuver most likely Dx:
a) direct inguinal hernia
b) indirect inguinal hernia
c) varicocele
d) femoral hernia

the correct answer is b


* Indirect hernia: Herniation of abdominal contents through the internal and then
external inguinal rings and eventually into the scrotum (in males).
- The most common hernia in both genders.
- Due to a congenital patent processus vaginalis.
- Indirect inguinal hernia increases in size it becomes apparent when the patient
coughs, and persists until reduced.

* Direct hernia: Herniation of abdominal contents through the floor of Hesselbach’s


Triangle. (Hesselbach’s triangle is an area bounded by the inguinal ligament, inferior
epigastricartery, and rectus abdominis)
- Hernial sac contents do not traverse the internal inguinal ring; they herniate
directly through the abdominal wall and are contained within the aponeurosis of
the external oblique muscle.
- Most often due to an acquired defect in the transversalis fascia from mechanical
breakdown that ↑ with age.
……………………………………………………………………………………………………………………………………………………
..
473) 3 weeks old male newborn with swelling of scrotum transparent to light &
irreducible:
a) epidydemitis
b) hydrocele

The correct answer is b

……………………………………………………………………………………………………………………………………………………..
474) a young girl experienced crampy abdominal pain & proximal muscular weakness
but normal reflexes after receiving septra (trimethoprim sulfamethoxazole) :
a) functional myositis
b) polymyositis
c) guillian barre syndrome
d) neuritis

the correct answer is d

475) Which drug causes SLE like syndrome:


a) hydralazine
b) propranolol
c) amoxicillin

The correct answer is a

- Drug-induced SLE causes: Chlorpromazine, Hydralazine, Isoniazid, Methyldopa,


Penicillamine, Procainamide, Quinidine, Sulfasalazine.
…………………………………………………………………………………………………………………………………………………….
.
476) When to give aspirin and clopidogrel?
a) pt with a hx of previous MI
b) Acute MI
c) hx of previous ischemic stroke
d) hx of peripheral artery disease
e) after cardiac capt

The correct answer is b

…………………………………………………………………………………………………………………………………………………….
.
477) 17 years with history of right iliac fossa pain rebound tenderness +ve guarding
what is the investigation that you will do?

a. Laparoscopy

b. US

c. CT scan

the correct answer is c

In appendicitis, CT scan with contrast has 95–98% sensitivity which shows


periappendiceal streaking.

478) Child with history of URTI before 3 weeks now has both knees tender, red and
inflamed. What is your diagnosis?

a. Rheumatoid arthritis

Q is not complete

The diagnosis of JRA is established by the presence of arthritis, the duration of the
disease for at least 6 weeks, and the exclusion of other possible diagnoses.
Although a presumptive diagnosis of systemic-onset JRA can be established for a
child during the systemic phase, a definitive diagnosis is not possible until arthritis
develops. Children must be younger than 16 years old at time of onset of disease;
the diagnosis of JRA does not change when the child becomes an adult. Because
there are so many other causes of arthritis, these disorders need to be excluded
before providing a definitive diagnosis of JRA (Table 89-2). The acute arthritides can
affect the same joints as JRA, but have a shorter time course. In particular, JRA can
be confused with the spondyloarthropathies, which are associated with spinal
involvement, and enthesitis, which is inflammation of tendinous insertions. All of
the pediatric spondyloarthropathies can present with peripheral arthritis before
other manifestations and initially may be diagnosed as JRA (Table 89-3).

* Juvenile rheumatoid arthritis: mono- and polyarthropathy with bony destruction


that occurs in patients ≤ 16 years of age and lasts > 6 weeks. Approximately 95% of
cases resolve by puberty. Subtypes: Pauciarticular: ≤ 5 joints involved, associated
with an risk of iridocyclitis that lead to blindness if left untreated. Polyarticular:
≥5 small joints. Systemic features are less prominent; carries a risk
of iridocyclitis. Acute febrile: The least common subtype; manifests as arthritis
withdaily high, spiking fevers and an evanescent, salmon-colored rash.
Hepatosplenomegaly and serositis may also be seen. No iridocyclitis is present;
remission occurs within one year.

…………………………………………………………………………………………………………………………………………………….
.
479) Old female came complain from pan in her joint increase with walking, what is
your diagnosis?
a. Osteoarthritis

Physical signs of OA: Crepitus,  range of motion, pain that worsens with activity
and weight bearing but improves with rest.

480) Patient came with history of neck discomfort, palpitation, cold sweat, TSH low,
T4 high, tender neck. What is your diagnosis?

a. Subacute thyroiditis.

- Thyroiditis: Inflammation of the thyroid gland. Common types are subacute


granulomatous, radiation, lymphocytic, postpartum, and drug-induced (e.g.,
amiodarone) thyroiditis.
- Hx/PE: In subacute and radiation, presents with tender thyroid, malaise, and URI
symptoms.
- Dx: Thyroid dysfunction (typically hyperthyroidism followed by hypothyroidism),
all with ↓ uptake on RAIU.
- Rx:
- β-blockers for hyperthyroidism; levothyroxine for hypothyroidism.
- Subacute thyroiditis: Anti-inflammatory medication.
…………………………………………………………………………………………………………………………………………………….
.
481) In ulcerative colitis, what you will start treatment with?
a. Corticosteroids ??

- Sulfasalazine or 5-ASA (mesalamine).


- Crticosteroids and immunosuppressants indicated if no refractory disease.
…………………………………………………………………………………………………………………………………………………….
.
482) Treatment of prostatitis?
a. Ciprofloxacin

treatment is for 4–6 weeks with drugs that penetrate into the prostate, such as
trimethoprim or ciprofloxacin.

……………………………………………………………………………………………………………………………………………………..
483) In IV canula and fluid:
a.Site of entry of cannula is a common site of infection.
…………………………………………………………………………………………………………………………………………………….
.
484) Patient with neck rigidity, rigor, fever, petechial rash over extremities. The
causative m\o is:
a. Meningococcal meningitis

Discuss meningitis !
…………………………………………………………………………………………………………………………………………………….
.
485) Female patient works in office came with vulva itching, yellow vaginal discharge:

a. Trichomoniasis

486) another one old c/o bilateral knee pain with mild joint enlargement
ESR and CRP normal dx:
a. Osteoarthritis
b. Rheumatoid arthritis
c. Gout?

The correct answer is a

……………………………………………………………………………………………………………………………………………………..
487) .Scitica increased incidence of :
a. Lumbar lordosis
b. Parasthesis
Q not complete but with these MCQs , correct answer is b

……………………………………………………………………………………………………………………………………………………..
488) old male c/o knee pain on walking with crepitus xray show narrow joint space
and subchondoral sclerosis:
a. Rheumatoid arthritis
b. Osteoarthritis
c. Gout

The correct answer is b

……………………………………………………………………………………………………………………………………………………..

489) 14y f with BMI 32.6 (associated big chart):


a. Overweight
b. Obese
c. Normal weight

The correct answer is b

BMI < 16 : severe under WT.


BMI 16 – 20 : under wt.
BMI 20 – 25 : normal
BMI 25 – 30 : over wt.
BMI 30 – 35 : obese classic 1
BMI 35 – 40 : obese classic 2
BMI > 40 : obese classic 3
490) vaginal discharge odorless watry microscopy show clue cells :
a. Bacterial vaginosis
b. Candidiasis

The correct answer is a

……………………………………………………………………………………………………………………………………………………..

491) Pregnant women on 3rd trimester infected with measels can I give her MMR?????

……………………………………………………………………………………………………………………………………………………..

492) lactating women infected with rupella ……..management is :


a. MMR
b. Stop lactation
The correct answer is a
MMR is safe during lactating

……………………………………………………………………………………………………………………………………………………..

493 ) which of following not live vaccine:


a. BCG
b. Hepatitis B
c. Oral polio
d. MMR

The correct answer is b

……………………………………………………………………………………………………………………………………………………..

493) the most commone cause of community acquired pneumonia:


a. Hemophilus influenza
b. Strept.pneumonia
c. Mycoplasma
d. Kliebsella

The correct answer is b

494) female after vaginal hesterectomy she complain of urin come from vagina………
dx:
a. Vesicovaginal fistula
b. Urethrovaginal fistula
c. Ureterovaginal fistula

The correct answer is a

……………………………………………………………………………………………………………………………………………………..

495) child rt ear pain and tenderness on pulling ear , no fever , O/E inflamed
odemateous rt ear canal with yellow discharge >>>>>>>>>>dx:
a. Otitis media
b. Otitis externa
c. Cholesteatoma

The correct answer is b

……………………………………………………………………………………………………………………………………………………
..
496) male with perianal pain ,tenderness ,fluctuant (perianal abscess)
a. Incision and drainage
b. Warm bath
The correct answer is a

…………………………………………………………………………………………………………………………………………………
…..
497) . pregnant 34w with vaginal bleeding………. what would you ask about :
a. Cigarette smoking
b. Recent sexual intercourse
Q not complete , but with theses MCQs may be correct answer is a

……………………………………………………………………………………………………………………………………………………
..
498) In duodenal obstruction of neoborn what is the sign that apper in xray:
a. Double bubble

499) the following more commone with type2 DM than type1 DM:
a. Weight loss
b. Gradual onset
c. Hereditary factors
d. HLA DR3+-DR4

The correct answer is c

……………………………………………………………………………………………………………………………………………………..
500) pt with open angle glaucoma and k/c of COPD and DM ttt:
a. Timelol
b. betaxolol
c. Acetazolamide

The correct answer is c

……………………………………………………………………………………………………………………………………………………..

501) pt child with back pain that wake pt from sleep So diagnosis
a. lumber kyphosis
b. osteoarthritis
c. RA
d. Scoliosis

The correct answer is a or d


…………………………………………………………………………………………………………………………………………

502) 59 y/o presented with new onset supraventicular tachycardia with


palpitation,no Hx of SO Or chest pain ,chest examination normal , oxygen sat in
room air = 98% no peripheral edemaOthers normal, the best initial investigation:

a. ECG stress test


b. Pulmonary arteriography
c. CT scan d.
Thyroid stimulating hormone

the correct answer is d

503) Lactating women presented with breast engorgment and tendress Your
managements:

a. Warm compressor and continoue breast feeding

b. Dicloxacillin and continoue breast feeding

c. Dicloxacillin and milk expression

d. Discontinoue breast feeding and cold compressor

the correct answer is a . tis case is ( engorgement of breast ) . but if there are
redness , swelling , fever this case is ( mastitis ) and choose b

504) The Fastest route of antipsychotic is:

a. IM

b. IV

c. Oral

d. Sublingual

the correct answer is b


505) While you are in the clinic you find that many patients presents with red follicular
conjactivitis (Chlamydia ) your management is:

a. Improve water supply and sanitation

b. Improve sanitation and destroying of the vector

c. Eradication of the reservoir and destroying the vector

d. Destroy the vector and improve the sanitation

the correct answer is a

506) The most important exogenous risk factor for osteoporosis is:

a. Alcohol intake

b. Age

c. Smoking

d. Lack of exercise

the correct answer is c

507) Patient with family history of coronary artery disease his BMI= 28 came to you
asking for the advice:

a. Start 800 calorie intake daily

b. Decrease carbohydrate daytime

c. Increase fat and decrease protein

d. Start with decrease ……. K calorie per kg per week

the correct answer is d


508) Lactating mother newly diagnosed with epilepsy , taking for it phenobarbital you
advice is:

a. Discontinue breastfeeding immediately

b. Breastfeed baby after 8 hours of the medication

c . Continoue breastfeeding as tolerated

the correct answer is c

-very vague question , some books avoid Phenobarbital during breast feeding if
possible. And in American academy of pediatric classified Phenobarbital as adrug
that cause major advers effect in some nursing infant, and should be given to
nursing women with cation .

509) Pregnant women has fibroid with of the following is True:

a. Presented with severe anemia


b. Likely to regress after Pregnancy
c. Surgery immediately
d. Presented with Antepartum He

the correct answer is b

Fibroids may also be the result of hormones. Reproductive hormones like


estrogen and progesterone can stimulate cell growth, causing fibroids to form.
During pregnancy, your influx of hormones may cause your fibroids to grow in
size. After pregnancy and during menopause most fibroids begin to shrink, due
to a lack of hormones.

510) A known case of chronic atrial fibrillation on the warfarin 5 mg came for follow up
you find INR 7 but no signs of bleeding you advice is:

a. Decrease dose to 2.5 mg


b. Stop the dose and repeat INR next day
c. Stop warfarin
d. Continoue same and repeat INR

the correct answer is b


INR ACTION

>10 Stop warfarin. Contact patient for examination. MONITOR INR

7-10 Stop warfarin for 2 days; decrease weekly dosage by 25% or by 1 mg/d for next week (7 mg
total); monitor INR

4.5-7 Decrease weekly dosage by 15% or by 1 mg/d for 5 days of next week (5 mg total); repeat
monitor INR

3-4.5 Decrease weekly dosage by 10% or by 1 mg/d for 3 days of next week (3 mg total); repeat
monitor INR.

2-3 No change.

1.5-2 Increase weekly dosage by 10% or by 1 mg/d for 3 days of next week (3 mg total);

<1.5 Increase weekly dose by 15% or by 1 mg/d for 5 days of next week (5 mg total);

511) Patient is a known case of CAD the best exercise:

a. Isotonic exercise

b. Isometric exercise

c. Anerobic exe

d. Yogha

the correct answer is c

1-anerobic exercise ( endurance ) : for improve cardiac function


2- weight bearing excercise ( isometric ) : for build muscle strength , bone density
3- stretching excercise : for prevent cramp , stiffness and back pain

512) The mechanism of action of Aspirin:

a. Inhibit cycloxgenase

b. Inhibit phospholipase A2

c. Inhibit phospholipid D

the correct answer is a


513) The absolute contraindication of breastfeeding is :

a. Asymptomatic HIV patient

b. Active hepatitis C

c. Pulmonary TB on treatment 3 months

The correct answer is a

Absolute contraindication of breastfeeding :

1-Infants with galactosemia.

2-Mothers who use illegal drugs.

3-Mothers infected with HIV, human T-cell lymphotropic virus type I


or type II, or who have an active herpes lesion on the breast.

4-Mothers taking any of the following medications: radioactive


isotopes, cancer chemotherapy agents, such as antimetabolites
.

514) A boy felt down on his elbow , the lateral x-ray shows:

a. Anterior Pad sign

b. Posterior pad sign

c. Anterior line of humerous intersecting the cubilium

d. Radial line forming 90 degree with cubilium

the correct answer is b

515) A known case of treated hodgkin lymphoma(mediastinal mass) with radiotherapy


Not on regular follow up presented with gradual painless difficulty in swallowing
and SOB , There is facial swelling and redness : DX

a. SVC obstruction

b. IVC obstruction

c. Thoracic aortic aneurysm

d. Abdominal aortic aneursm


The correct answer is a

________________________________________________________________________

516) Patient is presented with hand cellulitis and red streaks in the hand and tender
axillary lymphadenopathy. This condition is more likely to be associated with:

a. Malignancy

b. Pyoderma

c. Neuropathy

d. Lymphangitis
the correct answer is b (I'm not sure )

516) Young aged male presented to ER after blunt trauma to Abdomen, CT scan shows
intramural hematoma: your management is

a. Lapratomy with evacuation of the hematoma

b. Dissection of duodenum

c. Observation

the correct answer is c

517) Patient presented with sore throat, anorexia, loss of appetite , on throat exam
showed enlarged tonsils with petechi on palate and uvula , mild tenderness of
spleen and liver :DX

a. Group A strep
. b. EBV

The correct answer is b

518) Patient with GERD has barret esophagus , this metaplasia increase risk of :

a. Adenocarcinoma
b. Squmaou cell carcinoma
the correct answer is a

519) Complication of Sleep apnea is :

a.CHF
b. ……

the correct answer is a

sleep apnea : Hypoxic pulmonary vasoconstriction  PAH  Cor Pulmonale  CHF

complication of sleep apnea : sleep apnea increases health risks such as


cardiovascular disease, high blood pressure, stroke, diabetes, clinical depression, weight
gain and obesity. The most serious consequence of untreated obstructive sleep apnea is to
the heart. In severe and prolonged cases, there are increases in pulmonary pressures that
are transmitted to the right side of the heart. This can result in a severe form of congestive
heart failure (cor pulmonale).

520) Which of the following medication can be used as prophylaxis in appendectomy:

a. Cephalexin

b. Ceftriaxone

c. Metronidazole

d. Vancomycin

e. Ampicillin

The correct answer is b

1st line of antibiotic : 1- cefoxitin 2- cefotetan

2nd line of treatment : 1- metradinazole 2- ampicikkin-sulbactam

521) Which of the following prognostic factor for SLE:

a. ANA levels

b. Sex

c. Age

d. Renal involvement
The correct answer is d

522) The most common site for osteomyelitis is:

a. Epiphysis

b. Diaphysis

c. Metaphysis

d. Blood flow

The correct answer is c

523) In “holding breath holding” which of the following True:

a. Mostly occurs between age of 5 and 10


b. Increase Risk of epilepsy
c. A known precipitant cause of generalized convulsion
d. Diazepam may decrease the attack

Breath holding spells are the occurrence of episodic apnea in children, possibly associated with
loss of consciousness, and changes in postural tone. They are most common in children between
6 and 18 months and usually not present after 5 years of age. They are unusual before 6 months
of age. A positive family history can be elicited in 25% of cases. It may be confused with a seizure
disorder.

There are four types of breath holding spells.

1-The most common is termed simple breath holding spell, in which the manifestation is the
holding of breath in end expiration. There is no major alteration of circulation or oxygenation
and the recovery is spontaneous.

2-The second type are the Cyanotic breath-holding spells. They are usually precipitated by anger
or frustration although they may occur after a painful experience. The child cries and has forced
expiration sometimes leading to cyanosis (blue in color), loss of muscle tone, and loss of
consciousness. The majority of children will regain consciousness. The child usually recovers
within a minute or two, but some fall asleep for an hour or so. Physiologically, there is often
hypocapnea (low levels of carbon dioxide) and usually hypoxia (low levels of oxygen. There is no
"post ictal" phase (as is seen with seizures), no incontinence, and the child is fine in between
spells. EEGs are normal in these children. There is no relationship to the subsequent
development of seizures or cerebral injury as a consequence of breath holding spells.

3-In the third type, known as Pallid breath-holding spells, the most common stimulus is a
painful event. The child turns pale (as opposed to blue) and loses consciousness with little if any
crying. The EEG is also normal, and again there is no post ictal phase, nor incontinence. The child
is usually alert within a minute or so. There may be some relationship with adulthood syncope in
children with this type of spell.

4- A fourth type, known as Complicated breath-holding spells, may simply be a more severe
form of the two most common types. This type generally begins as either a cyanotic or pallid
spell that then is associated with seizure like activity. An EEG taken while the child is not having a
spell is still generally normal.

DX: clinical , good history include sequence of event , lack of incontinence and no post ictal
phase. Treatment : reassurance and iron.

524) Infant brought by the mother that noticed that the baby has decreasing feeding ,
activity and lethargic On examination febrile(39), tachycardic ,his bp 75/30, with
skin rash . DX:

a.Septic shock

Q not complete

525) Old patient newly diagnosed with hyperthyroidism presented with


(hyperthrodism symptoms) The best initial symptomatic treatment is:

a. BB ( beta-blocker )

b. PTU

The correct answer is a

_______________________________________________________________________

526) Infant presented with hemangioma on the back . your management is:

a. Intralesional injection of corticosteroids

b. Topical corticosteroids
c. Excise of the lesion

527) Pregnant lady , 34 wk GA , presented with vaginal bleeding more than her
menstruation. On examination , cervix is dilated 3 cm with bulging of the
membrane, fetal heart rate = 170 bpm . The fetus lies transverse with back facing
down . us done and shows that placenta is attached to posterior fundus and
sonotranulence behind placenta. Your management is :

a. C/S
b. Oxytocin
c.Tocolytics
d.Amniotomy
the correct answer is a

528) Infant with congenital hip dislocation:

a. +ve click in flexion ,abduction

b. The only treatment is surgery

c. Not reduced with flexion and abduction of the hip

the correct answer is a . this is ortolani test

529) In irritable bowel S. the following mechanism ?contraction and slow wave
myoelectricity seen in:

a.Constipation

b. Diarrhea

the correct answer is a

529) Which is not found in coarctation of the aorta:

a. Upper limb hypertension

b. Diastolyic murmur heard all over precordium

c. Skeletal deformity on chest x-ray


the correct answer is a (I'm not sure )

530) Female patient presented with tender red swelling in the axilla with history of
repeated black head and large pore skin in same area: ttt is

a. Immidate surgery

b. Topical antibiotic

c. Cold compressor

d. Oral antibiotic

531) In indirect hernia the relation of the sac to the cord structure is:

a. Anteromedial

b. Anterolateral

c. Posteromedial

d.Postrolateral

the correct answer is b

532) The most common cause of croup is:

a. Parainfluenza

b. Influenza

the correct answer is a

533) kwashikor disease usually associated with :

a. decrease protein intake, decrease carbohydrate

b. increase protein , increase carbo

c. decrease protein , increase carbo


the correct answer is c. in the book, written there is decrease protein and
adequate amount of carbohydrate, but with this only MCQs the answer is c

534) in cachectic patient, the body utilize the proteins of the muscles :

a. to provide Amino acid and protein synthesis

b. to maintain bloodflow to vital organ

c. to increase body fat

the correct anwser is a ( I'm not sure )

535) patient is complaining of memory loss. Alzehimer disease is diagnosed what is the
cause of this:

a. brain death cell

Q not complete

536) Parents brought their baby to you who is on bottle feeding. On exam whitish
lesion on either side of teeth seen with blackish lesion on maxillary incisors and
second molar teeth. There is history of leaving the baby with bottle in his mouth
during sleeping. The Dx:

a. Nursery dental caries

b.Gingvostomatis

the correct answer is a

537) Which of the following medication if taken need to take the patient immidiatly to
the hospital:

a. Penicillin

b. diphenhydramine

c. OCPs

d. Quinine or Quinidine

the correct answer is d (I'm not sure ) , quinidine is antiarrhythmia .

538) 43 y/o female presented with severe DUB other examination normal . your
management is
a. D &C

b. Ocps

c. Hysterectomy

d. Blood transfusion

the correct answer is a

539) Baby with vesicles on the face and honey comb crust which of the following
organism cause it: Staph aureus

540) Female patient presented with migraine headache which is pulsatile, unilateral ,
increase with activity . Dosn't want to take medication. Which of the following is
appropriate:

a. Bio feedback

b. TCA

c. BB

the correct answer is a

Biofeedback has been shown to help some people with migraines. Biofeedback is a
technique that can give people better control over body function indicators such as
blood pressure, heart rate, temperature, muscle tension, and brain waves. The two
most common types of biofeedback for migraines are thermal biofeedback and
electromyographic biofeedback.

541) Infant born with hemangioma on the rt eyelid what is appropriate time to
operate to prevent amylopia:

a. 1 day

b. 1 week
c. 3 months

d. 9 months

difficult Q. but may be the correct answer is b or c

542 ) Young patient on anti TB medication presented with vertigo which of the
following drug cause this:

a. Streptomycin

b. Ethambutol

c. Rifampcin
the correct answer is a , streptomycin cause 8th nerve damage.

543) The CPR for child is

a. 30 chest compression 2 ventilation … (Lone rescuer)

b. 15 chest c 2 ventilation … (2 or more rescures)

c. m15 chest compression 1 ventilation

the correct answer is a

544) Picture show large ulcer over medial side of the leg . what is your management

a. Shave biobsy

b. Elevate the legs and stocking

c. Topical steroids

the correct answer is b

……………………………………………………………………………………………………………………………………………………..
545) 2months infant with white plaque on tongue and greasy ,past h/o clamydia
conjunctivitis after birth treated by clinamycin what is ttt:
a. Oral nystatin
b. Topical steroids
c. Topical acyclovair
d. Oral tetracycline
the correct answer is a. oral nystatin : antifungal

……………………………………………………………………………………………………………………………………………………..
546) child rt ear pain and tenderness on pulling ear , no fever , O/E inflamed
odemateous rt ear canal with yellow discharge >>>>>>>>>>dx:
a. Otitis media
b. Otitis externa
c. Cholesteatoma

the correct answer is a

……………………………………………………………………………………………………………………………………………………..
547) 34y female with HIV pap smear negative, about cervical cancer screening :

a. After 3m if negative repeat after 6m


b. After 6m …………………………… annually
c. After 1y………………………………….annually

The correct answer is b ( I'm not sure , confuse with choice c )


548) female about 30y with breast cancer (given cbc –chem. And reavel low hb and
hematocrite….) what is the next step in mangment:
a. Staging
b. Lumpectomy
c. Mastectomy
d. chemotherapy

the correct answer is a

……………………………………………………………………………………………………………………………………………………..
549) child with moderate persistant BA On bronch.dilat inhaler. Presented with acute
exacerbation what will you add in ttt:
a. Corticosteroid inhaler
b. Ipratropum bromide inhaler

the correct answer is b

……………………………………………………………………………………………………………………………………………………..

550) about head and neck injury :


a. Hoarsness of voice and stridor can occure with mid facial injury
b. Tracheostomies contraindicated
c. Facial injury may cause upper air way injures

……………………………………………………………………………………………………………………………………………………..
551) lactating women 10 days after delivary developed fever ,malaise, chills tender Lt
breast with hotness and small nodule in upper outer quadrant with axillary LN
.Leucocytic count was 14 *10/L dx:
a. Inflammatory breast cancer
b. Breast abscess
c. Fibrocystic disease

the correct answer is b

552) 70y male with osteoporosis the T score of bone densometry would be :
a. -3.5
b. -2.5
c. 1
d. 2
e. 3.5

the correct answer is b

……………………………………………………………………………………………………………………………………………………..

553) child with epistaxis…….. management:


a. Compression on nose and leaning forward
b. ………………………………………………….backward

the correct answer is a

……………………………………………………………………………………………………………………………………………………..
554) scenario about female underwent abdominal operation she went to physician
For check……….. U/S reveal metal thing inside abdomen (a.e missed during operation)
What will you do :
a. Call the surgeon and ask him what to do
b. Call attorney and ask about legal action
c. Tell her what you found
d. Tell her that is one of possible complications of operation
e. Don't tell her what you found

The correct answer is c

……………………………………………………………………………………………………………………………………………………..
555) male ptn with scaly fine papular rash on fornt of scalp,nose and
retroauricular……..(i think tinea capitis) ttt is:
a. Ketoconazole cream…
b. Oral augmentin
c. ……… cream

the correct answer is a

556) All can cause gastric ulcer except:


a- Tricyclic antidepressant.
b- Delay gastric emptying.
c- Sepsis.
d- Salicylates.
e- Gastric outlet incompetent.

The correct answer is a


Tricyclic antidepressant use in treatment of peptic ulcer.
……………………………………………………………………………………………………………………………………………………..

557) 48year old female lost her menstruation for 2 cycles, the method of
contraception is condom, examination was normal except for dusky
discoloration of the cervix. What u will do next:
a.Progesterone challenge.
b.Beta HCG.
c.Pelvic u/s

The correct answer is b

……………………………………………………………………………………………………………………………………………………..

558) ttt of yeast vaginitis: Clotrimazole , Fluconazole , Ketokonazole

Treatment of yeast vaginitis ( candida albicans ) : topical azole or PO fluconazole ,


oral azole should be avoided in pregnant.

……………………………………………………………………………………………………………………………………………………..

559) 15 y/o female complaining of pain during menstruation, not sexually active,
medical hx unremarkable, physical examination normal, how to treat:
a- NSAID.
b- Danazole.
c-

The correct answer is a


……………………………………………………………………………………………………………………………………………………..

560) What is true about alpha blocker:


a. Causes hypertension.
b. Worsen benign prostatic hyperplasia.
c. Cause tachycardia.
d-

the correct answer is c


alpha blocker :cause orthostatic hypotension and tachycardia.

561) cyclic menstruation that increase in frequency:


a. Polymenorrhea.
b. Hypermenorrhea.
c. Menorrhagia.
d. Dysmenorrhea.
e.

The correct answer is a


Polymenorrhea : frequent menestration ( < 21 day cycle )
Hypermenorrhea or menorrhagia : abnormally heavy and prolonged menstrual
period at regular intervals . more than 80 ml of blood loss per cycle or prolonged
bleeding , more than 8 days.
Oligomenorrhea : increase length of time between menses ( 35-90 days b\w cycle)
Metrorrhagia : bleeding bwteen period.
Menometrorrhagia : exessive and irreglar bleeding.

……………………………………………………………………………………………………………………………………………………..

562) A pregnant lady came to you to screen her fetus for down syndrome, what is the
best method:
a. Amniocentesis. + Karyotyping
b. Choriocentesis.
c.

the correct answer is a

……………………………………………………………………………………………………………………………………………………..

563) What is the most common chromosomal abnormality?


a. Trisomy 13
b. Trisomy 21

the correct answer is b


-down syndrome ( trisomy 21 ) is the most common chromosomal abnormality.

……………………………………………………………………………………………………………………………………………………..
564) Best 2 test to screen for hepatocellular carcinoma:
a. Liver biopsy and alpha-fetoprotein.
b. Liver ultrasound and alpha-fetoprotein.
c. Abdomen CT and
d.

The correct answer is b

565) A child came to ER with fever, stridor, … , x-ray showed swollen epiglottis, in
addition to oxygen, what u will do?
a. Throat examination.
b. An emergency tracheostomy.
c. Endotracheal intubation.
d. Nasopharyngeal intubation.

The correct answer is c


……………………………………………………………………………………………………………………………………………………..

566) 24 y/o female newly diagnosed type 2 DM, she is wearing glasses for 10 years,
how frequent she should follow with ophthalmologist:
a. Every 5 years.
b. Annually

The correct answer is b


-for type 1 diabetic : retina screening annually beginning 5 years after onset of
diabetes, general not before onset of puberty.
- for type 2 diabetic : screening at the time of diagnosis then annual
……………………………………………………………………………………………………………………………………………………..

567) What is the initial management for a patient newly diagnosed knee
osteoarthritis.
a. Intra-articular corticosteroid.
b. Reduce weight.
c. Exercise.
d. Strengthening of quadriceps muscle.

The correct answer is b


……………………………………………………………………………………………………………………………………………………..

568) A lady came to your clinic said that she doesn’t want to do mammogram and
preferred to do breast self- examination, what is your response?
a- Mammogram will detect deep tumor.
b- Self-examination and mammogram are complementary.
c- Self-examination is best to detect early tumor
answer is b ( I'm not sure because MCQs not complete and depend on age of pt.

……………………………………………………………………………………………………………………………………………………..

569) What is the best frequency for breast self-examination?


a. Daily.
b. Weakly.
c. Monthly.
d. Annually.

The correct answer is c


570) Patient with left bundle branch block will go for dental procedure , regarding
endocarditis prophylaxis:
a. No need
b. Before procedure.
c. After the procedure.
d.

The correct answer is a


……………………………………………………………………………………………………………………………………………………..

571) classical characteristic for genital herpes.


Painful ulcers & vesicles

……………………………………………………………………………………………………………………………………………………..

572) most common vaginal bleeding :


a. cervical polyps
b. menstruation
c. ????

only with these MCQs , the correct answer is b


……………………………………………………………………………………………………………………………………………………..

573) best stimulant for breast milk secretion:

a. breast feeding
b. oxytocin
c. ???

the correct answer is a


……………………………………………………………………………………………………………………………………………………..

574) child with aspirin intake overdose ...what kind of acid base balance:
a. metabolic alkalosis wt respiratory
b. metabolic acidosis wt respiratory alkalosis
c. respiratory alkalosis with metabolic acidosid
d. respiratory acidosis with metabolic alkalosis

The correct answer is c


Aspirin toxicity : in early stages, salicylate will stimulate respiratory center 
RR  respiratory alkalosis that will be compensated by metabolic acidosis. In
late stage, it will interfere with COH, fat, & protein metabolism as well as
Oxidation phosphorylation lead to  lactate, pyrovate, & keton bodies. All will lead
to pH. Signe & symptoms includes: nausea, vomiting, RR, temp, HR,
sweating, cerebral or pulmonary edema, & coma. +ve anion gap. TTT: hydration,
correct K+, gastric lavage or activated charcoal, urine alkalization, hemodialysis)

575) therapeutic range of INR :


a. 2.5-3.5
b. 2.0-3.0
c. ???

The correct answer is b

……………………………………………………………………………………………………………………………………………………..

576) q about antidepressant:


a. start single type even patient have sever depression
b. start any one of them they all have the same efficacy
c. stop the medication after 2 weeks if no improvement

The correct answer is a

……………………………………………………………………………………………………………………………………………………..

577) patient with depression started on amitryptaline , he had headache or dizzness ,


vomitting?? im not sure what exactly was the symptoms
a. change to SSRI
b. ??

Q not complete , but amitriptyline is TCA .

……………………………………………………………………………………………………………………………………………………..

578) patient had horsenss of voice for 3 weeks... next to do:


a. throat swab
b. laryngoscopy
c. ???

The correct answer is b , I'm not sure because MCQs are not complete.

……………………………………………………………………………………………………………………………………………………..
579) single diagnostic for stroke:
a. high cholesterol
b. high systolic blood pressur

the correct answer is b , I'm not sure , but with these mcqs , b is correct.

580) unfaivrable prognosis for schezophrenia:

a. family Hx

b. failed marrige

c. adolscen age?? not sure

d. presence of psychosis

the correct answer is a


……………………………………………………………………………………………………………………………………………………..

581) Rt lung :
a. have 1 fissure
b. contain 7 segment
c. ??? read about lung anatomy segment names!!

Rt lung has 3 lobes & 10 segments (medial basal segment only in inf. Lobe of Rt lung)
Lft lung has 2 lobes & 9 segments (Lingular segments in lft lung)

……………………………………………………………………………………………………………………………………………………..

582) q about specifity:


a. test is +ve in disese population
b. test is +ve in healthy poplation
c. test is -ve in diseased population
d. test is -ve in healthy population

The correct answer is d


……………………………………………………………………………………………………………………………………………………..

583) stage 3 colon ca  Give Chemo ASAP


……………………………………………………………………………………………………………………………………………………..

584) patient with upper abdominal pain, nausea vomitting,with back pain, he is
smoker for long time daily, fecal fat was +ve
a. acute pancreatitis
b. chronic pancreatitis
c. pancreatic CA

The correct answer is b

585) in school, 10 of patient had the dis of the first week , then 30 pt. had the dieses
the next week , the percentage infection of this school is:
a. 10%
b. 20%
c. 30%
d. 40%

Q is not clear or not complete, but I think the correct answer is b.


……………………………………………………………………………………………………………………………………………………..

586) patient had headche describe it as aband around his head, increase with stress
and , i dont remeber the ques DX is:
a. tension headache
b. migraine
c. cluster headache

The correct answer is b

……………………………………………………………………………………………………………………………………………………..
587) Patient was presented by bollus in his foot , biopsy showed sub dermal lysis ,
fluorescent stain showed IgG , what is the most likely diagnosis :
A. Bolus epidermolysis .
B. Pemphigoid vulgaris .
C. Herpetic multiform .
D. Bullous pemphigoid .

The correct answer is d

……………………………………………………………………………………………………………………………………………………..

588) Patient was presented by difficulties of breathing from one side of his nose , on
examination there was erythramatus swelling , what is the best initial treatment :
A. Decongestant .
B. Steroid .
C. Sympathomimetics . ??
The correct answer is c ( I'm not sure )

Explain: this swelling might be nasopharyngeal angiofibroma which is highly vascular


swelling & prone to epistaxis, so to decrease the size we might use local or systemic
decongestants which are sympathomimetics, so answer is not clear.

589) Patient with cystic nodule (acne) and scars , what is the best treatment :
A. Retinoin .
B. Erythromycin .
C. Doxycyclin .

The correct answer is a ( I'm not sure )

this seems to be an advanced inflammatory stage of acne since there r scars so if


the retinoiD meant here is systemic (isotretinin) so it will be the choice. If topical it
will not be effective as it is effective only in the non-inflammatory stage
(comedones). Both erythromycin (local) & doxycyclin (systemic) can be used but in
advanced stage systemic is better.

……………………………………………………………………………………………………………………………………………………..

590) Patient was presented by constipation, vomiting , abdominal distension , with old
scar in the lower abdomen , x ray showed dilated loops with air in the rectum ,
what is the best initial management :
A. NGT decompression , and IV line .
B. Rectal decompression and antibiotics .
C. Suppositories .

The correct answer is a

this is a case of intestinal obstruction due to adhesions from previous abdominal


surgery (commonest cause) so the initial treatment would be decompression of the
intestine & replace lost fluids

……………………………………………………………………………………………………………………………………………………..

591) Which of the following is not living vaccine :


A. Hepatitis B .
B. MMR .
C. Oral polio
D. BCG

The correct answer is a


Hepatitis B is the only vaccine prepared by genetic engineering so its not living vaccine.

592) Six years old child was born to a mother with hepatitis B , he does not received
any vaccines before , what you will give now :
A. DTP , MMR , Hib
B. DTP , MMR .
C. DT , MMR , Hib
D. DT, MMR .

The correct answer is d

both Pertussis & H.influenza vaccine are not needed after 1 year of age.

……………………………………………………………………………………………………………………………………………………..

593) Which of the following describes the end of the early inflammatory phase .
A. Formation of eschar .
B. Formation of ground base of collagen .
C. The end of angiogenesis .

The correct answer is c

……………………………………………………………………………………………………………………………………………………..

594) Female patient , known case of VitD deficiency , smoking , and recurrent fall,
which of the following is the greatest exogenous risk for osteoporosis :
A. Advanced age .
B. Recurrent fall .
C. Vit D .
D. Smoking .

The correct answer is d

Advanced age & female gender are the most important factors for osteoporosis but
they are endogenous. Smoking is an independent exogenous risk factor for
osteoporosis
595) Blood sugar in DM type 1 is best controlled by :
A. Short acting insulin .
B. Long acting .
C. Intermediate .
D. Hypoglycemic agents .
E. Basal and bolus insulin .

The correct answer is e

Very vague question. We can exclude hypoglycemic agents. Short acting insulin is
best in emergencies like DKA as it can be given IV. We can use either long acting
alone daily or a mixture of short & intermediate acting insulin daily. Basal & bolus ,
( short acting + intermediate or long ), bolus’ of short-acting or very-short-acting
insulin before meals to deal with the associated rise in blood-sugar levels at these
times. In addition, they take an evening injection of long- or intermediate-acting
insulin that helps normalise their basal (fasting) glucose levels. This offers greater
flexibility and is the most commonly adopted method when intensified insulin
therapy is used to provide optimal glycaemic control.

……………………………………………………………………………………………………………………………………………………..

596) Which of the following is true regarding perths disease :


A. Commonly seen between 11-16 years of age .
B. Always unilateral .
C. May present by painless limp .
D. Characteristically affect the external rotation of hip .
E. More in female .

The correct answer is c

Perthes disease (Leg-Calve perthes disease) is a condition affecting the hip joint
where there is degenerative avascular necrosis of the femoral head. It affects
children aged 3-12 years & is more common in males. It is most commonly but not
always unilateral ( 85% is unilateral ). It presents mainly by severe hip pain &
limping that increases by movement but it can present by painless limp. It
characteristically affects the internal rotation & abduction of the hip & limits these
movements.
597) Which of the following is true regarding gastric lavage :
A. Patient should be in the right lateral position .
B. It is not effective after 8 hours of aspirin ingestion .

The correct answer is b

lavage is effective only 1 hour after ingestion of any poison. After that its ineffective

……………………………………………………………………………………………………………………………………………………..

598) SSRI was prescribed to a patient with depression , the effect is suspected to be
within :
A. One day .
B. Two weeks .
C. Three to four weeks .

The correct answer is c


Allow 2 – 6 weeks to take effect , and treat for > 6 months

……………………………………………………………………………………………………………………………………………………..

599) Randomized controlled study will be stronger by :


A. Systemic >>>>
B. Following at least 50 % of the participant .

???????

……………………………………………………………………………………………………………………………………………………..

600) Partner lost his wife by AMI 6 months ago , presented by loss of appetite , low
mood , sense of guilt , what is the diagnosis :
A. Beverament .
B. Major depression episode .

The correct answer is a

- Major depression is a psychiatric condition that occurs regardless of events that


happen in life, while normally most people would have beverament after death of a
close person.

601) An adult was presented by sore throat , congestion , fatigue , petechia in soft
palate , tender spleen , and liver , what is the most likely diagnosis :
A. EBV .
Only 1 choice?

This could be Rubella infection as petechia in soft palate (Forschheimer spots)


occur in rubella also. But liver & spleen affection are more common in EBV. Also
since it is an adult then we might suspect leukemia as the cause.

……………………………………………………………………………………………………………………………………………………..

602) Patient was presented by blepharitis , acne roseca , but no keratitis , what is the
best treatment :
A. Topical chlorophenicol .
B. Topical gentamicin .
C. Oral doxycyclin .

The correct answer is c

……………………………………………………………………………………………………………………………………………………..

603) Diabetic patient was presented by spastic tongue , dysarthria , spontenous crying
what is the most likely diagnosis :
A. Parkinson .
B. Bulbar palsy .
C. Pseudobulbar .
D. Myasthenia gravis .

The correct answer is c

This is a bit tricky. Bulbar palsy is the LMNL of the last 4 CN, while pseudobulbar
palsy is the UMNL of the last 4 CN. So spasticity of tongue is UMNL. But Diabetes
causes LMNL of cranial nerves due to peripheral neuropathy. So maybe the cuase
here is CNS affection due to atherosclerosis from macroangiopathy of diabetes.

604) Which of the following is describe the normal developmental stage for 6 months
old child :
A. Sits without support .
B. Rolls front to back . (7-8 months)
C. No head lag . (3 months)
D. Stand alone . (1 year)
The correct answer is a

……………………………………………………………………………………………………………………………………………………..

605) Which of the following personality is characterized by inflexibility ,


perfectionism ?
A. OCD .
B. Not otherwise specified .
C. Narcissistic .

The correct answer is b

Obsessive compulsive personality disorder is different from obsessive compulsive


disorder and is characterized mainly by perfectionism while OCD is characterized by
repetitive actions due to compulsion. if OCPD is present in the choices it would be
the answer.

……………………………………………………………………………………………………………………………………………………..

606) What is the commonest cause of otorrhea :


A. Acute otitis media .
B. Basal skull fracture .
C. External otitis .

The correct answer is a

……………………………………………………………………………………………………………………………………………………..

607) Patient was presented by ear pain , red tympanic membrane , apparent vessels ,
with limited mobility of the tympanic membrane , what the most likely diagnosis :
A. Acute otitis media .
B. Tympanic cellulitis .
C. Mastoditis .

The correct answer is a

608) What is the management of acute congestive glaucoma :


A. IV acetazolamide and topical pilocarpine . (true)

……………………………………………………………………………………………………………………………………………………..

609) Eight years old child with late systolic murmur best heard over the sterna border ,
high pitch , crescendo , decrescendo ,>> diagnosis is :
A. Physiological murmur .
B. Innocent murmur .
C. Ejection systolic murmur .
D. Systolic regurgitation murmur .

The correct answer is c ( I'm not sure )

……………………………………………………………………………………………………………………………………………………..

610) Child was presented by coryza , skin rash , conjunctivitis , and multiple spots in the
mouth :
A. Measles .
B. Rubella .

The correct answer is a.


Measles : due to conjunctivitis which is not present in rubella

……………………………………………………………………………………………………………………………………………………..

611) Which of the following could be seen in patient with bulimia :


A. Hypokalmeia .
B. Metabolic acidosis .

The correct answer is a

bulimia is aka bing eating which means the patient eats a lot then does forced
vomiting so there is loss of acids & electrolytes which leads to hypokalemia &
metabolic alkalosis.

612) Maximum spinal height is reached after menarche by how many years ?
A. Months .
B. Two years .
C. Three years .

The correct answer is c


menarche is at 12-14 years and maximum height is about 17 years

……………………………………………………………………………………………………………………………………………………..

613) Pregnant lady 34 weeks of gestation presented by vaginal bleeding , which of the
following is relevant to ask about :
A. Smoking .
B. Desire of future pregnancy .
C. The result of last pap smear .
D. Hx of vaginal irritation .

The correct answer is a , because smoking is an important risk factor for antepartum
he

……………………………………………………………………………………………………………………………………………………..

614) Female patient with marginal placenta previa , with 6 cm dilated cervix , what is
your management :
A. Fetal monitoring .
B. Delivery if fully dilated .
C. Tocolytics >

The correct answer is a ( I'm not sure )

I couldn’t find the answer correctly so based upon the given data, fetal monitoring
is a must in any case of high risk pregnancy. Also as the PP is marginal, there is
possibility of delivery without severe PPH and the cervix is already dilated.

615) Which of the following drugs increase the survival in a patient with heart failure :
A. Beta blocker .
B. ACE inhibitors .
C. Digoxin .
D. Nitrites .

The correct answer is b


New updated information. As ACE inhibitors inhibit aldosterone which if present in
high concentrations causes modification of the cardiac myocytes in the long term.

……………………………………………………………………………………………………………………………………………………..

616) Elderly patient presented by SOB , rales in auscultation , high JVP , +2 lower limb
edema , what is the main pathophysiology :
A. Left ventricular dilatation .
B. Right ventricular dilatation .
C. Aortic regurgitation.
D. Tricuscpid regurgitation .

The correct answer is a ( cofuuuuuuuse )

Difficult question. Here we have both symptoms of Lft ventricular failure (SOB,
Rales) & Rt ventricular failure (High JVP & LL edema). So, more commonly lft
ventricular failure leads to rt ventricular failure due to overload and not vice versa .
so the most correct is Lft ventricular dilatation.

……………………………………………………………………………………………………………………………………………………..

617) Patient with high output fistula , for which TPN was ordered , after 2 hours of the
central venous catheterization , the patient become comatose and unresponsive ,
what is the most likely cause :
A. Septic shock .
B. Electrolytes imbalance .
C. Delayed response of blood mismatch .
D. Hypoglycemia .
E. Hypernatremia .

The correct answer is a

618) Female patient presented by tender mass below the axilla , with previous history
of pores and black heads in that’s area <<< what is your management :
A. Go to surgery immediately .
B. Oral antibiotics .
C. Topical antibiotics .
D. Steroid .

The correct answer is a , This is mostly an infected abscess.

……………………………………………………………………………………………………………………………………………………..

619) Child was presented by erythema , and swelling in his hand after 18 hours of bee
sting , what is your management in this case :
A. Epinephrine .
B. Antihistaminic drugs .
C. Hospitalization .

The correct answer is b


18 hours is a long duration so its unlikely to be anaphylactic shock & more likely a
regular reaction to bee stings so antihistaminics is the best solution

……………………………………………………………………………………………………………………………………………………..

620) Which of the following is true regarding metformin :


A. Main complication is hypoglycemia .
B. Can lead to weight gain .
C. It suppress the hepatic glauconeogenesis .

The correct answer is c

……………………………………………………………………………………………………………………………………………………..

621) Child was presented by congested throat , coryza , high grade fever , which of the
following is true regarding this condition :
A. Viral > bacterial .
B. Bacterial > viral .
C. Antibiotics should be given any way .
D. It is most likely due to EBV .

The correct answer is b ??????

622) Benign tumors of stomach represent almost :


A. 7 %
B. 21 %
C. 50 %
D. 90 %

The correct answer is a

……………………………………………………………………………………………………………………………………………………..

623) 60 years old patient presented by recurrent venous thrombosis including superior
venous thrombosis , this patient most likely has :
A. SLE .
B. Nephrotic syndrome .
C. Blood group O .
D. Antiphospholipid syndrome .

The correct answer is d

……………………………………………………………………………………………………………………………………………………..

624) Elderly male patient was presented by left lower abdomen tenderness , fever ,
elevated WBC count , what is the most likely diagnosis :
A. Inflammatory bowel .
B. Diverticulitis .

The correct answer is b

……………………………………………………………………………………………………………………………………………………..

625) 22 years old male patient was presented by recurrent attacks of diarrhea ,
constipation , and abdominal pain relieved after defecation , but no blood in the
stool , no weight loss : what is the diagnosis :
A. Irritable bowel Syndrome

626) Patient was presented by back pain relieved by ambulation , what is the best
initial treatment :
A. Steroid injection in the back .
B. Back bracing .
C. Physical therapy .

The correct answer is c

……………………………………………………………………………………………………………………………………………………..

627) Patient with long history of constipation presented by painful defecation ,


followed by bleeding , pain persist for hours after defecation what is the diagnosis
:
A. Anal fissure .
B. Thrombosed external piles .
C. Internal hemorrhoid .

The correct answer is a

……………………………………………………………………………………………………………………………………………………..

628) Elderly male patient underwent colectomy for colon cancer in which
micrometastais was detected in the lymph nodes , what is the best explaniation :
A. Good prognosis .
B. Liver metastasis .
C. It is sensitive to chemotherapy . (Dukes class C cancer best for chemotherapy)
D. It is locally advanced .

The correct answer is c


……………………………………………………………………………………………………………………………………………………..

629) IV drug abuser was presented by fever , arthralgia , conjunctival hemorrhage ,


what is the diagnosis :
A. Bacterial endocarditis .

……………………………………………………………………………………………………………………………………………………..
630) Female patient was presented by dysurea , epithelial cells were seen urine
analysis , what is the explanation in this case :
A. Contamination .
B. Infection .

The correct answer is b

631) Patient was presented by tremor , fever , palpitation , diagnosed as case of


hyperthyroidism , what is your initial treatment :
A. Surgery .
B. Radio iodine .
C. Beta blockers . Propylthioracil .

The correct answer is c , firstly B-blocker then Prophylthiouracil because we are


afraid of arrhythmias

……………………………………………………………………………………………………………………………………………………..

632) Young female patient presented by pain during the first 2 days of menses ,
menarche was 2 years back , what is your initial treatment :
A. OCP .
B. NSAID .

The correct answer is b , NSAID it is the best initial treatment for dysmenorrheal

……………………………………………………………………………………………………………………………………………………..

633) Female is planning to get pregnant , she want to get MMR vaccine also , what is
your action :
A. Delay the pregnancy 2-3 months after vaccination .
B. It is safe during pregnancy .

The correct answer is a

……………………………………………………………………………………………………………………………………………………..
634) Which of the following is true regarding varicella vaccine during breast feeding :
A. It is safe .
B. No breast feeding except after 3 days of the immunization .
Couldn’t find the answer

……………………………………………………………………………………………………………………………………………………..
635) Patient underwent abdominal surgery due to intestinal perforation many years
back , presented by abdominal pain , distension , constipation , what is the best
investigation in this case :
A. Barium enema .
B. Ultrasound .
C. Small bowel barium study .

The correct answer is c


636) True negative test is best described as following :
A. Not suspected to have the disease who actually do not have .

……………………………………………………………………………………………………………………………………………………..

637) Patient with AMI and multiple PVC , is your treatment for this arrhythmia :
A. Amiadrone .
B. No treatment .

……………………………………………………………………………………………………………………………………………………..

638) Adenosine dose should be reduced in which of the following cases :


A. Chronic renal failure .
B. Patients on thiophyline .

The correct answer is a

……………………………………………………………………………………………………………………………………………………..

639) Which the following is the commonest complication of patient with chronic atrial
fibrillation :
A. Sudden death .
B. Cerebra vascular accidents .

The correct answer is b , due to multiple atrial thrombi

……………………………………………………………………………………………………………………………………………………..

640) Which of the following is most likely seen in case of active glomerlonephritis :
A. RBC casts .

……………………………………………………………………………………………………………………………………………………..
641) Target HbA1c is less than :
A. 6.5
B. 8
C. 9

The correct answer is a


642) Female patient with candida most likely has :
A. DM .
B. SLE .

The correct answer is a

……………………………………………………………………………………………………………………………………………………..

643) Which of the following indicates good prognosis in schizophrenia :


A. Family history of schizophrenia .
B. Gradual onset .
C. Flat mood .
D. Prominent affective symptoms .
E. No precipitating factors .

The correct answer is d

……………………………………………………………………………………………………………………………………………………..

644) Pregnant woman with suspected DVT , what is the best initial investigations :
A. Duplex US .
B. D dimer .
C. Platysomgraphy .
D. Venogram .

The correct answer is a

……………………………………………………………………………………………………………………………………………………..

645) Which of the following is true regarding crohns disease :


A. Partial thickness involvement .
B. Fistula formation .
C. Continuous area of inflammation .
D. Mainly involve the recto sigmoid area .

The correct answer is b


646) Patient serology showed antibodies to the surface antigen for hepatitis B , what is
your diagnosis :
A. Previous infection or immunization .
B. Chronic carrier .
C. Highly infective .

The correcr answer is a

……………………………………………………………………………………………………………………………………………………..

647) Young male patient presented by acute scrotal pain , US showed reduced blood
flow , what is the diagnosis :
A. Testicular torsion .
B. Trauma .
C. Infection .
D. Hernia .

The correct answer is a

……………………………………………………………………………………………………………………………………………………..

648) Female patient is sure that she is pregnant for 2 months , on examination , the
uterus is larger than suspected , B-hcg is very high , the doctor diagnosed her as
having tumor which is chemo sensitive , what is the diagnosis :
A. Ovarian cancer
B. Endometrial cancer .
C. Gestational trophoblastic .

The correct answer is c

……………………………………………………………………………………………………………………………………………………..

649) Which of the following is true regarding infertility :


A. It is Failure to conceive withing 6 months . (1 year)
B. Male factor > female factors . (the reverse)
C. It could be due to high prolactin levels .
D. Rare to be due anovulotion . (common)
E. Only diagnosed by HSG . (need full lab & imaging investigations)

The correct answer is c

650) 32 years old female patient presented by irregular menses , menses occurs every
two months , on examination every thing is normal , which of the following is the
LEAST important test to ask about first :
A. CBC .
B. Pelvic US .
C. Coagulation profile .
D. DHES .

The correct answer is c

……………………………………………………………………………………………………………………………………………………..

651) Patient with early rheumatoid arthritis , what is your management to decrease
the limitation of movement :

- Do not use analgesics or steroids, use DMARDs like methotrexate or antiTNF,


hydroxychloroquine

……………………………………………………………………………………………………………………………………………………..

652) Patient with truncal obesity , easy bruising , hypertension , buffalo hump , what is
the diagnosis :
A. Cushing .

……………………………………………………………………………………………………………………………………………………..

653) Patient is known case of cervical spondylolysis , presented by parasthesis of the


little finger , with atrophy of the hypothenar muscles , EMG showed cubital tunnel
compression of the ulnar nerve , what is your action now :
A. Ulnar nerve decompression .
B. Steroid injection .
C. CT scan of the spine .

The correct answer is a

654) Well known case of SCD presented by plueritic chest pain , fever , tachypnea ,
respiratory rate was 30 , oxygen saturation is 90 % what is the diagnosis :
A. Acute chest syndrome .
B. Pericarditis :
C. VOC .

The correct answer is a , or pneumonia would be more correct if it was the answer

……………………………………………………………………………………………………………………………………………………..
655) Child with hemangioma around the eye , operation should be done within ……… :
A. 1 Week
B. 3 months .
C. 6 months .

The correct answer is a ( I'm not sure )

We only fear of reduction in the function of the eye due to hemangioma so it


should be removed.

……………………………………………………………………………………………………………………………………………………..

656) Which of the following is the recommended diet to prevent IHD :


A. Decrease the intake of meat and dairy .
B. Decrease the meat and bread .
C. Increase the intake of fruit and vegetables .

????

The correct answer is c

……………………………………………………………………………………………………………………………………………………..

657) Which of the following can lead to polyhydrmnios :


A. Duodenal atresia .
B. Renal agenesis .  Oligohydramnios
C. Post term pregnancy . Oligohydramnios
D. Diabetes inspidious .

The correct answer is a

658) Adult polycystic kidney disease is inherited as:


A. Autosomal dominant .
B. Autosomal recessive .
C. X linked .

The correct answer is a

……………………………………………………………………………………………………………………………………………………..

659) The best treatment for bacteroid :


A. Clindamycin .

……………………………………………………………………………………………………………………………………………………..
660) You have an appointment with your patient at 10 am who is newly diagnosed
DM , you came late at 11 am because you have another complicated patient ,
what are you going to say to control his anger :
A. Do not say any thing .
B. Told him that there is another patient who really need your help .

Q not complete, but may be The correct answer is b

……………………………………………………………………………………………………………………………………………………..

661) Well known case of DM was presented to the ER with drowsiness , in the
investigations : Blood sugar = 400 mg/dl , pH = 7.05 , what is your management ?
A. 10 units insulin + 400 cc of dextrose .
B. 0.1 unit/kg of insulin , subcutaneous .
C. NaHCO .
D. One liter of normal saline .

The correct answer is d

……………………………………………………………………………………………………………………………………………………..

662) What is the best method for history taking :


A. Yes or no questions .
B. Open ended .
C. Silent listening .

The correct answer is b

663) What you will give to prevent hemorrhagic disease of newborns :


A. Vitamin K .

……………………………………………………………………………………………………………………………………………………..
664) Arterial injury is characterized by :
A. Dark in color and steady .
B. Dark in color and spurting .
C. Bright red and steady .
D. Bright red and spurting .

The correct answer is d

……………………………………………………………………………………………………………………………………………………..
665) 14 years old girl complaining of painless vaginal bleeding for 2-4 days every
3Weeks to 2 months ranging from spotting to 2 packs per day; she had 2ry sexual
ccc 1 year ago and had her menstruation since 6 months on clinical examination
she is normal sexual ccc, normal pelvic exam appropriate action
a. OCP can be used

b. You should ask for FSH and prolactin level

c. Don’t do anything & explain this is normal

the correct answer is c

……………………………………………………………………………………………………………………………………………………..
666) 4 years old child what can he do

a. Copy square and triangle

b. Speak in sentences

the correct answer is a , if copy square 4 years , but if copu circle 3 years

……………………………………………………………………………………………………………………………………………………..
667) baby can sit without support, walk by holding fourniture, Pincer grasp, pull to
stand how old is he

a. 8 months

b. 10 months

c. . 12 month

d. 18 month

The correct answer is b

668) Patient came after deep laceration at the anterior part of the wrist:
a. Wrist drop
b. Sensory loss only
c. Claw hand
d. Unable to do thumb opposition

The correct answer is d


2- Radial nerve injury : wrist drop , common with humers injury (humers groove)
3- Unlar nerve injury : claw hand , common with elbow injury
4- Median nerve inury :unable to do thumb opposition , common with wrist injury

……………………………………………………………………………………………………………………………………………………..

669) Best way to diagnose post streptococcus glomerulonephritis (spot diagnosis):


a) Low C3
b) ?or RBC casts
c) ?
The correct answer is a ( I'm not sure )
With these MCQs , I think the correct answer is a , but most sensitive in diagnosis of
post streptococcus glomerulonephritis is increase ASO titer : lumpy-bumby
immunofluorescence. or (streptozyme test, which tests antibodies to ASO) ( if ASO
titer written with MCQs , it is correct answer .

…………………………………………………………………………………………………………………………………………………….
.

670) COPD patient with emphysema has low oxygen prolonged chronic high CO2, the
respiratory drive is maintained in this patient by:
a. Hypoxemia
b. Hypercapnemia
c. Patient effort voluntary
d. ?

The correct answer is a ( I'm not sure )

……………………………………………………………………………………………………………………………………………………..

671) Child with ear pain with positive pump test for tympanic membrane, treatment is:
a. Maryngiotomy
b. Amoxicillin/Potassium …….
?
The correct answer is a
672) Patient has fatigue while walking last night. He is on atrovastatin for 8 months,
Ciprofluxacin, Dialtizem and alphaco….. the cause of this fatigue is:
a. Dialtizem and Atrovastatin
b. Atrovastatin and Ciprovluxacin
c. Atrovastatin and Alphaco
d. ?

the correct answer is b ( I'm not sure )

……………………………………………………………………………………………………………………………………………………..

673) Patient with ischemic stroke present after 6 hours, the best treatment is:
a) ASA
b) TPA
c) Clopidogril
d) IV heparin
e) Other anticoagulant

The correct answer is a


1- TPA : administered within 3h of symptoms onset ( if no contraindication)
2- ASA: use with 48h of ischemic stroke to reduce risk of death.
3- Calopidogril : can be use in acute ischemic
4- Heparin & other anticoagulant : in patient has high risk of DVT or AF

……………………………………………………………………………………………………………………………………………………..

674) Patient with lateral and vertical diplobia, he can’t abduct both eyes, the affected
nerve is:
a) II
b) III
c) VI
d) V

I think this is diplopa because nerve 4 , but with this Q I choose c

675) Infant with heamangioma on forehead obscuring the vision, best thing to do is to
remove the lesion to avoid amblyobia, after:
a) One day
b) One week
c) One month
d) 6 months
e) One year

The correct answer is b ( I'm not sure )

……………………………………………………………………………………………………………………………………………………..

676) Photophobia, blurred vision, keratic ?? behind cornea and cells in anterior
chamber, the best treatment is :
a) Topical antifungal
b) Topical Acyclovir
c) Antibiotic
d) ?

The correct answer is b

……………………………………………………………………………………………………………………………………………………..

677) Most common medical problems faced in primary health care is:
a) Coryza
b) UTI
c) Hypertension
d) Diabetes

The correct answer is a

……………………………………………………………………………………………………………………………………………………..

678) Old lady afraid of Osteoprosis, to avoid the risk, you should advise her to do:
a) Weight bearing exercise
b) ?
c) ?
d) ?

Q is not complete.

679) Obese lady with essential hypertension, lab work showed: high Na, High K, The
reason for Hypertension is:
a) Obesity
b) High Na intake
c) High K intake
d) ?

The correct answer is a , more than 85% of essential hypertension with BMI >
25 .

…………………………………………………………………………………………………………………………………………………….
.

680) Patient with eruptive purpuric rash, hepatosplenomegaly, …………


a) ?
b) ?
c) ?
d) ?

……………………………………………………………………………………………………………………………………………………..

681) Single thyroid nodule showed high iodine uptake, best treatment is:
a) Radio Iodine 131
b) Send home
c) Antithyriod medication
d) Excision if present

The correct answer is c

……………………………………………………………………………………………………………………………………………………..
682) Athletic with tina pedis best treatment is:
a) Topical antifungal
b) Systemic antifungal
c) Drug starts with trebenafine
d) ?

The correct answer is a

683) Young patient with congested nose, sinus pressure, tenderness and green nasal
discharge, has been treated three times with broad spectrum antibiotics
previously, what is your action?
a) Give antibiotic
b) Nasal corticosteroid
c) Give anti histamine
d) Decongestant

The correct answer is b

……………………………………………………………………………………………………………………………………………………..

684) Thyrotoxicosis include all of the following, Except:


a) Neuropathy
b) Hyperglycemia
c) Peripheral Proximal myopathy
d)

Q is not complete , all MCQs are wrong , may be choice d is correct if mention .

……………………………………………………………………………………………………………………………………………………..

685) Male patient has hair loss started as fronto-temporal and moving toward the
vertex (top of the head) the diagnosis is:
a) Androgenic alopecia
b) Tinea Captus
c) ?
d) ?

The correct answer is a

……………………………………………………………………………………………………………………………………………………..

686) Young healthy male has abdominal pain after basketball. Examination fine except
for Left paraumbilical tenderness, what to do:
a) Abdominal US
b) Flat plate graph
c) Send home & reassess within 48 hours
d) ?

The correct answer is c ( I'm not sure ) , confuse with choice a

687) 10 year-old boy with ….to tell that spinal cord length will stop after:
a) ?
b) ?
Length stop at L1 \ L2

……………………………………………………………………………………………………………………………………………………..

688) Child with atopic dermatitis at night has stridor plus barking cough on and off
from time to time, diagnosis is:
a) BA
b) Croup
c) Spasmadic Croup
d) ?

The correct answer is c


Spasmadic croup : recurrent sudden upper airway obstruction which present
as sridor and cough . approximately 50% of children have atopic disease.

……………………………………………………………………………………………………………………………………………………..

689) Sickle cell anemia patient presented with asymptomatic unilateral hip pain, most
likely diagnosis is:
a) Septic arthritis
b) Avascular Necrosis
c) ?

The correct answer is b

……………………………………………………………………………………………………………………………………………………..

690) Best drug to treat depression in children and adolescent is:


a) Fluxetine (Prozac)
b) ?

……………………………………………………………………………………………………………………………………………………..

691) Old male with neck stiffness, numbness and parasthesia in the little finger and
ring finger and positive raised hand test, diagnosis is:
a) Thoracic outlet syndrome
b) Impingement syndrome
c) Ulnar artery thrombosis
d) Do CT scan for Cervical spine
The correct answer is a
692) Patient diagnosed with obstructive jaundice best to diagnose common bile duct
obstruction:
a) ERCP
b) US
c) ?

The correct answer is a

……………………………………………………………………………………………………………………………………………………..

693) Endometriosis best diagnosed by


a) US
b) Laproscopy
c) Lapratomy
d) ?

The correct answer is b

……………………………………………………………………………………………………………………………………………………..

694) Child was sick 5 days ago culture taken showed positive for meningococcal.
Patient now at home and asymptomatic your action will be:
a) Rifampicin
b) IM Ceftrixone
c) ?
d) ?
Q not complete ( i.m ceftriaxon single dose or oral rifampicin 2-5 days )

……………………………………………………………………………………………………………………………………………………..

695) Infant with bright blood, black stool and foul smelling stool. Best way to know the
diagnosis:
a) US
b) Radio Isotop scan
c) Angiogram
d) ?

All MCQs are wrong , if stool analysis is present , it is correct.

696) Primary hyperaldosteronism associated with:


a) Hypernatremia
b) Hypomagnesemia
c) Hypokalemia
d) Hyperkalemia

The correcr answer is c

……………………………………………………………………………………………………………………………………………………..
697) A vaccination for pregnant lad with DT
a) Give vaccine and delivery within 24 hrs
b) Contraindicated in pregnancy
c) Not contraindicated in pregnancy
d) ?

The correct answer is c

……………………………………………………………………………………………………………………………………………………..
698) A study done to assess the risk of long taking Ca in two groups the diseased group
with long Ca plus control according to geographical location, site, and population.
It adds (??) this type of study:
a) Cohort
b) Case Control (retrospective)
c) Correlation study
d) ?

The correct answer is a

Cross-Sectional Study:
status of individual with respect to presence and absence of both exposure and
disease assessed at one point in time

case control study ( retrospective ) :


samples a group of people who already have a particular outcome (cases) and
compares them to a similar sample group without that outcome (controls)

cohort study : ( prospective, incidence , longitudinal )


subjects are sampled and as a group are classified on the basis of presence or
absence of exposure to a particular risk factor
……………………………………………………………………………………………………………………………………………………..

699) Define Epidemiology


a) ? “The study of the distribution and determinants of disease prevalence in man”

700) Case Control description


a) Start with the outcome the FU risk factors
b) ?

…………………………………………………………………………………………………………………………………………………….
.
701) Using gastric lavage :
a) Useless after 8 hours of ASA ingestion
b) No benefit after 6 hours of TCA ingestion
c) ?
The correct answer is a

……………………………………………………………………………………………………………………………………………………..
702) 10 years old child with rheumatic fever treated early, no cardiac complication.
Best to advice the family to continue prophylaxis for:
a) 1 month
b) 3 ys
c) 4 ys
d) 15 ys

The correct answer is d ( I'm not sure , if there is 11 years choose it )


American heart associated has recommended all patient with history
rheumatic fever be placed on longe term penicillin prophylaxic. Duration of
prophylaxis depends on presence or absent of carditis, but for children
without carditis , duration minimum 5 years or at age 21, whichever is longer.

703) Picture of large neck mass only no other manifestations or organomegaly or


lymphadenopathy, diagnosis is:
a) Mononucleosis
b) I would say Goiter
c) ?
d) Lymphoma

The correct answer is b

……………………………………………………………………………………………………………………………………………………..
704) Patient with nausea, vomiting, and diarrhea developed postural hypotension.
Fluid deficit is:
a) Intracellular
b) Extracellular
c) Interstitial

The correct answer is b


……………………………………………………………………………………………………………………………………………………..

705) Cardiac syncope:


a) Gradual onset
b) Fast recovery
c) Neurological sequence after
d) ?

The correct answer is b

……………………………………………………………………………………………………………………………………………………..

706) Best way to decrease pain in elderly with bilateral knee pain and crepitation is:
a) NSAID
b) Decrease weight
c) Exercise
d) ?

The correct answer is b , (I'm not sure )

707) Young female with whitish grey vaginal discharge KOH test ?? smell fish like
diagnosis is:
a) Gonorrhea
b) Bacterial Vaginosis
c) Trachomanous Vaginalis
d) ?

The correct answer is b

……………………………………………………………………………………………………………………………………………………..

708) Old lady with osteoporosis asked for treatment for prevention:
a) VIT. D
b) VIT. E
c) Retonic Acid
d) ?
The correct answer is a

……………………………………………………………………………………………………………………………………………………..

709) Young male with morning stiffness at back relieved with activity and uveitis:
a) Ankylosing Spondylitis
b) ?

……………………………………………………………………………………………………………………………………………………..

710) Best way to prevent infection in medical practice in pediatric


a) Wear gloves
b) Wash hand
c) Wear mask
d) Wear gown

The correct answer is b

……………………………………………………………………………………………………………………………………………………..

711) High risk for developing colon cancer in young male is:
a) Smoking, high alcohol intake, low fat diet
b) Smoking, low alcohol intake, high fat diet
c) Red meat diet, garden’s disease (Gardner syndrome)
d) Inactivity, smoking

The correct answer is c


712) Alternative therapy for severe depression and resistance to anti-depressant
medications are:
a) SSRI
b) TCA
c) ECT

The correct answer is c

……………………………………………………………………………………………………………………………………………………..

713) Patient had history of pancreatic cancer on chemotherapy then improved


completely, came to doctor concerning about recurrence of cancer and a history
of many hospital visits. This patient has:
a) Malingering
b) Hypochondriasis
c) Factitious
d) Conversion

The correct answer is b


……………………………………………………………………………………………………………………………………………………..

714) Patient came with neck swelling, ,moves when patient protrude his tongue.
Diagnosis is:
a) Goiter
b) Tyroglossus Cyst
c) Cystic Hygroma
d) ?

The correct answer is b

……………………………………………………………………………………………………………………………………………………..

715) Pregnant patient came with neck swelling and multiple nodular non-tender goiter
the next evaluation is:
a) Thyroid biopsy
b) Give anti-thyroid medication
c) Radiation Iodine
d) TSH & Free T4, or just follow up

The correct answer is d

716) Young patient with HTN came complaining of high blood pressure and red, tender,
swollen big left toe, tender swollen foot and tender whole left leg. Diagnosis is:
a) Cellulitis
b) Vasculitis
c) Gout Arthritis
d) ??

The correct answer is a , because tender and swollen whole left leg.

……………………………………………………………………………………………………………………………………………………..

717) What is the injection that is routinely given to newborn to inhibit hemorrhage:
a) Vit. K
b) Vit. C
c) Vit. D
d) Vit. E

The correct answer is a

……………………………………………………………………………………………………………………………………………………..

718) Patient with strong genetic factor for colon cancer, what is the medication that
could decrease the risk of colon cancer:
a) Folic Acid.
b) Vit. C
c) Vit. K or A
d) Vit. E

The correct answer is a ( folaic acid and vit. C both are prevent colon cancer , but
folat reduce risk in people who genic predisposing )

……………………………………………………………………………………………………………………………………………………..

719) Patient with asthma, well controlled by albutarol, came complaining of asthma
symptoms not respond to albutarol, what medication could be added:
a) Corticosteroid inhaler
b) Long acting B-agonist
c) Oral corticosteroid
d) Theophyline

The correct answer is a

720) Henosch-Scholen purpura affect:


a) Capillary
b) Capillary and venule
c) Arteriole, capillary and venule
d) Artery to vein

The correct answer is c

……………………………………………………………………………………………………………………………………………………..

721) Contraceptive pill that contain estrogen increase risk of:


a) Breast Ca
b) Ovary Ca
c) Cervical Ca
d) ?

The correct answer is a

……………………………………………………………………………………………………………………………………………………..

722) Patient came with upper respiratory tract infection with red conjunctiva, the
cause is:
a) Viral infection
b) Bacterial infection
c) Fungal infection
d) ?

The correct answer is a

……………………………………………………………………………………………………………………………………………………..
723) Healthy patient with family history of DM type 2, the most factor that increase
chance of DM are:
a) HTN and Obesity
b) Smoking and Obesity
c) Pregnancy and HTN
d) Pregnancy and Smoking

The correct answer is a

724) Patient complaining of back pain and hypersensitive skin of the back, on
examination, patient had rashes in the back, tender, red base distributed as blunt
shape on the back, diagnosis is:
a) Herpes Zoster
b) CMV
c) ?
d) ?

The correct answer is a ( q not complete )

……………………………………………………………………………………………………………………………………………………..

725) Old patient complaining of hematuria, on investigation, patient has bladder


calculi, most common causative organism is:
a) Schistosoma
b) CMV
c) ? virus
d) ? virus

The correct answer is a

……………………………………………………………………………………………………………………………………………………..

726) Blood culture show gram negative rod shape that grow only on charcoal free
fungal organism is:
a) Staph. Aureus
b) Chlamydia
c) Klebsiella
d) Mycoplasma

All above are wrong , legionella : grame negative rod growth on charcoal agar

……………………………………………………………………………………………………………………………………………………..

727) Klebesilla feacalis cause the following disease:


a) Pneumonia
b) ?
c) ?
d) ?
Enterococcus faecalis  Cause Gastroenteritis

……………………………………………………………………………………………………………………………………………………..

728) Dermatomyositis came with the following symptoms:


a) Proximal muscle weakness
b) Proximal muscle tenderness
c) ?
d) ?

The correct answer is a

……………………………………………………………………………………………………………………………………………………..

729) Bursitis of the elbow joint caused by:


a) Elbow trauma
b) Autoimmune disease
c) Staph. Aureus
d) ? rupture of bursa

The correct answer is a

……………………………………………………………………………………………………………………………………………………..

730) Patient came with symptoms of anxiety including palpitation, agitation, and
worry. The first best line for treatment is:
a) SSRI
b) TCA
c) B-blocker
d) MAOI
The correct answer is a , I'm not sure
May be the correct answer is b because there is agitation ( also, side effect of
SSRI)

……………………………………………………………………………………………………………………………………………………..

731) Pregnant diagnosed with UTI. The safest antibiotic is:


a) Ciprofloxacin
b) Ampiciln
c) Tetracycline
d) ?

With these MCQs , the correct answer is b , but if present nitrofurantion is more
accurate answer . UTI in pregnancy treated by : nitrofurantion or
cephalosporine ( 3– 7 days ) in symptomatic or asymptomatic UTI . avoid
fluroroquinolone ( which include : ciprofloxacin, gatifloxacine, levofloxacin,
norfloxacin ).
732) Patient is complaining of right side pharynx tenderness on examination patient
had inflamed right tonsil and redness around tonsil with normal left tonsil. The
diagnosis is:
a) Parenchymal tonsillitis
b) Quinse parapharyngeal abscess
c) ?
d) ?

The correct answer is b

……………………………………………………………………………………………………………………………………………………..

733) Patient came complaining of fever, night sweating, and hemoptysis with positive
PPD test. Examination was normal, CXR shows infiltrate of left apical lung but in
lateral X-ray showed nothing the repeated PPD test showed normal result
diagnosis is:
a) Sarcoidosis
b) Reactivated TB
c) Mycoplasma infection
d) Viral infection

The correct answer is b

…………………………………………………………………………………………………………………………………………………….
.

734) Femal patient came with lower abdominal pain, fever on exam patient has lower
abdominal tenderness and tender cervical fornix, the most appropriate way to
diagnose the problem is:
a) Laproscopy
b) Heterosalpingography
c) Abdominal CT
d) Radionuclar Study

The correct answer is a

……………………………………………………………………………………………………………………………………………………..
735) The best non-medical therapy is proven to be of benefit for osteoarthritis is:
a) Muscle strength exercise
b) Give NSAID
c) Back slap
d) ?
The correct answer is a
736) Hematological disease occurs in children, treated with heparin and fresh frozen
plasma what is the disease:
a) Hemophilia A
b) Hemophilia B
c) Von-wille brand disease
d) DIC thrombosis

The correct answer is d

……………………………………………………………………………………………………………………………………………………..

737) Child with URTI is complaining of bleeding from nose, gum and bruising the
diagnosis is:
a) Hemophilia A
b) ITT
c) ?
d) ?

The correct answer is b

……………………………………………………………………………………………………………………………………………………..

738) Patient came complaining of vague abdominal pain for 6 hours then shifted to
right lower quadrant diagnosis is :
a) Acute appendicitis
b) Diverticulitis
c) ?
d) ?

The correct answer is a

……………………………………………………………………………………………………………………………………………………..

739) Female patient is complaining of abdominal distension, fever and nausea


abdominal x-ray showed (Ladder sign) management is:
a) Colostomy
b) Ileus treatment
c) Rectal de-obstruction
d) ?

The correct answer is b ( I'm not sure because Q IS NOT COMPLETE )

740) The best stimulus for breast milk secretion is :


a) Estrogen
b) Breast feeding
c) ?

The correct answer is b


……………………………………………………………………………………………………………………………………………………..

741) Female patient did urine analysis shows epithelial cells in urine, it comes from:
a) Vulva
b) Cervix
c) Urethra
d) Ureter

The correct answer is c

……………………………………………………………………………………………………………………………………………………..

742) All of the following are risk factors for heart disease except:
a) High HDL
b) Male
c) Obesity

The correct answer is a

……………………………………………………………………………………………………………………………………………………..

743) The most signs and symptoms of abruption of placenta is:


a) Vaginal bleed
b) Fetal distress
c) Uterus pain and back pain
d) Abnormal uterine contraction

The correct answer is a

744) Sign of severe hypokalemia is:


a) P-wave absence
b) Peak T-wave
c) Wide QRS complex
d) Seizure

The correct answer is d , ( I'm not sure , may be choice e if mention , is corect )
severe hypokalemia is defined as a level less than 2.5 mEq/L.

Severe hypokalemia is not linked with any symptoms, but may cause:
1- muscle
2- myalgia or muscle pain
3- disturbed heart rhythm including ectopy (disturbance of the electrical
conduction system of the heart where beats arise from the wrong part of the
heart muscle)
4- serious arrhythmias (electrical faster or slower than normal)
5- greater risk of hyponatremia (an electrolyte disturbance in humans when the
sodium concentration in the plasma decreases below 135 mmol/L) with confusion
and seizures

ECG changes in hypokalemia :


1-T-wave flattening
2-U-wave : ( additional wave after the T wave )
3-ST – segment depression

ECG changes in hyperkalemia :


1- peak T wave
2- wide QRS ( in severe case )
3- PR prolong ( in severe case )
4- loss of P wave

……………………………………………………………………………………………………………………………………………………..

745) Child came with his father and high BMI and look older than other children with
same age, on exam child has >95th percentile of weight and tall, management is:
a) Observe and appoint
b) Life style change
c) Give program to decrease the weight
d) ?

The correct answer is a

746) Pregnant on 36th week came with 7 cm cervical width at 0 station. During birth,
CTG shows late deceleration, management is:
a) Give Oxytocin
b) O2 and change mother position
c) Give Mg sulfate
d) ?

The correct answer is b

Type of deceleration etiology management

early Head compression from No treatment


uterine contraction
(normal )

late Uteroplacenta insufficiency  Place patient on side


and fetal hypoxima  Discontinue oxytocin.
 Correct any hypotension
 IV hydration.
 If decelerations are associated with tachysystole
consider terbutaline 0.25 mg SC
 Administer O2
 If late decelerations persist for more than 30
minutes despite the above maneuvers, fetal scalp
pH is indicated.
 Scalp pH > 7.25 is reassuring, pH 7.2-7.25 may be
repeated in 30 minutes.
 Deliver for pH < 7.2 or minimal baseline
variability with late or prolonged decelerations and
inability to obtain fetal scalp pH

variable Umbilical cord  Change position to where FHR pattern is most


compression improved. Trendelenburg may be helpful.
 Discontinue oxytocin.
 Check for cord prolapse or imminent delivery
by vaginal exam.
 Consider amnioinfusion
  Administer 100% O2

747) The way to determine the accuracy of occult blood test for 11,000 old patients is
by measuring:
a) Sensitivity
b) Specificity
c) Positive predictive value
d) Negative predictive value

The correct answer is a

……………………………………………………………………………………………………………………………………………………..

748) Sickle cell patient, asymptomatic with history of recurrent gall-stones and
recurrent crisis the management is:
a) Cholecystectomy
b) Hydroxyurea
The correct answer is a

……………………………………………………………………………………………………………………………………………………..

749) Patient came with HTN, KUB shows small left kidney, arteriography shows renal
artery stenosis, what is the next investigation:
a) Renal biopsy
b) Renal CT scan
c) Renal barium
d) Retrograde pyelography

The correct answer is

……………………………………………………………………………………………………………………………………………………..

750) The way to differentiate between low iron level from iron deficiency anemia and
anemia of chronic disease is:
a) Ferritin
b) TIBC
c) Serum Iron
d) Serum Transferrin

The correct answer is a

751) Patient came with hallucination and illusion the medication that should be given
is:
a) Carbamezapin
b) ? Haloperidol
c) ?

Q not complete, but with this scenario and MCQs , the correct answer is b

……………………………………………………………………………………………………………………………………………………..

752) As doctor if you see patient and you face difficulty to get accurate information
from him the best tactic to do it is:
a) Ask direct question
b) Ask open question
c) Control way of discussion
d) ?

The correct answer is a


……………………………………………………………………………………………………………………………………………………..

753) (long question) patient came with MDD so during communication with patient
you will find :
a) Hypomania
b) Late morning awake
c) Loss of eye contact
d) ?

The correct answer is b

……………………………………………………………………………………………………………………………………………………..

754) Child patient after swimming in pool came complaining of right ear tenderness on
examination patient has external auditory canal redness, tender, and discharge
the management is:
a) Antibiotics drops
b) Systemic antibiotics
c) Steroid drops
d) ?

The correct answer is a

755) Child came with inflammation and infection of the ear the most complication is:
a) Labrynthitis
b) Meningitis
c) Encephalitis
d) Mastoiditis

The correct answer is d

……………………………………………………………………………………………………………………………………………………..

756) Elderly patient complaining of urination during night and describe when he feel
the bladder is full and need to wake up to urinate, he suddenly urinate on the bed
this is:
a) Urgency incontinence
b) Urge incontinence
c) Stress incontinence
d) Flow incontinence

The correct answer is d

types Hx of urine loss mechanism treatment


total Uncontrolled loss at all Loss of sphincter efficiency or surgery
times and in all position abnormal connection b\w
urinary tract and skin( fistula)

stress After increase abdominal Urethral sphincter Kegal exerciseand


pressure (coughing, insufficiency due to laxity of pessary
sneezing, lifting ) pelvic floor musculature , surgery
common in multiparous
women

urge Strong, unexpected urge to Detrusor hyperreflexia or Anticholinergic –


void that is unrelated to sphincter dysfunction or medication or TCA :
position or activity neurologic disorder behavior training

overflow Chronic urinary retention Increase intravesical pressure Placement of urethra


that just exceed the outlet catheter , treat
resistance , allow small underlying causes
amount of dribble out

757) Newborn came with red-lump on left shoulder, it is:


a) Hemangioma
b) ?
c) ?

Q is not complete

……………………………………………………………………………………………………………………………………………………..

758) Child came to ophthalmology clinic did cover test, during eye cover , his left eye
move spontaneously to left, the most complication is:
a) Strabismus
b) Glaucoma
c) Myobloma
d) ?

The correct answer is a

……………………………………………………………………………………………………………………………………………………..

759) Newborn came with congenital hepatomegaly, high LFT, jaundice the most
organism cause this symptoms is:
a) Congenital TB
b) Rubella
c) HIV
d) CMV

The correct answer is d

……………………………………………………………………………………………………………………………………………………..

760) Gross motor assessment at age of 6 months to be asked is:


a) Sitting without support
b) Standing
c) Role from prone to supine position
d) Role from supine to prone position

The correct answer is a

761) Female child came with precocious puberty the most cause is:
a) Idiopathic
b) Adrenal tumor
c) Brain tumor
d) ? ovarian tumor

The correct answer is a

……………………………………………………………………………………………………………………………………………………..

762) Hemorrhoid usually occurs in:


a) Pregnancy and portal HTN
b) ?
c) ?
d) ?

Q is not complete
Most common causes of hemorrhoid : increase straining ( constipation ) , portal
HTN , increase abdominal pressure ( chroming cough , pelvic tumor ) , obesity ,
pregnancy , smoking

……………………………………………………………………………………………………………………………………………………..

763) The immediate urgent referral of child that take


a) 10 pills contraceptive
b) 10 pills antibiotics?
c) 75 mg ?

?????

……………………………………………………………………………………………………………………………………………………..

764) Most unwanted side effect of anti-cholinergic drugs is :


a) Constipation
b) ?
c) ?

more than 50% of patients taking anticholinergic have side effects : dry mouth,
blurry vision, constipation and urinary retention.

765) Patient with DM II with good vision, to prevent eye disease (Retinal back ground)
to develop is to avoid:
a) HTN, Smoking
b) Obesity, Smoking
c) HTN, Obesity
d) ?

The correct answer is c


The risk factors that increase diabetic retinopathy background are:
1- HTN
2- Poor glucose control or long case D.M
3- Raised level of fat ( cholesterol)
4- Renal disease
5- Pregnancy ( but not in diabetes caused by pregnancy )
……………………………………………………………………………………………………………………………………………………..

766) Best medication to be given for GDM (gestational) is:


a) Insulin
b) Metformin
c) ?

The correct answer is a

767) A child is complaining of severe headache which is unilateral, throbbing and


aggravated by light, diagnosis is:
a) Migraine
b) Cluster Headache
c) Stress Headache
d) ?

The correct answer is a

……………………………………………………………………………………………………………………………………………………..

768) The most important factor for smoker to quit is :


a) Patient desire
b) Give nicotine pills
c) Give programmed plan
d) Change life style

The correct answer is a

……………………………………………………………………………………………………………………………………………………..
769) Patient is complaining of irritation, tachycardia, night sweating, labs done showed
TSH: Normal, T4: High, diagnosis is:
a) Grave’s disease
b) Secondary Hypothyriodism
c) Hashimoto’s thyroiditis
d) ?

All MCQs are wrong , choice d may be correct if mention


Normal TSH and increase T4 : in thyroid tropin secretary pitutay adenoma =
thyroid resistance .

……………………………………………………………………………………………………………………………………………………..

770) The most active form is:


a) T4
b) T3
c) TSH
d) TRH

The correct answer is b

771) Middle age man found to have heaviness in his groin. On physical examination
there was swelling just above his testis which apparent with valsalva maneuver.
What is the diagnosis:
a) Direct inguinal hernia
b) Indirect inguinal hernia
c) Femoral Hernia
d) Testicular mass
e) Hydrocele
f) Varicocele

The correct answer is b

……………………………………………………………………………………………………………………………………………………..

772) Gardener has recurrent conjunctivitis. He can’t avoid exposure to environment. In


order to decrease the symptoms in the evening, GP should advise him to:
a) Cold compression
b) Eye irrigation with Vinegar Solution
c) Contact lenses
d) Antihistamines

The correct answer is d

……………………………………………………………………………………………………………………………………………………..
773) 48 year-old male complaining of lower back pain with morning stiffness for 30
minutes only. On exam he was having spasm centrally on the lower back. What is
the appropriate management :
a) Epidural steroids injection
b) Back brace
c) Facet lysis
d) Physiotherapy

The correct answer is d

……………………………………………………………………………………………………………………………………………………..
774) 41 weeks pregnant lady last biophysical profile showed oligohydroamnios. She
has no complaints except mild HTN. What is the appropriate management :
a) Wait
b) Induce labor post 42 wks
c) Induce labor
d) Do biophysical profile twice weekly
The correct answer is c
775) Full term wide pelvis lady, on delivery station +2, vertex, CTG showed late
deceleration, the most appropriate management:
a) C/S
b) Suction
c) Forceps Delivery
d) Spontaneous Delivery

The correct answer is c (I'm not sure )

……………………………………………………………………………………………………………………………………………………..

776) Recent study revealed that anti psychotic meds cause the following complication:

a. wt gain
b. alopecia
c. cirrhosis

the correct answer is a

……………………………………………………………………………………………………………………………………………………..

777) compilication of rapid correction of hypernatremia :


a.brain edema

in hypernatremia : gradually correction to prevent cerebral edema


in hyponatremia : gradually correction to prevent ,myelinolysis ( include
paraparesis \ quadriparesis , dysarthria and coma

……………………………………………………………………………………………………………………………………………………..
778) prophylaxis of cholera :

Cholera is an infection of the small intestine that is caused by the bacterium Vibrio
cholerae. The main symptoms are profuse watery diarrhea ( rice-watery diarrhea )
vomiting and dehydration . infection by fecal-oral rout .

treatment : 1- rehydration , 2-antibiotic : young & adult : doxcycline or tetracycline ,


for children : SMX-TMP , for pregnant : furazolidone .

prphylaxis : good hygiene and sanitation and oral vaccine , in epidemic public : mass
single dose of vaccine and tetracycline.

778)which one of the anti TB medications cause tinnitus, imbalance..

a. streptomycin

b. isoniazide

c. pyrizinamide

the correct answer is a , sterptomycine : cause 8th nerve damage

…………………………………………………………………………………………………………………………………………………….
.
779) the following combination drug should be avoided :

a. levodopa & digoxin

Q is not complete

……………………………………………………………………………………………………………………………………………………..
780)young male had pharyngitis,then cough ,fever,most likely org

a. staph aureus

b. strept pneumonia

the correct answer is b

……………………………………………………………………………………………………………………………………………………..
781)most effective measure to prevent spread of infection among health care workers
& pts in a nursery:

a. wash hand befor and after examining each pt

b. wear gown and gloves before entering the nursery

c. wear shoe cover

the correct answer is a

782) 27 yrs old female with perianal pain for 4 days tender erythematous fluctuating

a. Abx

b. local CS

c . Sitz bath

e. evac & drain

the correct answer is e

……………………………………………………………………………………………………………………………………………………..
783)female pt developed extreme from zoo,park, sporting events, the fear prevented
her from going out:

a. agoraphobia

b. social phobia

c. schizophrenia

the correct answer is a , ( agoraphopia : fear going out from the home )

……………………………………………………………………………………………………………………………………………………..
784)child starts to smile:

a. at birth
b. 2month

c. 1month

The correct answer is b

…………………………………………………………………………………………………………………………………………………….
.
785)child recognize 4 colors, 5 words, hops on one foot, consistent with which age:

a. 12 mons

b. 24 mons

c. 36 mons

d. 18 mons

the correct answer is c

786)pt with hx of 5 yrs HTN on thiazide, came to ER midnight screaming holding his Lt
foot, o/e pt afebrile,Lt foot tender erythema, swollen big toe most tender and
painful, no other joint involvement

a. cellulitis

b. gouty arthritis

c. septic arthritis

the correct answer is b

……………………………………………………………………………………………………………………………………………………..
787)23 yrs old married for 3 mons, c/o not getting pregnant, they have intercourse 3-4
times/week, normal gynecologic hx, husband 25 yrs old healthy wt would you
advice:

a. cont. trying
b. obtain sperm analysis
c. study of tubes patency

the correct answer is a

……………………………………………………………………………………………………………………………………………………..
788)pt with AF what is the most common complication:
a. cerebrovascular events

b. v.tach

c. AMI

d. v.fib

the correct answer is a

……………………………………………………………………………………………………………………………………………………..
789)case scenario : oral+genital ulcer +arthritis

a. behcet disease

b. syphilis

c.herpes simplex

the correct answer is a

790) 20 yrs old lady, pregnant, exposed to rubella virus since 3 days , never was
vaccinated against rubella mumps or measles , what's the best thing to do:

a. give IG

b. vaccine

c. do nothing

d. terminate the pregnancy

the correct answer is c , only supportive . ( 100 % correct  )

……………………………………………………………………………………………………………………………………………………..

790)which of the following not compatible with head engagement:

a. vertex at zero station

b. crowning of the head

c. 3/5 head felt in the abdomen

d. BPD at ischial spines


the correct answer is c , When the fetal head is engaged, 2/5 or less of the head
is palpable above the pelvic

791)58 yrs old female, known case of osteopenia, she's asking you abt the best way to
prevent compression vertebral fracture, what would you advice her:

a. avoid obesity

b. vit.D daily

c. wt bearing exercise

the correct answer is c , cnfuse with choice b

mild osteopenia ( T score -1 to -2 ): life style modification ( stop smoking ) , daily


calcium intak ( 1500 mg daily ) , exercise , and reassessed after 3 – 5 years .

severe osteopenia ( T score -2 to – 2.5 ) and osteoporosis ( T score > 2.5 ) : same
mild pus pharmacological .

……………………………………………………………………………………………………………………………………………………..
792) Mother complains of sharp pain on radial styloid when carrying her baby. The
pain increase with extension of the thumb against resistance, Finkelstein test was
positive, Dx :
a. Osteoarthritis of radial styloid
b. De Quervain Tenosynovitis
the correct answer is b

…………………………………………………………………………………………………………………………………………………….
.

793) 1st line in Trigeminal Neuralgia management:


a. Carbamazepine

…………………………………………………………………………………………………………………………………………………….
.
794) Contraindicated in acute glaucoma management:
a. Pilocarpine
b. Timolol
c. B-blockers, CA inhibitors, NSAID, Mannitol
d. ?

All of MCQs can be use in acute glaucoma . may be choice d is correct if


mention.

795) True about systolic hypertension:


a. could be caused by mitral Regurg
b. More serious than diastolic hypertension
c. systolic>140 and diastolic <90
d. ?

the correct answer is c

…………………………………………………………………………………………………………………………………………………….
.
796) Pregnant 38GA, presented in labor, dilated cervix, station 42, late deceleration on
CTG, management:

a. continuo spontaneous labor

b. Forcep delevary

c. Vacuum Delevary

d. CS

The correct answer is a ( I'm not sure )

……………………………………………………………………………………………………………………………………………………..
797) Pt, febrile, tender prostate on PR:

a-Acute Prostitis
……………………………………………………………………………………………………………………………………………………..

798) Proven to prevent some cancers:


a. Ca
b. Folic Acid
c. Vit.D

the correct answer is c

……………………………………………………………………………………………………………………………………………………..

799) Patient with cont. Murmur:


a. PDA
b. Coarctation of Aorta

the correct answer is a

800) Old man, post OR, Complains of unilateral face swelling:


a. Sjorjan syndrome
b. Sarcoidosis
c. Salivary gland tumor
d. Salivary gland stone

the correct answer is d

……………………………………………………………………………………………………………………………………………………..
801) At what level LP done:
a. L2-L3
b. L3-L4
c. L5-S1

The correct answer is b

……………………………………………………………………………………………………………………………………………………..

802) 2months old with scaling lesion on scalp and forehead, Dx:
a. Seberrhoic Dermatitis
b. Erythema multiforme

the correct answer is a

……………………………………………………………………………………………………………………………………………………..
803) Pt have high Blood Pressure on multible visits, so he was diagnosed with
hypertension, what is the Pathophysiology:
a. increased peripheral resistance
b. increased salt and water retention

the correct answer is a

……………………………………………………………………………………………………………………………………………………..
804) 13years old with hx of pneumonia and managed with abx 2 weeks back, now he
came with diarrhea, abdominal pain, and +ve WBC in stool, the causative
organism is:
a. Clostridium dificile

……………………………………………………………………………………………………………………………………………………..

805) Best Manag. Of Clost.Deficile is:


a. Metronidazole
b. Doxacycline

the correct answer is a

806) Prophylaxis of arrhythmia post MI :


a. Quinidine
b. Quinine
c. Lidocaine
d. Procainamide

the correct answer is c , if b-blocker is present choose it

……………………………………………………………………………………………………………………………………………………..
807) Chronic Diarrhea is a feature of:
a. HyperNatremia
b. HyperCalcemia
c. HypoMagnesemia
d. Metabolic Alkalosis

The correct answer is d

……………………………………………………………………………………………………………………………………………………..
808) Pt with have Polyuria and thirst, he had Hx of bipolar on lithium, Dx:
a. Nephrogenic Diabetes Insipidus
b. central Diabetes Insipidus

the correct answer is a

……………………………………………………………………………………………………………………………………………………..
809) In city with population of 15000 people & 105 birth /yr , 4 are still birth ,3 died
within months ,2 died before their 1st birthday , with 750 p come out & 250 come
in what is the birth mortality rate in this city :
a. 4
b. 6
c. 8
d. 9

Q not complete ( I did't finde any thing about bierth mortality rate )

810) Female with recently inserted IUCD coming with watery brownish vaginal
discharge & abdominal pain what is the most likely Dx :
a. Uterine rupture
b. Ovarian torsion
c. Bacterial vaginosis
d. Ectopic pregnancy

The correct answer is d

……………………………………………………………………………………………………………………………………………………..
811) 38 y/o male calling you complaining of fear from going outside & fear from seeing
the front door & don’t sit in the room containing the front door what is most
likely Dx:
a. Depression
b. Agoraphobia
c. Malingering
d. Anti-social personality

The correct answer is b

……………………………………………………………………………………………………………………………………………………..
812) 60 y/o male known to have ( BPH) digital rectal examination shows soft prostate
with multiple nodularity & no hard masses , the pt request for ( PSA) for screening
for prostatic ca what will you do :
a. Sit with the pt to discuss the cons & rods in PSA test
b. Do trans-rectal US because it is better than PSA in detection
c. Do multiple biopsies for different sites to detect prostatic ca
d. …………………….
The correct answer is a

……………………………………………………………………………………………………………………………………………………..
813) What is the most effective ttt for rocasea :
a. Clindamycine
b. Erythromycin
c. Topical steroids
d. ……..

Q is not clear or not complete. ( if mention tetracycline choose it )

Treatment of Rosacea : Oral tetracyclines & topical metronidazole or topical


erthyromycin or topical clinamycine . for severe case : isotretinoin . surgical
treatment for rhinophyma .

814) 7 y/o child coming with SOB and wheezing he was sitting in bed , leaning forward ,
with drooling & strider what is Dx :
a- Epiglottits
b- Bronchial asthma
c- ………
d- ………

The correct answer is a

……………………………………………………………………………………………………………………………………………………..

815) 10 y/o child brought by his parents because they where concern about his weight ,
he eats a lot of fast food and French fries , your main concern to manage this pt
is :
a- His parents concerning about his weight
b- His BMI > 33
c- Family Hx of heart disease
d- Eating habit ( fast food , French fries )

……………………………………………………………………………………………………………………………………………………..
816) What is an absolute contraindication of OCP :
a. Hx of previous DVT
b. Ovarian ca
c. Breast ca
d. ……….

The correct answer is a , both hx of DVT and breast cancer are absolute
contraindication , but in hx DVT is more accurate

……………………………………………………………………………………………………………………………………………………..
817) OCP is proven to :
a. Decrease ovarian ca
b. Decrease endometrial ca
c. Increase breast ca
d. Increase risk of ectopic pregnancy

The correct answer is c

818) Baby born & discharge with his mother , 3weeks later he started to develop
difficulty in breathing & become cyanotic what is most likely DX :
a- VCD
b- Hypoplastic lt ventricle
c- Coarctaion of aorta
d- Subaortic hypertrophy

The correct answer is b

…………………………………………………………………………………………………………………………………………………….
.
819) 60 y/o male Dx to have acute pancreatitis ( there was values for ranson criteria
that I couldn’t memorize ) what is the appropriate nutrition :
a- TPN
b- Regular diet with low sugar
c- High protein ,high ca , low sugar
d- Naso-jujenal tube

The correct answer is d

……………………………………………………………………………………………………………………………………………………..

820) 15 y/o boy brought by his mother with lab values shows microcytic hypochromic
anemia what is most likely Dx :
a- Thalassemia
b- IDA ( iron deficiency anemia )
c- SCD
d- …….

The correct answer is b

……………………………………………………………………………………………………………………………………………………..
821) Female with Hx of lt flank pain radiating to groin , symptoms of UTI & N+V what is
DX:
a- Appendicitis
b- Diverteculitis
c- Renal colic
d- ……..

The correct answer is c

822) Old pt with neck swelling , nodular ,disfiguring , with Hx of muscle weakness , cold
intolerance , hoarseness , what is your management :
a- Levothyroxine
b- Carbamazole
c- Thyroid lobectomy
d- Radio-active iodine

The correct answer is a

……………………………………………………………………………………………………………………………………………………..
823) Pregnant woman with symptoms of hyperthyroidism , TSH low :
a- Propylthyiouracil
b- Radio-active iodine
c- Partial thyroidectomy
d- ………….

The correct answer is a

……………………………………………………………………………………………………………………………………………………..
824) Pt with thyroid mass , firm ,2x2 cm what is most appropriate for Dx :
a- Neck US
b- FNA
c- Neck CT
d- Surgery

The correct answer is b

……………………………………………………………………………………………………………………………………………………..
825) 56 y/o female complaining of back pain , her mother suffer from vertebral
compression Fx , investigation shows early osteoporosis what is the best ttt :
a- Advice with exercise
b- Vit D , ca supplement , …….
c- …………….
d- ……………

The correct answer is b

……………………………………………………………………………………………………………………………………………………..
826) What is the most common non-traumatic Fx caused by osteoporosis :
a- Colle's Fx
b- Femoral Fx
c- Vertebral compression Fx

The correct answer is c , because most common non traumatic .


827) 4 or 5 ( not sure ) brought by his parents with weight > 95 th percentile , height < 5th
percentile & bowing of both legs what is the appropriate management :
a- Liver & thyroid function tests
b- Lower limb X-ray
c- Pelvis X-ray
d- Thyroid or ( liver not sure ) function test

The correct answer is a

…………………………………………………………………………………………………………………………………………………….
.
828) At which stage sapration of chromatoids occur :
a- Metaphase
b- Telophase
c- ………….
d- ……………

The correct answer is a

……………………………………………………………………………………………………………………………………………………..
829) Elderly pt with DM2 , HTN , smoker , sedentary lifestyle , inestigation low HDL,
high LDL , ( ‫ ) المهم أن كل المآسأي فيه‬which of what will be effective in this pt :
a- Level of HDL , smoking , obesity
b- Triglyceride level , ……………………
c- ………………………………………………….
d- ………………………………………………….( all were long sorry )

MCQs are not complete.

……………………………………………………………………………………………………………………………………………………..
830) pt known case of ulcerative colitis with erythematous rash in lower limb what is
most likely DX:
a- erythema nodusum

831) Pt known to have ulcerative colitis coming with skin lesion around Tibia which is
with irregular margins what is most likely Dx:
a- Pyoderma gangirenosum
…………………………………………………………………………………………………………………………………………………….
.
832) Young pt victim of RTA coming with SOB ( or not breathing not sure ) absent of
breath sound & chest movement in RT side but resonance in percussion what is
your next step :
a- Do stat chest X-ray
b- Emergency surgery
c- …………………………….
d- ………………………………

Needle aspiration then chest tube

……………………………………………………………………………………………………………………………………………………..
833) Psychiatric pt non-compliance to his medication what is the ttt :
a- …………. Given IM
b- ………………….. given IV
c- ………………..oral
d- …………………..oral

Q is not complete

……………………………………………………………………………………………………………………………………………………..
834) Which psychiatric disease is treated with electroconvulsive therapy :
a- Paranoia
b- ………………………..

The correct answer is : Major depression

835) Pt with bilateral eye discharge , watery , red eyes, corneal ulceration what is the
most common cause :
a- Dust & pollen
b- Hypertension & ……………
c- Ultra-violet light & stress
d- ………………………………….
The correct answer is a

……………………………………………………………………………………………………………………………………………………..
836) 4 y/o baby felt down his mother pulled him by his arm & since then he kept his
arm in pronation position what is your management:
a- Splint ………….
b- Do x-ray for the arm before any intervention
c- Orthopedic surgery
d- ……………………………….

The correct answer is b

..……………………………………………………………………………………………………………………………………………………
838) diabetic pt on medication found un conscious his blood suger was 60 what is the
most common to cause this problem:

a- Sufonylurease
b- Bigunides
c- ……………………

The correct answer is a

……………………………………………………………………………………………………………………………………………………..

839) what is most sensitive indicator for factitious fever :

a- Pulse rate
b- …………………….
c- ……………………….

Factitious fever: Fever produced artificially by a patient. This is done by artificially


heating the thermometer or by self-administered pyrogenic substances. An artificial
fever may be suspected if the pulse rate is much less than expected for the degree
of fever noted. This diagnosis should be considered in all patients in whom there is
no other plausible explanation for the fever. Patients who pretend to have fevers
may have serious psychiatric problems.

840) young male ( I think he was a student ) with headache involving the whole head &
bilateral eye glob that increase with stress what is most likely DX:

a- Tension headache
b- Migraine
c- …………………….
d- ……………………….
Q not cmplete

……………………………………………………………………………………………………………………………………………………..
841) pt turns to be erratic , for 4 month he said that’s people in TV knows what people
are thinking about , in last 2 month he claim that he has special power that no
one has what is the most likely DX :

a- Uni-polar ….. depression


b- Bipolar …. Mania
c- SCZ
d- ……………………

The correct answer is c

……………………………………………………………………………………………………………………………………………………..
842) common site of anal fissure is :

a- Anterior
b- Posterior
c- Lateral
d- …………………….

The correct answer is : b

..……………………………………………………………………………………………………………………………………………………
843) common site of hernia sac in relation to the cord content :

a- Antro-lateral
b- Medial
c- Anywhere
d- Posterior

The correct answer is a

844) what a 4 years child can do :

a- Draw square & triangle


b- Say complete sentence
c- Tie his shoes
d- ……………………………..

The correct answer is a

………………………………………………………………………………………………………………………………………………
845) You received a call from a father how has a son diagnosed recently with DM-I for
six months, he said that he found his son lying down unconscious in his bedroom,
What you will tell him if he is seeking for advise:

a. Bring him as soon as possible to ER


b. Call the ambulance
c. Give him his usual dose of insulin
d. Give him IM Glucagone
e. Give him Sugar in Fluid per oral

The correct answer is e

……………………………………………………………………………………………………………………………………………………..
846) factor which determine recurrence of breast cancer :

a- Site & size of breast mass


b- No of lymph nodes
c- Positive estrogen receptor
d- Positive progesterone receptor

The correct answer is b

……………………………………………………………………………………………………………………………………………………..
847) teacher in school presented with 3 days Hx of jaundice ( they wrote it ictric as if it
will a differacne) & abdominal pain , N+V , 4 of school student had the same
illness in lab what is true regarding this pt :

a- Positive for hepatitis A IgG


b- Positive hepatitis A IgM
c- Positive hepatitis B core
d- Positive hepatitis B c anti-body

The correct answer is b

848) regarding weight gain in pregnancy what is true :

a- Pregnant woman should consume an average calorie 300-500 per day


b- Regardless her BMI or body weight she should gain from 1.5 – 3 lb which
represent the baby's growth
c- …………………………………………………………..
d- ……………………………………………………………….

Q is not complete

Weight gain during pregnancy :


- 100 – 300 Kcal / day , 500 Kcal / day in breastfeeding
- Wt. gain : 1 – 1.5 kg / month , 11 – 16 kg gain during pregnancy.
( for change from Ib to Kg : divided by 2.2)

……………………………………………………………………………………………………………………………………………………..
849) what condition is an absolute contraindication of lactation :

a- Mother with open pulmonary TB for 3 month


b- Herpes zoster in T10 drematome
c- Asymptomatic HIV
d- …………………………………………….

The correct answer is c

……………………………………………………………………………………………………………………………………………………..
850) what is most sensitive for DX of duodenal ulcer :

a- Epigasteric pain starting 30-60 min after the meal


b- Epigasteric pain staring immediately after a meal
c- Increasing of pain when lying supine
d- Pain radiating to the back

The correct answer is d

……………………………………………………………………………………………………………………………………………………..
851) female at 40 yr with BMI >28 what is your management :

a- Reduce calorie in take to 800 /day


b- In general reduce calorie intake
c- ………………………………….
d- ………………………………………..

Q is not complete, but with these MCQs , the correct answer is b

852) 70 y/o female say that she play puzzle but for a short period she can't play
because as she develop headache when playing what u will exam for
a. astigmatism
b. glaucoma

the correct answer is a


……………………………………………………………………………………………………………………………………………………..

852) prophylaxis for meningitis ttt contact


a. cimitidine
b. rifampicin

the correct answer is b


……………………………………………………………………………………………………………………………………………………..
853)rheumatic fever pt has streptococcal pharingitis risk to develop another attack
a. trimesmore than normal
b. 100%
c. 50%

the correct answer is c

……………………………………………………………………………………………………………………………………………………..

854)child with unilateral nasal obstruct with bad odor


a. unilateral adenoid hypertrophy
b. FB

the correct answer is b

……………………………………………………………………………………………………………………………………………………..

855)PT. do colectomy for colon cancer routine follow up every


a. 6 months
b. 3 months
c. 9 months
d. 1 years

the correct answer is d

856) for patient with osteoarthritis , what is your initial action :

a. Intraarticular corticosteroid
b. Quadriceps strengthening exercise
c. Climb stairs exercise

The correct answer is b

……………………………………………………………………………………………………………………………………………………..

857) 70-y old female patient with osteoporosis , what is her T score :

a. (-2.5)
b. (-1)
c. (1)
d. (2)

The correct answer is a

……………………………………………………………………………………………………………………………………………………..
858 ) diabetic patient , diagnosed 2 weeks back came to your clinic at scheduled
appointment supposed to be at 10:00 AM but because you were having another
complicated case , he had to wait for more than an hour , and he was extremely
angry , what u will do :

a. be empathetic as this anger is mostly because of the new morbidity diagnosed


at this patient
b. you start your talk with him by saying “I was having a hard case “
c. Don’t say anything regarding being late unless he brings it up
d. you star YOUR TALK WITH HIM BY SAYINH “you seem furious”

……………………………………………………………………………………………………………………………………………………..
859) 1st step in epidemic study is :

a. verifying diagnosis

Q is not complete

860) Randomized control trials become stronger if :

a. you follow more than 50% of those in the study

b. systematic assignment predictability by participants

Q is not complete , but with these MCQs , the correct answer is b

……………………………………………………………………………………………………………………………………………………..

861) child with multiple painful swellings on the dorsum of hands , feet , fingers and
toes ,his CBC showed Hb =7,RBC’s on peripheral smear are crescent shaped , what
is your long-term care :

a. corticosteroids

b. penicillin V
c. antihistaminic

the correct answer is b , this pt. is sickle cell anemia

……………………………………………………………………………………………………………………………………………………..

862) 54 y old patient , farmer , coming complaining of dry eye , he is smoker for 20
years and smokes 2 packs/ day , your recommendation :

a. advise him to exercise


b. stop smoking
c. wear sunscreen

The correct answer is b , smoking increase risk of dry eye (international study )

……………………………………………………………………………………………………………………………………………………..
863) patient is wearing contact lenses for vision correction since ten years , now
coming c/o excessive tearing when exposed to bright light , what will be your
advice to him :

a. wear hat
b. wear sunglasses
c. remove the lenses at night
d. saline eye dros 4 times / day

The correct answer is d ( I'm not sure )

864) management of mild epistaxis :

a. pinching the fleshy part of nose while telting head foreword

……………………………………………………………………………………………………………………………………………………..

865) most common site of malignancy in paranasal sinuses :

a. Maxillary sinus ( correct )

…………………………………………………………………………………………………………………………………………………….
.

866) patient with hypertension , what is the best non-pharmacological method to


lower the elevated blood pressure :

a. weight reduction ( correct )


……………………………………………………………………………………………………………………………………………………..

867) newly diagnosed patient with hyper tension having Na=147 , K=3 , what is the
most likely cause of his secondary hypertension :

a. hyperaldosteronism ( correct )

……………………………………………………………………………………………………………………………………………………

868) 50 year old patient , diagnosed with hypertension , he is used to drink one glass
of wine every day , he is also used to get high Na and high K intake , his BMI is
30kg/m , what is the strongest risk factor for having hypertension in this patient :

a. wine

b. high Na intake

c. high K intake

d. BMI=30

the correct answer is d , ( I'm not sure )

869) all of the following tests are necessary to be done before initiating lithium except:

a. liver function tests

……………………………………………………………………………………………………………………………………………………..

870) in dementia , best drug to use :

a. haloperidol

b. galantamine

the correct answer is b

treatment of dementia is cholinesterase inhibitor ( galantamine, donepezil,


rivastigmine, and tracing )

……………………………………………………………………………………………………………………………………………………..
871) most important point to predict a prognosis of SLE patient : ??

a. degree of renal involvement

b. sex of the patient

c. leucocyte count

the correct answer is a

…………………………………………………………………………………………………………………………………………………….
.

872) what is the antibiotic that you will give if you will do appendectomy :

a. doxycycline

b. ceftriaxone

c. metronidazole

d. cefuroxime

The correct answer is d

873) most common site for ectopic pregnancy :

a. fallopian tubes

…………………………………………………………………………………………………………………………………………………….
.

874)child with low grade fever , rash , pharingitis , and tender splenomegally , +ve
EBV , Dx :

a. infectious mononeucleosis

…………………………………………………………………………………………………………………………………………………….
.

875) best early sign to detect tension pneumothorax :

a. tracheal shift
b. ditended neck veins

c. hypotensionv

the correct answer is b , I'm not sure , I think tracheal shift is the late sign .

……………………………………………………………………………………………………………………………………………………..

876) First sign of magnisium sulfate toxicity is :


a. Loss of deep tendon reflex

……………………………………………………………………………………………………………………………………………………..

877) Patient with mashroom toxicity will present with


a. Constipation
b. Hallucination
c. Anhydrosis

the correct answer is b

878) Child ate overdose of iron , best immediate management


a. Gastric lavage
b. Induce vomiting manually
c. Emetic drugs
d. Ipecac

the correct answer is a

……………………………………………………………………………………………………………………………………………………..

879) Status epilepticus is :


a. Continuous sizure activity more than 30 min without regaining consciousness

…………………………………………………………………………………………………………………………………………………….
.

880) First sign of increased ICP is :


a. contralateral papillary dilatation
b. Ipsilateral papillary dilatation
c. Decrease level of consciousness

the correct answer is c


……………………………………………………………………………………………………………………………………………………..

880) Patient with history of head trauma unable to move the fork to his mouth , where
is the lesion:
a. cerebellum
b. frontal lobe
c. temporal

the correct answer is a

……………………………………………………………………………………………………………………………………………………..

881)Patient with lumbar canal stenosis , I forget the presentation , what is the ttt:
a. Physiotherapy
b. Lumbar dicsectomy

Q IS NOT COMPLETE
Mild to moderate : NSAID + physiotherapy
Advance : epidural corticosteroid
Refractory : surgical laminectomy

882)Most common cause of otorrhia :


a. Csf leak
b. Acute otitis media
c. Cerumen leak

The correct answer is b

……………………………………………………………………………………………………………………………………………………..

883)Most common symptoem of acute otitis media :


a. Pain
b. Discharge
c. Tinnitus
d. Vertigo

The correct answer is a

……………………………………………………………………………………………………………………………………………………..

884)12 yo boy is mocked at school because he is obese , ate a lot of pill to sleep and
never wake up again , best management is :
a. Refer to mental professional
b. Tell him that most kid grow out before they grow up
c. Advice healthy food
……………………………………………………………………………………………………………………………………………………..

885)12 yo boy brought by his parent for routine evaluation , his is obese but otherwise
healthy , his parents want to measure his cholesterol level , what is the best
indicator of measuring this child cholesterol :
a. His parent desire
b. Family hx of early cva
c. High bmi

The correct answer is b

886)Diabetic pt. have neovasclarization and vetrous hemorrhage , next step :


a. Refer to ophthalmologist

……………………………………………………………………………………………………………………………………………………..

887)Histopatholgy of resected inflamed appendix will show :


a. Neutrophil in muscular wall
b. Lymphoid hyperplasia with multineucleated gaint cel

The correct answer is a

……………………………………………………………………………………………………………………………………………………..

888)A child had bee bite presented after 18 hour with left arm erythema and itching
and…. Long scenario , what to do:
a. Antihistaminic
b. Oral steroid
c. Subcutaneous epinephrine

The correct answer is a

……………………………………………………………………………………………………………………………………………………..

889)Old man with urinary incontinence , plapable bladder after voiding , urgency and
sense of incomplete voiding dx;
a. Stress incontenece
b. Overflow
c. Reflex
The correct answer is b

……………………………………………………………………………………………………………………………………………………..

890)Ttt of scabis :
a. Permethrin

891)Picture , hyperkeratotic , scaly lesion over the extensor surface of knee and
elbow , what to do to avoid exacerbation :
a. Steroid
b. Avoid sun exposure
c. Avoid trauma

The correct answer is a

……………………………………………………………………………………………………………………………………………………..

892)Which is true about allopuranol :


a. Good if given during acute gout
b. Uricisoric
c. Reduce the chance of uric acid stone
d. Can be antagonize by salysilate

The correct answer is c

……………………………………………………………………………………………………………………………………………………..

893)man walking in street and saying bad words to stranger , he is not aware of his
conditiond , what is the description :
a. flight of idea
b. insertion of idea
c. loosening of association

……………………………………………………………………………………………………………………………………………………..

894)which of the following suggest that thyroid nodule is bengin rather that
malignant:
a. history of childhood head and neck radation
b. hard consistency
c. lemphadenopathy
d. presense of multiple nodules
the correct answer is d
thyroid cancer : ( cold and solid nodule )

895)child presented after sting bite with abd pain , arthalgia , lower extremity and
buttock rash , normal RFT , +ve fecal occult stool , dx :
a. HSP
b. Lyme diases

The correct answer is a

……………………………………………………………………………………………………………………………………………………..

896)best indicator for labor progress is :

a. frequency of contractions

b. strength of contractions

c. descent of the presenting part

the correct answer is c

……………………………………………………………………………………………………………………………………………………..

897)lactating mother is taking Phenobarbital for sizure prophylaxsis , what to advice


her :
a. stop breast feeding immediately
b. breast feed after taking the pill by 8 hour
c. start weaning
d. does not matter

the correct answer is a

……………………………………………………………………………………………………………………………………………………..

898)best single way to reduce high blood pressure is :


a. smoke cessation
b. decrease lipid level
c. reduce weight
the correct answer is c

……………………………………………………………………………………………………………………………………………………..
899)drug of choice for supraventricular tachycardia is :
a.adenosine

900)patient with blood group A had blood transfusion group B , the best statement
that describe the result is :
a. type IV hypersensitivity
b. inflammatory reaction ….
c. ……….
The correct answer is : It is type 2 hypersensitivity

……………………………………………………………………………………………………………………………………………………..

901)a nurse gave blood transfusion through CVP , 2 hours later the patient is comatose
and unresponsive , dx :
a. Septic shock
b. Blood group mismatch

The correct answer is a ( I'm not sure )

……………………………………………………………………………………………………………………………………………………..

902) 100% O2 given for prolonged periods can cause all except:
a. Retrosternal Pain
b. Seizures
c. Depression
d. Ocular Toxicity
e. ???

The correct answer is c ( I'm not sure because choice e is not mention )
903) Organophosphorus poisoning, what is the antidote?
a. Atropine
b. Physostigmine
c. Neostigmine
d. Pilocarpine
e. Endrophonium

The correct answer is a

……………………………………………………………………………………………………………………………………………………..

904) Which of the following pulse character goes with the disease?
a. Collapsing pulse  Severe anemia
b. Pulsus Alternans  Premature ventricular complexes
c. Slow rising Pulse  Mitral stenosis
d. Pulsus Bisferens  Mitral Regurgitation
e. Pulsus paradoxus  I think Aortic stenosis or regurgitation

The correct answer is a

PLUSE DEFINITION CAUSES

Collapsing pulse Pulse with fast upstroke and fast 1- Sever anemia
downstroke 2- Aortic regurge
3- Thyrotoxicosis

Pulse alternans Altrnans weak and strong pulse 1- LV dysfunction

Pulsus paradoxus Decline systolic more than 10 mm Hg 1- Cardiac tamponade


during inspiration 2- Asthma & COPD

Pulsus parvus et Weak and delayed pulse 1- Aortic stenosis


tardus

Slow rising pulse Slow upstroke pulse 1- Aortic stenosis

Pulsus bisferiens pulse characterized by two strong 1- Aortic reguirge


systolic peaks separated by a 2- AR + AS
midsystolic dip

905) Entamoeba histolytica cysts are destroyed best by:


a. Boiling
b. Iodine added to water
c. Chlorine added to water
d. Freezing

The correct answer is a

……………………………………………………………………………………………………………………………………………………..
906) A scenario about an old male with symptoms suggesting parkinsonism such as
difficulty walking, resting tremors and rigidity in addition to hypotension. Then he
asks about what is the most common presenting symptom of this disease
a. Rigidity
b. Tremors
c. Unsteady Gait
d. Hypotension
e. ???
The correct answer is b

……………………………………………………………………………………………………………………………………………………..
907) Regarding postpartum Psychosis:
a. Recurrences are common in subsequent pregnancies
b. It often progresses to frank schizophrenia
c. It has good prognosis
d. It has insidious onset
e. It usually develops around the 3rd week postpartum

The correct answer is a

……………………………………………………………………………………………………………………………………………………..
908) A pregnant female develops lesions on the vulva and vagina and she was
diagnosed as genital herpes, what should be included in her future health care?
a. Cesarian section should be done if the lesions did not disappear before 2
weeks of delivery date
b. Oral acyclovir to treat herpes
c. Termination of pregnancy because of the risk of fetal malformations
d. Avoidance of sexual intercourse for 1 month after the healing of the lesions
e. ?????
The correct answer is b

HSV in pregnant treated by : oral acyclovir 400 mg TID for 5-7 days . if HSV was
present at time of labor : c – section
909) A picture of 2 warts in the index finger.
Diagnosis is  Viral Warts

……………………………………………………………………………………………………………………………………………………..

910) What is the most common sequalae of acute pancreatitis ?


a. Abscess
b. Pseudocyst
c. Bowel obstruction

The correct answer is b


Most common complication of acute pancreatitis: pancreatic pseudocyt and
fistula formation

……………………………………………………………………………………………………………………………………………………..

911) An old patient with history of cerebrovascular disease & Ischemic heart disease,
presents with a pattern of breathing described as: A period of apnea followed by
slow breathing which accelerates & becomes rapid with hyperpnea & tachycardia
then apnea again. What is this type of breathing?
a. Hippocrates
b. Chyene-stokes breathing
c. Kussmaul breathing
d. One type beginning with O letter and contains 3 letters only
e. ????

The correct answer is b

1- Chyene-stokes respiration : rapid deep breathing phase followed by period of


apnea , present with heart failure , stroke, brain trauma , also can be with sleep
or high altitude

2-kusmmaul's breathing : rapid and deep breathing . present with metabolic


acidosis particularly in diabetic ketoacidosis

912) A patient with mixed 1st & 2nd degree burns in head & neck region, what is the
most appropriate management?
a. Apply silver sulfadiazine and cream to all burned areas,cover them and admit
to hospital
b. Apply cream to 2nd degree burns and cover them, give IV fluids
c. Debridement of 2nd degree burns and ….
d. Apply silver sulfadiazine then Vaseline ointment to all areas then discharge the
patient

The correct answer is a


1sr degree : epidermis + painful ( no blister )
2nd degree : epidermis + partial dermis + painful _ blister
3rd degree : epidermis + full thickness dermis + painless

Rule of nine to estimate BSA :


1- Head and neck : 9%
2- Each upper arm : 9%
3- Each loer limb : 18%
4- Each post. & ant. Trunk : 18%
5- Perineum & genitalia : 1%

-Silver sulfadiazine and mafenide used for all types of burn


-supportive : tetanus and I.V nacrotic can be used for all types of burn
- I.V fluid use only in ( > 10% of BSA) in 2nd & 3rd degree of burn according parkland
formula : ( fluid in first 24h = 4 * wt. in kg * BSA . 50% in first 8 hours and another
in next 16 hours )

In this patient 1st deree and 2nd degree < 10% ( head and neck 9%) : so, not use
fluid

……………………………………………………………………………………………………………………………………………………..

913) 2 years old child with ear pain & bulging tympanic membrane, what is the
diagnosis?
a. Otitis media
b. Otitis externa
c. Otomycosis
d. Bullous myringitis

The correct answer is a

914) Besides IV fluids, what is the most important drug to be given in anaphylaxis?
a. Epinephrine
b. Steroids
c. ??? Other choices

The correct answer is a

……………………………………………………………………………………………………………………………………………………..

915) An adult patient in 20s or 30s of age presents by history of 1 month of fever, 5
days of headache & 2 days of altered sensorium. On examination there is nuchal
rigidity, then there is a table showing investigations which include
Hb: 10 g/dl
Blood WBC: 18,000
CSF Examination: WBCs elevated: 77% lymphocytes, 33% Neutrophils. Protein ???.
Glucose??

What is the diagnosis?

a. Viral meningioencephalitis
b. Tuberculous meningitis
c. Ph…. Meningitis
d. ???

Q is not complete , but with this scenario and MCQs , The correct answer is b

916) Regarding chronic fatigue syndrome, which is true?


a. Antibiotics may reduce the symptoms
b. Antidepressants may reduce the symptoms
c. Rest may reduce the symptoms
d. ???
e. ????

Q IS not complete , but with these MCQs , the correct answer is b.

chronic fatigue syndrome:


characterizes by profound mental and physical exhaustion. In associated with
multiple system and neurospsychiatric symptoms that last at least 6 mounth.
Must be new ( not life long ) , must not be relived by rest, and must result in
greater than 50% reduction in previous activity. Presentation with 4 or more of
the following : poor memory / concentration, myalgia, arthalagia, sore throat,
tender lymph node, recent onset headach, unrefreshing sleep, excessive
tiredness with exercise. Treatment by : cognitive and excercise therapy .also,
diet, physiotherapy, dietary supplements, antidepressants.

……………………………………………………………………………………………………………………………………………………..

917) A patient complains of dry eyes, a moisturizing eye drops were prescribed to him
4 times daily. What is the most appropriate method of application of these eye
drops?
a. 1 drop in the lower fornix
b. 2 drops in the lower fornix
c. 1 drop in the upper fornix
d. 2 drops in the upper fornix

……………………………………………………………………………………………………………………………………………………..

918) A patient with penetrating abdominal stab wound. Vitals are: HR 98, BP 140/80,
RR 18. A part of omentum was protruding through the wound. What is the most
appropriate next step:
a. FAST Ultrasound
b. DPL (Diagnostic peritoneal lavage)
c. Explore the wound
d. Arrange for a CT Scan
e. Exploratory laparotomy
The correct answer is e
919) A patient presents with loin pain radiating to the groin. Renal stones are
suspected. What is the test that has the most specificity & sensitivity in
diagnosing this condition?
a. Noncontrast spiral CT scan of the abdomen
b. Ultrasound
c. KUB
d. Intravenous pyelography (IVP)
e. Nuclear Scan

The correct answer is a

……………………………………………………………………………………………………………………………………………………..

920) An old patient presents with history dizziness & falling down 1 day ago
accompanied by history of Epigastric discomfort. He has very high tachycardia I
think around 130-140 and BP 100/60. What is the diagnosis?
a. Peptic ulter
b. GERD
c. Leaking aortic aneurysm
d. ???

The correct answer is c

……………………………………………………………………………………………………………………………………………………..

921) A child is about to be given flu vaccine, what allergy should be excluded before
giving the vaccine?
a. Chicken
b. egg
c. Fish
d. ???

The correct answer is b

……………………………………………………………………………………………………………………………………………………..

922) Which of the following is proven to reduce the incidence of cancer?


a. Salt
b. Mineral water
c. Vitamin D
d. Fiber

The correct answer is c


923) A female patient with history of cyclic abdominal pain, inability to conceive, heavy
menses, and examination showed tenderness & nodularity in uterosacral
ligaments. What is the diagnosis?
a. Endometriosis

……………………………………………………………………………………………………………………………………………………..

924) A female patient on the 3rd week postpartum. She says to the physician that the
frequently visualizes snakes crawling to her baby’s bed. She knows that it is
impossible but she cannot remove the idea from her head. She says she wakes up
around 50 times at night to check her baby. This problem prevents her from
getting good sleep and it started to affect her marriage. What is this problem she
is experiencing?
a. An obsession
b. A hallucination
c. A postpartum psychosis
d. A Delusion

The correct answer is a


Obsession : persistent, unwanted, and intrusive ideas , thoughts, impulses or
images

……………………………………………………………………………………………………………………………………………………..

925) Regarding screening for cancer, which of the following is true?


a. Screening for cervical cancer had decreased in recent years
b. Screening for breast cancer had decreased in recent years
c. Screening for Colorectal cancer is inadequate for the high-risk groups
d. Screening for lung cancer has reduced the mortality rate of lung cancer
e. Screening for tobacco use is now adequately done by health professionals

……………………………………………………………………………………………………………………………………………………..

926) Regarding postpartum depression, what is the most appropriate intervention to


reduce the symptoms?
a. Include family in the therapy
b. Isolation therapy
c. Add very low doses of impiramine
d. Encourage breastfeeding

The correct answer is a

927) Which of the following is a side effect of Buprione, a drug used to help smoking
cessation:
a. Arrhythmia
b. Xerostomia
c. Headache
d. Seizure

The correct amswer is d

……………………………………………………………………………………………………………………………………………………..
928) A 60 years old patient with history of heart attack 6 weeks ago, complaining of not
getting enough sleep. Psychiatric evaluation is unremarkable for depression or
anxiety, what should be given to this patient?
a. Amytriptiline
b. Buspirone
c. Buprione
d. Zolbidim

The correct answer is d

……………………………………………………………………………………………………………………………………………………..
929) A patient is asked to face the wall, bend his waist, and let his hands hang down
without support. This test is used as a screening tool for which of the following?
a. Lower limb asymmetry
b. Rectal prolapsed
c. Scoliosis
d. ????

The correct answer is c


This test is called for (Adam's Forward Bend Test )

930) A patient presents with sore throat, cervical lymphadenopathy, mild


splenomegaly, EBV antibodies are +ve. What is the diagnosis?
a. Infectious Mononucleosis (EBV )

……………………………………………………………………………………………………………………………………………………..

931) An old patient presents with knee pain, limitation of movement and quadriceps
muscle atrophy. On examination there is limited range on movement, pain on
movement & Crepitus. What is the diagnosis?
a. Osteoarthritis
b. Rheumatoid arthritis
c. Ankylosing Spondylitis

The correct answer is a

……………………………………………………………………………………………………………………………………………………..

932) A patient presents with long time history of knee pain suggestive of osteoarthritis.
Now he complains of unilateral lower limb swelling and on examination there is
+ve pedal & tibial pitting edema. What is the next appropriate investigation?
a. CXR
b. ECG
c. Echocardiography
d. Duplex ultrasound of lower limb

The correct answer is d


Osteoarthritis relief by rest . So, immobility pt. can lead to DVT

……………………………………………………………………………………………………………………………………………………..

933) Alopecia is a side effect of which antiepileptic?


a. Phenytoin
b. Carbamazepine
c. Na Valproate
d. ???

The correct answer is c


1-phenytoin : gingival hyperplasia, hirsuteism, ataxia
2-carbamazepine : agranulocytosis, hepatotoxicity, aplastic anemia
3- Na Valproate : transient hair loss.

934) Which of the following is an indication for tonsillectomy?


a. Sleep apnea
b. Asymptomatic large tonsils
c. Peripharygeal abscess
d. Retropharyngeal abscess

The correct answer is a

……………………………………………………………………………………………………………………………………………………..

935) Which of the following is a gram –ve rods that grow on charcoal yeast agar?
a. Legionella

……………………………………………………………………………………………………………………………………………………..

936) A 7 years old child had history of chest infection which was treated with
antibiotics. The patient presented 6 weeks after cessation of antibiotics with
abdominal pain, fever and profuse watery diarrhea for the past month. Which of
the following organisms is responsible for the patient’s condition?
a. Giardia Lamblia
b. Clostridium Difficile
c. Escherichia coli
d. Clostridium Perfringens

The correct answer is b

……………………………………………………………………………………………………………………………………………………..

937) Which of the following features of ulcerative colitis distinguishes it from crohn’s
disease
a. Possible malignant transformation
b. Fistula formation
c. Absence of granulomas
d. Colon involvement

The correct answer is a

938) A very very long scenario about a female patient with vaginal discharge
“malodorous watery in character” with pH of 6 & +ve clue cells but there is no
branching pseudohyphe. (He is telling you the diagnosis is vaginosis & there is no
fungal infection) Then he asks about which of the following drug regimens should
NOT be used in this paitent:
a. Metronidazole (PO 500 gm for 7 days)
b. Metronidazole (PO 2 large dose tablets for 1 or 2 days)
c. Metronidazole (IV or IM …..)
d. Miconazole ( PO …..)
e. Clindamycin (PO …..)

The correct answer is d


This case is " bacteria vaginosis" and treatment by : metradinazole or
clindamycine . avoid miconazole because it is antifungal

……………………………………………………………………………………………………………………………………………………..
939) What is the condition that produces malodorous watery vaginal discharge with
+ve clue cells in wet mount preparation slides:
a. Bacterial vaginosis
b. Vaginal Candidiasis
c. Trichomonas vaginalis
d. Gonorrhea
The correct answer is a

……………………………………………………………………………………………………………………………………………………..
940) Which of the following is the best treatment for Giardiasis:
a. Metronidazole

……………………………………………………………………………………………………………………………………………………..
941) A woman with 9weeks History of elevated erythematous wheals overall her
body , she also has lip swelling. There is no History of recent travel or food allergy
or drug ingestion, Diagnosis:
a. Chronic Angioedema & Urticaria
b. Contact Dermatitis
c. Solar Urticaria
d. Cholinergic Urticaria
e. Cold Urticaria
The correct answer is a
Chronic urticaria : if more than 6 months
Solar urticaria : due to sunlight
Cholinergic urticariae : due to brief increase in body temperature.
Cold urtiaria : due to exposure to cold
942) A 17 years old male with history of mild intermittent asthma. Attacks occur once
or twice weekly in the morning and no attacks at night. What should be the initial
drug to give?
a. Inhaled short acting B2 agonist as needed
b. Inhaled high dose corticosteroid as needed

The correct answer is a

……………………………………………………………………………………………………………………………………………………..
943) A child presented with sore throat. Culture from the throat revealed +ve
meningiococci. The patient is now asymptomatic. Which of the following should
be done?
a. Reassurance
b. Rifampicin oral for 7 days
c. IM ceftriaxone 1 dose
d. Ceftriaxone oral
e. ??????

The correct answer is c ( I'm not sure )


2-day regimen of oral rifampin or a single dose of IM ceftriaxone or a single dose
of oral ciprofloxacin).

……………………………………………………………………………………………………………………………………………………..
944) Which of the following increases the quality of the randomized controlled study &
make it stronger:
a. Systemic Assignment predictability by participants
b. Open Allocation
c. Including only the participants who received the full intervention
d. Following at least 50 % of the participants
e. Giving similar intervention to similar groups

The correct answer is a

……………………………………………………………………………………………………………………………………………………..
945) A scenario about an infant who presented with difficulty breathing and sweating,
examination shows hyperactive precordium with loud S2 and pansystolic murmur
grade 3/6 best heard in the left 3rd parasternal region. What is the Diagnosis?
a. VSD
b. Mitral Regurgitation
c. PDA
d. Large ASD
The correct answer is a

946) Post IHD , your advise :


a. Yoga
b. isotonic exercise

???????????

……………………………………………………………………………………………………………………………………………………..

947) very obese girl , what's the first step to reduce weight :
a. increase water intake
b. decrease calorie intake

the correct answer is b


……………………………………………………………………………………………………………………………………………………..

948) 40 y/o male , presented with large hands , hepatomegaly ….. diagnosis :
a. acromegaly
b. gigantism

the correct answer is a


…………………………………………………………………………………………………………………………………………………….
.
949) cousre of facial nerve when comes out from stylomastoid …
a. Deep to parotid gland & lateral to external carotid & retromandibular vein

……………………………………………………………………………………………………………………………………………………..
950) old female with hemorrhoids for 10 years , no complication , your action ?
a. observe
b. surgery
c. increase fiber diet

the correct answer is c

……………………………………………………………………………………………………………………………………………………..

951) femal about 30y c/o abdominal pain related to menses (scenario going with
endometriosis)…………next step in dx:
a. Laparoscopy
b. U/S
c. CT

The correct answer is a


Confirm diagnosis by laparoscopy , US & MRI can use but if normal can not
exclude endometriosis .
952) female about 30y with breast cancer (given cbc –chem. And reavel low hb and
hematocrite….) what is the next step in mangment:
a. Staging
b. Lumpectomy
c. Mastectomy
d. Chemotherapy

………………………………………………………………………………………………………………………………………………….....

953) pt with open angle glaucoma and k/c of COPD and DM ttt:
a. Timelol
b. betaxolol
c. Acetazolamide

the correct answer is c


………………………………………………………………………………………………………………………………………………….....

954) infant with runny nose and fever after that develop dysnea ,wheezing with
working accessory muscles and chest retractions with cyanosis, temp 38 dx:
a. Viral pnumonia
b. Bacterial pneumonia
c. Bronchiolitis

The correct answer is c

955) 34y female with HIV pap smear negative, about cervical cancer screening :
a. After 3m if negative repeat after 6m
b. After 6m …………………………… annually
c. After 1y………………………………….annually

The correct answer is b (I'm not sure ), may be c .

 Screening should begin no later than age 21.


 Screening should begin earlier than age 21 if the patient is sexually active. In this
case, it should start 3 years after initiation of vaginal intercourse.

 Once initiated, screening should be performed annually if a traditional, glass-


slide-based technique is used. If liquid medium is used, Pap smear screening may
be performed every other year.

 After age 30, for women who have had 3 consecutive, normal Pap smears,
screening frequency may be reduced to every two to three years.

 Women who are HIV positive, immunocompromised due to disease or


medication, or are DES daughters, should continue annual screening.

 Screening may stop following a total hysterectomy (including the cervix), if the
the patient is at low risk, and has had three consecutive normal Pap smears within
the last 10 years.

 High risk patients, including those with a history of cervical cancer, DES
exposure in-utero, HIV positive, immunocompromised from medication, and
those tested positive for HPV, should continue to be screened indefinitely.

 Screening may stop after age 70, if the patient is low risk, and has had three
normal Pap smears over the last 10 years.

 Screening may be omitted in the case of women with life-threatening or other


serious illness

………………………………………………………………………………………………………………………………………………….....
956) which drug increase incidence of reflux oesophagitis:
a. Theophylline
b. Amoxicilline
c. Metoclopramide
d. Rantidine
e. Lansoprazole

The correct answer is a


957) most effective ttt of cluster headach:
a. Ergotamine nebulizer
b. S/C Sumatriptan
c. 100% O2
d. IV Verapamil

The correct answer is c


………………………………………………………………………………………………………………………………………………….....
958) old pt. with HTN and migraine ttt:
a. B blockers
b. ACE I
c. Ca blockers

The correct answer is a

………………………………………………………………………………………………………………………………………………….....
959) lactating women 10 days after delivery developed fever ,malaise, chills tender Lt
breast with hotness and small nodule in upper outer quadrant with axillary LN
.Leukocyte count was 14 *10/L dx:
a. Inflammatory breast cancer
b. Breast abscess
c. Fibrocystic disease
d. ????
???????

………………………………………………………………………………………………………………………………………………….....

960) newborn with fracture mid clavicle what is true:


a. Most cases cause serious complication
b. Arm sling or figure 8 sling used
c. Most patient heal without complications

The correct answer is c


-most clavicles fracture in newborn no need to treatment apart from
careful handling. If the fracture is displaced and baby in pain, simple sling
is require.

961) 70y male with osteoporosis the T score of bone densometry would be :
a. 3.5
b. -2.5
c. -1
d. 2
e. 3.5

The correct answer is b


- The T score is the number of standard deviation (SD ) a patient's BMD deviated
from the mean.
- T score > (- 1) : normal BMD
- T score (-1 to -2.5) : ostopenia
- T score < ( -2.5 ) : osteoprosis

………………………………………………………………………………………………………………………………………………….....
962) what is the most complication after hesterectomy:
a. Ureteral injury
b. Pulmonary embolism
c. Haemorrhage

The correct answer is c


………………………………………………………………………………………………………………………………………………….....
963) young female with BMI 18 , fine hair allover body , feeling of she is fat ,doesn’t
eat well with excessive exercise………………..
a. Anorexia nervosa?
b. Body dysmorphic disorder
c. Bulimia nervosa

The correct answer is a


………………………………………………………………………………………………………………………………………………….....
964) what's advantage of mature human milk over cow's milk:
a. More protein
b. More Iron content
c. More calories
d. More fat

The correct answer is b

………………………………………………………………………………………………………………………………………………….....
965) young male c/o of deformity of jaw .past h/o of nasoplasty and blepharoplasty
O/E nothing abnormal ………..dx:
a. Body dysmorphic syndrome
966) the most common cause of non-traumatic subarachnoid haemorrhage:
a. Rupture aneurysm
b. Vessels abnormality
c. Hypertension

The correct answer is a


-Most common causes of subarachnoid hemorrhage are trauma and berry
aneurysm
………………………………………………………………………………………………………………………………………………….....
967) Pt. with chest pain and s.o.b decreased by leaning forword . O/E friction rub and
increased JVP >>>> (a case of pericardial effusion) ECG will show:
a. ST changes
b. Low voltage
c. Increase PR interval

ECG changes during pericarditis : diffuse ST elevation , PR depression followed by


t-wave intervention , and low voltage.
………………………………………………………………………………………………………………………………………………….....
968) The causative organism of psudomembranous colitis is:
a. Clostridium difficile

………………………………………………………………………………………………………………………………………………….....
969) scenario about arthritis (I couldn't remember the details ) but Joint aspirate
reveal Gram negative diplococci …..dx:
a. Nisseria gonorrhea

N. gonorrhoeae : gram negative diplococci and can also cause conjunctivitis,


pharyngitis, proctitis or urethritis, prostatitis and orchitis.

……………………………………………………………………………………………………………………………………………….....
970) Vertigo & …….. is caused by which of the following drug ?
a. Aphotercine B
b. Pnicilline reaction
c. INH

……………………………………………………………………………………………………………………………………………….....

971) Baby with red macule & dilated capillary on the rt side of the face

Sturge-Weber Syndrome or Nevus Flammeus (one of them will come)

Don’t choose milia or cavernous haemangioma

972) A 25 year old secondary school teacher that every time enters the class starts
sweating and having palpitation, she is a fired to give wrong information and be
unparsed. What is the diagnosis:
a. Specific Phobia
b. Social Phobia

The correct answer is b


……………………………………………………………………………………………………………………………………………….....
973) At which chromosome is the cystic fibrosis gene:
a. Long arm chromosome 7
b. Short arm chromosome 7
c. Long arm chromosome 8
d. Short arm chromosome 8
e. Long arm chromosome 17

The correct answer is a


……………………………………………………………………………………………………………………………………………….....

974) Regarding hepatocellular carcinoma (Hepatoma) Which is true:


a. More common in females
b. ..
c. ..
d. The most common cancer in Africa and Asia
e. Increase risk in chronic liver disease

The correct answer is e , chronic disease especially Hepatitis B

……………………………………………………………………………………………………………………………………………….....

975) A patient is having a 2 year history of low interest in live, he doesn’t sleep well
and can’t find joy in life, What is the most likely diagnosis:
a. Dysthymia
b. Major depressive disorder
c. Bipolar disorder
d. ..
e. ..

The correct answer is a

976) What is the mechanism of OCD drugs:


a. Increase availability of Serotonin
b. Decrease production of Serotonin
c. Increase production of Serotonin
d. .. Serotonin
e. .. Serotonin

The correct anser is a

……………………………………………………………………………………………………………………………………………….....

977) What is the MOST accurate test to detect early pregnancy:


a. ..Serum BHCG
b. ..
c. Urine pregnancy test
d. Ultrasound
e. ..
The correct answer is a
……………………………………………………………………………………………………………………………………………….....

978) A case of a patient with thin cervix and little amount of cervical mucus, how
would you treat her: ?????
a. Estrogen injections

……………………………………………………………………………………………………………………………………………….....

979) The most common cause on chronic interrupted rectal bleeding is:
a. Diverticulosis
b. Hemorrhoids

The correct answer is b


……………………………………………………………………………………………………………………………………………….....

980) A child is having a croup early morning, the most common cause is:
a. Post nasal drip

981) A burn patient is treated with Silver Sulfadiazine, the toxicity of this drug can
cause:
a. Lycosytosis
b. Neutropenia
c. Electrolyte disbalance
d. Hypokalemia
e. ..

The correct answer is b , side effect of sulfadiazine

……………………………………………………………………………………………………………………………………………….....

982) A case scenario about a patient who had appendectomy, after that he has
abdominal pain and constipation and absent bowel sound, the most likely cause
is:
a. Ilus paraticus

Q is not complete
……………………………………………………………………………………………………………………………………………….....

983) A case scenario describing a patient who had a right chest trauma than developed
tension pneumothorax, the immediate management is:
a. Insert a needle in the second intercostal space
b. Insert a needle in the fifth intercostal space
c. Give IV antibiotics
d. ..
e. Insert a tube in the fifth intercostal space

The correct answer is a


……………………………………………………………………………………………………………………………………………….....

984) A patient that have a penile ulcer that healed after that he developed a palm and
sole rash, the most likely cause is:
a. Syphilis

Q is not complete

985) A case scenario about a patient who has on and off episodes of abdominal pain
and was found to have multiple gallstones, the largest is 1 cm and they are not
blocking the duct, What will you do:
a. Give pain killers medication
b. Remove gallbladder by surgery

Q is not complete , but with these MCQs , the correct answer is b

……………………………………………………………………………………………………………………………………………….....

986) Most common symptoms of soft tissue sarcoma :


a. Paralysis
b. On growing mass
c. Pain
d. ..

The correct answer is b


……………………………………………………………………………………………………………………………………………….....

987) Patient use illegal drug abuse and the blood show RNA virus. Which hepatitis
a. A
b. B (DNA)
c. C
d. E

The correct answer is c , RNA virus transmitted by blood id HCV


……………………………………………………………………………………………………………………………………………….....
988) A case scenario about bronchial carcinoma, which is true: ????
a. The most common cancer in females
b. Squamous cell carcinoma spreads faster
c. Adenocarcinoma is usually in the upper part
d. Elevation of the diaphragm on the x-ray means that the carcinoma has
metastasize outside the chest
e. Bronchoscopy should be done

The correct answer is c

989) old female with pubic itching with bloody discharge, then she developed pea
shaped swelling in her labia, most likely:
a. Bartholin cyst
b. Bartholin gland carcinoma
c. Bartholin abscess
d. ..

The correct answer is c


……………………………………………………………………………………………………………………………………………….....

990) Patient has decrease visual acuity bilateral , but more in rt side , visual field is not
affected , in fundus there is irregular pigmentations and early cataract formation .
what you will do
a. Refer to ophthalmologist for laser therapy
b. Refer to ophthalmologist for cataract surgery
c. ..

The correct answer is a

……………………………………………………………………………………………………………………………………………….....

991) Inflammatory bowel disease is idiopathic but one of following is possible


underlying cause:
a. Immunological

……………………………………………………………………………………………………………………………………………….....

992) pregant lady deliverd Anencephalytillbirth occurance of neural tube defect in next
pregnancy
a. 8%
b. 2%
c. 10%
d. 20%
The correct answer is b , ( 2 – 4 % )
…………………………………………………………………………………………………………………………………………………

993) typist theaner atrophy ,+ve tephlen sign


a. Median nerve
b. Ulnar
c. Brachial
The correct answer is a
994) painful pile
a. Exison daringe
b. Sitz path and steriod supp
c. Ab
d. Fiber food and analgesics
The correct answer is a
…………………………………………………………………………………………………………………………………………………

995) 39 years old HIV patient with TB recive 4 drugs of tt after one month
a. Continue 4 drugs for 1 years
b. Cintinue isoniazide for 9months
c. Contiue isonizide for 1 year

The correct answer is a


…………………………………………………………………………………………………………………………………………………
996) female not married with normal investigation except FBS=142.
RBS196. so ttt
a. give insulin subcutaneous
b. advice not become married
c. barrier contraceptive is good
d. BMI control

The correct answer is d

…………………………………………………………………………………………………………………………………………………
997) Entamoeba histolytica cysts are destroyed best by:
a. Boiling
b. Iodine added to water
c. Chlorine added to water
d. Freezing

The correct answer is a


998)A scenario about an old male with symptoms suggesting parkinsonism such as
difficulty walking, resting tremors and rigidity in addition to hypotension. Then he asks
about what is the most common presenting symptom of this disease
a. Rigidity
b. Tremors
c. Unsteady Gait
d. Hypotension
E. ???
The correct answer is b

……………………………………………………………………………………………………………………………………………………..
998) Regarding postpartum Psychosis:
a. Recurrences are common in subsequent pregnancies
b. It often progresses to frank schizophrenia
c. It has good prognosis
d. It has insidious onset
e. It usually develops around the 3rd week postpartum

The correct answer is a

……………………………………………………………………………………………………………………………………………………..
999) Gastrictomy post-op 1 day. He have temperature 38.8 & pulse 112. What is the
most common cause ?

a. wound infection.

b. inflammatory mediator in the circulation.

c. UTI

d. normal

the correct answer is a or b

most common causes of post-operation fever : ( 6 w )

1- W ound
2- Wine = pneumonia or atlectasia
3- Water = UTI
4- What happen = transfusion
5- Walk = DVT
6- Wonder drug = asprin,erythromycin , isonizide
Immediately ( 24 h ) : post-op inflammation , wound infection , transfusion ,
streptococcus
Acute ( 3 – 7 days ) : pneumonia

1000) Malaria ?

- the most common cases is caused by Plasmodium falciparum.

……………………………………………………………………………………………………………………………………………………
..

1001 ) Polyartheralgia rhumatica. What is the thing that suggest it rather than  ESR
& C-reactive protein:

a. proximal muscle weakness

b. proximal muscle tenderness

the correct answer is a

……………………………………………………………………………………………………………………………………………………..
1002) The cause of insulin resistance in obese is:

a.  insulin receptors kinase activity

b.  number of insulin receptor

c.  circulation of anti-insulin

d.  insulin production from the pancreas

e.  post-receptor action

the correct answer is e (I'm not sure )


1003) Pt came with total bilirubin: 5.8 & direct bilirubin: 0.4

a. Guillain–Barré syndrome

b. Dubin-junson syndrome

c. Biliary sclerosis

d. Crigler-Najjar syndrome

e. Gilbert Syndrome

the correct answer is e


……………………………………………………………………………………………………………………………………………………

1004) Pt complaining of hypotension & bradycardia. Electrolytes show: Na, K, Cl,
Urea. So the cause of this is:

a. hyponatremia

b. hyperkalemia

c. hyperchloremia

d. uremia

the correct answer is a ( not sure )

‫تمنياتي للجميع بالتوفيق‬

‫ محمد الحربي‬:‫أخوكم‬

Orthopaedic – Qassim University

You might also like